BIOC 385 Exam 4 Master Set

Pataasin ang iyong marka sa homework at exams ngayon gamit ang Quizwiz!

29. (6 pts) Which FOUR of the following gene variants are good candidates to function as Thrifty Genes?

(2) Low basal level expression of Uncoupling Protein 1. (3) Increased catalytic activity of fatty acid synthase. 6) Gherlin mutant with a high affinity for the ghrelin receptor (9) High activity lipoprotein lipase on the membrane of adipocytes

What are the names of the four neurons labeled A, B, C, and D in the figure to the right?

(A) POMC, (B) NPY/AGRP, (C) Anorexigenic (D) Orexigenic

When 15N aspartate is fed to animals, many N15 aa's appear in protein within a short time a) explain this observation and include the most likely enzyme reaction responsible for this finding b) what is the enzyme cofactor for this reaction, and what is the type of chemical bond that must be formed for the reaction to proceed?

(a) Aspartate is a common amino group donor for transamination, such as: Aspartate +α-Keto acid → Oxaloacetate +α amino acidBecause the α-keto acid in the equation can come from any amino acid, the 15N from aspartate is rapidly transferred to other amino acids. (b) The cofactor is pyridoxal phosphate, which forms a Schiff base between the cofactor carbonyl carbon and an amino group from either a lysine of the enzyme or a free amino acid

Cats were fasted overnight then given a single meal that contained all aa's except arginine. In 2 hours, their blood ammonia levels rose by 800%. A control group of cats were fasted then given the same meal but with arginine and showed no change in blood ammonia levels. a) fasting was required to obtain this difference in ammonia levels, why? b) what caused the ammonia levels to rise in the experimental group?

(a) Fasting lowered blood glucose, so that the fed amino acids are deaminated to form substrates for gluconeogenesis. (b) Arginine is required for the urea cycle, thus the experimental animals were unable to remove ammonia via the urea cycle

Kwashiorkor is a dietary deficiency disease characterized by decreased pigment in the skin and hair. a) what major food group is likely missing from the diet? b) what would you conclude if adding the missing food group to the diet had no effect on the skin and hair condition?

(a) Kwashiorkor is caused by a lack of protein in the diet. The decreased pigmentation is caused by a deficiency in tyrosine, which is the precursor to melanin pigments. Moreover, because phenylalanine is converted to tyrosine by phenylalanine hydroxylase, phenylalanine deficiency also contributes to the skin and hair condition. (b) In such a case, the individual may have a defect in an enzyme required to convert tyrosine to melanin

The hypoxanthine analog 6-mercaptopurine can be converted to 6-mercaptopurine-5'-monophosphate by the salvage pathway to generate a potent competitive inhibitor of adenylosuccinate synthetase and IMP dehydrogenase which generate AMP and GMP from IMP. Leukemia patients are sometimes treated with 6-mercaptopurine to decrease AMP and GMP production in rapidly dividing cancer cells. It has been found that the inhibitory effect of mercaptopurine on leukemia cell growth is enhanced if the treatment includes allopurinol, a specific inhibitor of the purine degradation enzyme xanthine oxidase. a) explain the improved efficacy of 6-mercaptopurine when combined with allopurinol b) how would the 6-mercaptopurinol treatment regimen need to be modified if the leukemia patient had a deficiency in xanthine oxidase? what other nucleotide metabolic disease would this be similar to, with regard to modifying a standard disease treatment on the basis of a patient's genetic profile?

(a) Xanthine oxidase inactivates 6-mercaptopurine by converting it to 6-thiouric acid, which lowers its effective concentration as a competitive inhibitor of adenylosuccinate synthetase and IMP dehydrogenase in leukemic cells. Therefore, by including allopurinol in the treatment, inactivation of 6-mercaptopurine by xanthine oxidase is repressed, leading to a higher effective concentration. (b) A leukemia patient with xanthine oxidase deficiency should be treated with a reduced dose of 6-mercaptopurine to avoid possible toxic side effects. Addition of allopurinol to this treatment would likely have no effect and should not be done. This is a similar problem to treating a cancer patient who has a deficiency in dihydropyrimidine dehydrogenase with 5-fluorouracil, as illustrated in Figure 18.26

Gene variants that are good candidates for Thrifty Genes:

- Acetyl-CoA carboxylase with a greater affinity for citrate -hexokinase with decreased affinity for fructose -glucokinase with decreased affinity for glucose -acetyl-CoA-carboxylase with a low affinity for palmitoyl-CoA -decreased catalytic activity of fatty-acyl-CoA-synthetase

Undiagnosed phenylketonuria (PKU) can lead to severe neuronal damage and mental incapacity, which is avoidable if the disease is diagnosed at birth. In contrast, albinism is easly diagnosed at birth, but there is no treatment, which leads to an increased risk of skin cancer in these patients. Choose the three statements that BEST explain the reasons for these differences.

- PKU is the result of substrate accumulation, which can be avoided using a restricted diet. However, albinism is due to a missing product, which cannot be replaced through injections or dietary regimens. -Defects in the enzyme tyrosinase occur in every cell of the body and there is no way to replace this enzyme without embryonic human gene therapy. In contrast, defects in the enzyme phenylalanine hydroxylase can be ameliorated by restricting certain amino acids in the diet. -Phenylalanine is found in many types of food, including aspartame, and therefore by avoiding foods with high amounts of phenylalanine, the build-up pyruvate metabolites can be controlled. In contrast, skin pigments must be made from tyrosine and there is no way to do this if tyrosine metabolism is compromised.

Which cellular change(s) would not encourage the Krebs bicycle and thus the probability of the urea cycle taking place to completion? Choose one or more: A. increased aspartate charging to mitochondrial tRNAs B. increased import of ornithine into the mitochondria C. increased citrulline being transported into the cytoplasm D. increased conversion of oxaloacetate into citrate

- increased aspartate charging to mitochondrial tRNAs - increased conversion of oxaloacetate into citrate

Gout can be very painful and is the result of uric acid crystals accumulating in the big toe. Which THREE statements below BEST describe biochemical causes of gout.

-Defects in the enzyme hypoxanthine-guanine phosphoribosyltransferase lead to accumulation of hypoxanthine -Loss of allosteric control in an enzyme that generates metabolic precursors of uric acid. -Increased flux through the purine degradation pathway owing to elevated expression of the rate-limiting enzyme in the pathway.

Undiagnosed phenylketonuria (PKU) can lead to severe neuronal damage and mental incapacity, which is avoidable if the disease is diagnosed at birth. In contrast, albinism is easly diagnosed at birth, but there is no treatment, which leads to an increased risk of skin cancer in these patients. Choose the THREE statements that BEST explain the reasons for these differences.

-Defects in the enzyme tyrosinase occur in every cell of the body and there is no way to replace this enzyme without embryonic human gene therapy. In contrast, defects in the enzyme phenylalanine hydroxylase can be ameliorated by restricting certain amino acids in the diet. -Phenylalanine is found in many types of food, including aspartame, and therefore by avoiding foods with high amounts of phenylalanine, the build-up pyruvate metabolites can be controlled. In contrast, skin pigments must be made from tyrosine and there is no way to do this if tyrosine metabolism is compromised. -PKU is the result of substrate accumulation, which can be avoided using a restricted diet. However, albinism is due to a missing product, which cannot be replaced through injections or dietary regimens.

Undiagnosed phenylketonuria (PKU) can lead to severe neuronal damage and mental incapacity, which is avoidable if the disease is diagnosed at birth. In contrast, albinism is easly diagnosed at birth, but there is no treatment, which leads to an increased risk of skin cancer in these patients. Choose the three statements that BEST explain the reasons for these differences.

-Defects in the enzyme tyrosinase occur in every cell of the body and there is no way to replace this enzyme without embryonic human gene therapy. In contrast, defects in the enzyme phenylalanine hydroxylase can be ameliorated by restricting certain amino acids in the diet. -Phenylalanine is found in many types of food, including aspartame, and therefore by avoiding foods with high amounts of phenylalanine, the build-up pyruvate metabolites can be controlled. In contrast, skin pigments must be made from tyrosine and there is no way to do this if tyrosine metabolism is compromised. -PKU is the result of substrate accumulation, which can be avoided using a restricted diet. However, albinism is due to a missing product, which cannot be replaced through injections or dietary regimens.

Based on what is known about neuronal control of energy metabolism in humans, why is it so hard to lose weight by dieting? More than one answer may be correct.

-Food can be addictive to some people, much like narcotics. Therefore, the reward they get from binge eating leads to food cravings during diet-induced "withdrawal." -Insulin levels decrease when blood sugar decrease, which is one of the objectives of dieting, therefore it decreases the signal to eat less; metabolize more. The net result is that you eat more; metabolize less because your body thinks you are starving for sugar. -PYY3-36 signals to the brain that you have food in your stomach and it inhibits the signal to the brain to eat more; metabolize less. Dieters are often so hungry that they eat too fast and the PYY3-36 inhibitory signal fails. -Ghrelin levels increase when your stomach is empty, which is one of the objectives of dieting, yet ghrelin signals to the brain to eat more; metabolize less. The net result is you always feel hungry and eventually give in. -Leptin levels decrease when fat deposits decrease, which in turn, decreases the signal to eat less; metabolize more. The net result is that you eat more; metabolize less because your body thinks you are starving.

How does depletion of liver glycogen stores during starvation lead to a shift from glucose utilization to fatty acid oxidation as an energy source for most tissues?

-Lack of insulin signaling reduces GLUT4 translocation. -Glucagon activates hormone-sensitive lipase. -Lack of liver glycogen stores results in a drop in serum glucose concentration

Based on what is known about neuronal control of energy metabolism in humans, why is it so hard to lose weight by dieting? More than one answer may be correct.

-Leptin levels decrease when fat deposits decrease, which in turn, decreases the signal to eat less; metabolize more. The net result is that you eat more; metabolize less because your body thinks you are starving. -Food can be addictive to some people, much like narcotics. Therefore, the reward they get from binge eating leads to food cravings during diet-induced "withdrawal." -Ghrelin levels increase when your stomach is empty, which is one of the objectives of dieting, yet ghrelin signals to the brain to eat more; metabolize less. The net result is you always feel hungry and eventually give in. -Insulin levels decrease when blood sugar decrease, which is one of the objectives of dieting, therefore it decreases the signal to eat less; metabolize more. The net result is that you eat more; metabolize less because your body thinks you are starving for sugar. -PYY3-36 signals to the brain that you have food in your stomach and it inhibits the signal to the brain to eat more; metabolize less. Dieters are often so hungry that they eat too fast and the PYY3-36 inhibitory signal fails.

What are 3 mechanisms by which cancer cells become resistant to the drug methotrexate?

-Overexpression of drug transporter -DHFR mutation blocking drug binding -DHFR gene amplification

In the reaction mechanism of the E. coli ribonucleotide reductase, which state does the cysteine side chain never exist in?

-SOH

Glucose-6P is a central metabolite in human physiology and functions as a subtrate or product in at least six metabolic pathways. What biochemical mechanisms determine glucose-6P flux through these pathways. Choose the THREE best answers.

-Substrate availability -Allosteric control -Enzyme levels

Defects in nucleotide metabolism that lead to a buildup of uric acid cause the disease gout. Select the TWO true statements below regarding potential treatments for gout.

-Supplying humans with the enzyme urate oxidase theoretically could be effective in reducing uric acid levels. -An inhibitor of xanthine oxidase, such as allopurinol, will limit the conversion of hypoxanthine to uric acid, and could be effective in reducing uric acid levels.

What occurs during starvation?

-There's an increase of fatty acids released from adipose tissue. -Skeletal muscle proteins are -degraded. -There's an increase of gluconeogenesis in the liver and kidneys.

How does the cell regulate the substrate specificity of ribonucleotide reductase to maintain equal amounts of all four deoxynucleoside triphosphates (dNTPs)?

-When dGTP is bound to the specificity site, more dADP is produced. -When dTTP is bound to the specificity site, more dGDP is produced.

Which cellular change(s) would not encourage the Krebs bicycle and thus the probability of the urea cycle taking place to completion?

-increased conversion of oxaloacetate into citrate -increased aspartate charging to mitochondrial tRNAs

) ADA-SCID is caused by mutations in the enzyme adenosine deaminase leading to accumulation of deoxyadenosine (dA). Excess dA increases production of dATP and inhibition of DNA replication in developing immune cells, causing the SCID immunodeficiency phenotype. Which combination of allosteric effectors bound to the specificity site and the activity site of ribonucleotide reductase provides the BEST biochemical explanation for the ADA-SCID phenotype?

. ATP bound to the specificity site; dATP bound to the activity site of ribonucleotide reductase

Which sequence is MOST likely to contain an O-linked oligosaccharide in the ER? (The ... represents additional amino acids N-terminal or C-terminal to those shown.)

...Ala-Thr-Leu-Phe-Ser-Gly...

How many more passes through b-oxidation does the saturated fat stearic acid (18:0) require, compared with the monounsaturated fat oleic acid (18:1)? 0 1 2 17

0

How many more passes through beta-oxidation does the saturated fat stearic acid (18:0) require, compared with the monounsaturated fat oleic acid (18:1)?

0

Starting from the oxidative phase of the pentose phosphate pathway, how many ATP and high-energy reduced molecules, respectively, are made in going from one glucose molecule to glyceraldehyde-3-phosphate and fructose-6-phosphate?

0, 2

How many of the nitrogens in orotidine-5-monophosphate arise from glutamine?

1

Which is the step in glucagon signaling where adipose triglyceride lipase mediates a reaction? 1. A hydrolysis reaction cleaves a fatty acid from the stored triacylglycerols to generate diacylglycerol and a fatty acid. 2. Perilipin is phosphorylated on the surface of lipid droplets. 3. Albumin transports the free fatty acid. 4. A GDP-GTP exchange stimulates cyclic AMP production by the enzyme adenylate cyclase.

1

Which is the step in glucagon signaling where adipose triglyceride lipase mediates a reaction? 1. A hydrolysis reaction cleaves a fatty acid from the stored triacylglycerols to generate diacylglycerol and a fatty acid. 2. Perilipin is phosphorylated on the surface of lipid droplets. 3. Albumin transports the free fatty acid. 4. A GDP-GTP exchange stimulates cyclic AMP production by the enzyme adenylate cyclase.

1

During the import of one protein from the cytosol to the nucleus via Ran-dependent nuclear import, __________ high-energy phosphate molecule(s) are hydrolyzed.

1 GTP

During the import of one protein from the cytosol to the nucleus via Ran-dependent nuclear import, __________ high-energy phosphate molecule(s) are hydrolyzed. 2 ATP 1 GTP 1 GTP and 1 ATP 2 GTP

1 GTP

Put the following steps in the correct order for the ribonucleotide reductase reaction mechanism. Step 1 of the six step reaction mechanism is shown at the right to help you get started:

1) A free radical at Y122 abstracts a hydrogen atom from C439 to form a cysteine thiyl radical 2) A thiyl free radical removes a hydrogen atom from the C-3' position of the ribose ring to generate a radical 3) The hydroxyl group on C-2' of the ribose ring gains a proton from the C225 residue of the enzyme 4) A water molecule is removed in the reaction leaving behind a cation on C-2' that is stabilized by the radical on C-3' 5) Cation on C-2' is reduced by C462 creating the deoxynucleotide and the radical at C-3' is reduced by C43 6) Deoxynucleoside diphosphate is released from the enzyme andC225-C462 disulfide bond is reduced by thioredoxin

Put the following steps in the correct order for the ribonucleotide reductase reaction mechanism. Step 1 of the six step reaction is shown above to help you get started.

1) A free radical at Y122 abstracts a hydrogen atom from C439 to form a cysteine thiyl radical. 2) A thiyl free radical removes a hydrogen atom from the C-3' position of the ribose ring to generate a radical. 3) The hydroxyl group on C-2' of the ribose ring gains a proton from the C225 residue of the enzyme. 4) A water molecule is removed in the reaction leaving behind a cation on the C-2' that is stabilized by the radical on C-3'. 5) Cation on the C-2' is reduced by C462 creating the deoxynucleotide and the radical at C-3' is reduced by thioredoxin. 6) Deoxynucleoside diphosphate is released from the enzyme and C225-C462 disulfide bond is reduced by thioredoxin.

Insulin signaling activates glucose uptake in skeletal and adipose tissue. Place the steps of this process in the order that they would occur in both tissues.

1) Serum glucose concentration >4.5mM 2) Activity of glucokinase in pancreatic beta cells increases 3) Release of insulin from pancreatic intracellular vesicles 4) Insulin binds to receptor on the cell surface 5) Translocation of GLUT4 to cell surface 6) Increased glucose uptake by skeletal muscle and adipose tissue

what are the three mechanisms by which cancer cells can become resistant to methotrexate treatment

1) mutation in the target enzyme DHFR so it does not bind the drug but is still active 2) amplification of DHFR to overproduce the enzyme 3) overexpress multidrug resistance protein to efflux the drug

Describe the 3 mechanisms by which cancer cells can become resistant to methotrexate treatment; which of these mechanisms might be overcome by developing a methotrexate derivative using structure based drug design?

1) mutation in the target enzyme DHFR so it does not bind the drug but is still inactive 2) amplification of the DHFR gene to overproduce the enzyme 3) overexpress the multidrug resistant protein to efflux the drug. Mechanism 1 is the most likely to be overcome with a new drug design whereas mechanisms 2 and 3 would be unaffected by the new drug.

In purines, __________ nitrogen atoms and __________ carbon atoms originate from glycine.

1, 2

A volunteer with Flying Samaritans brings a small boy to the local physician for treatment. Based on the background information listed below, what is the most likely biochemical basis for the boy's symptoms? =========== • The boy's body is hunched over and he is barely able to walk on his own • The physician noticed his wrists and ankles were thickened and enlarged The patient's abdomen is distended and his breastbone is protruding The volunteer measures his height and it is at the 30th percentile for age The villagers live in a large cave during the day to avoid the hot sun

1, 25-hydroxyvitamin D3 deficiency

Which FIVE of the following gene variants are good candidates to function as Thrifty Genes? 1) Acetyl-CoA carboxylase with high affinity for citrate 2) Glucokinase with decreased affinity for glucose 3) Decreased catalytic activity of fatty acyl-CoA synthetase 4) Hexokinase with decreased affinity for fructose 5) Acetyl-CoA carboxylase with low affinity for palmitoyl-CoA 6) Gherlin mutant with a low affinity for the ghrelin receptor 7) Decreased catalytic activity of fatty acid synthase 8) High basal level expression of Uncoupling Protein 1 9) Low activity lipoprotein lipase on the membrane of adipocytes

1, 4, 2, 5, 3

Which FIVE of the following gene variants are good candidates to function as Thrifty Genes? 1) Acetyl-CoA carboxylase with high affinity for citrate 2) Glucokinase with decreased affinity for glucose 3) Decreased catalytic activity of fatty acyl-CoA synthetase 4) Hexokinase with decreased affinity for fructose 5) Acetyl-CoA carboxylase with low affinity for palmitoyl-CoA 6) Gherlin mutant with a low affinity for the ghrelin receptor 7) Decreased catalytic activity of fatty acid synthase 8) High basal level expression of Uncoupling Protein 1 9) Low activity lipoprotein lipase on the membrane of adipocytes

1, 4, 2, 5, 3

Symptoms: 1. Boy's body is hunched over and he is barely able to walk on his own 2. wrists and ankles were thickened and enlarged 3. abdomen is distended and the breastbone is protruding 4. His height is at the 30th percentile for his age 5. Lives in a large cave during the day to avoid the hot sun

1,25-hydroxyvitamin D1 deficiency

Volunteer with Flying Samaritans brings a small boy to the physician The boy's body is hunched over and he is barely able to walk on his own The physician noticed his wrists and ankles were thickened and enlarged The patient's abdomen is distended and his breastbone is protruding The volunteer measures his height and it is at the 30th percentile for age The villagers live in a large cave during the day to avoid the hot sun

1,25-hydroxyvitamin D3 deficiency

Put the following steps in the correct order for the ribonucleotide reductase reaction mechanism. Step 1 of the six step reaction mechanism is shown above to help you get started.

1. A free radical at Y122 abstracts a hydrogen atom from C439 to form a cysteine thiyl radical 2. A thiyl free radical removes a hydrogen atom from the C-3' position of the ribose ring to generate a radical 3. The hydroxyl group on C-2' of the ribose ring gains a proton from the C225 residue of the enzyme 4. A water molecule is removed in the reaction leaving behind a cation on C-2' that is stabilized by the radical on C-3' 5. Cation on C-2' is reduced by C462 creating the deoxynucleotide and the radical at C-3' is reduced by C439 6. Deoxynucleoside diphosphate is released from the enzyme and C225-C462 disulfide bond is reduced by thioredoxin

Put the following steps in the correct order for the ribonucleotide reductase reaction mechanism

1. A free radical at Y122 abstracts a hydrogen atom from C439 to form a cysteine thiyl radical 2. A thiyl free radical removes a hydrogen atom from the C-3' position of the ribose ring to generate a radical 3. The hydroxyl group on C-2' of the ribose ring gains a proton from the C225 residue of the enzyme 4. A water molecule is removed in the reaction leaving behind a cation on C-2' that is stabilized by the radical on C-3' 5. Cation on C-2' is reduced by C462 creating the deoxynucleotide and the radical at C-3' is reduced by C439 6. Deoxynucleoside diphosphate is released from the enzyme and C225-C462 disulfide bond is reduced by thioredoxin

Put the following steps in the correct order for the ribonucleotide reductase reaction.

1. A free radical at Y122 abstracts a hydrogen atom from C439 to form a cysteine thiyl radical 2. A thiyl free radical removes a hydrogen atom from the C-3' position of the ribose ring to generate a radical at that position 3. The hydroxyl group on C-2' of the ribose ring gains a proton from the C225 residue of the enzyme 4. A water molecule is removed in the reaction leaving behind a cation on C-2' that is stabilized by the radical on C-3' 5. Cation on C-2' is reduced by C462 creating the deoxynucleotide and the radical at C-3' is reduced by C439 6. Deoxynucleoside diphosphate is released from the active site and the C225-C462 disulfide bond is reduced by thioredoxin

Put the following steps in the correct order for the ribonucleotide reductase reaction mechanism. Step 1 of the six step reaction mechanism is shown above to help you get started.

1. A free radical at Y122 abstracts a hydrogen atom from C439 to form a cysteine thiyl radical 2.A thiyl free radical removes a hydrogen atom from the C-3' position of the ribose ring to generate a radical 3.The hydroxyl group on C-2' of the ribose ring gains a proton from the C225 residue of the enzyme 4. A water molecule is removed in the reaction leaving behind a cation on C-2' that is stabilized by the radical on C-3' 5. Cation on C-2' is reduced by C462 creating the deoxynucleotide and the radical at C-3' is reduced by C439 6. Deoxynucleoside diphosphate is released from the enzyme and C225-C462 disulfide bond is reduced by thioredoxin

What is the name of the disease caused by an adenosine deaminase deficiency? What are the clinical symptoms?

1. ADA-SCID 2. Characterized by a defective immunize system, resulting from extremely low levels of infection-fighting B and T cells

Two Purines

1. Adenine 2. Guanine

Defects in nucleotide metabolism that lead to a buildup of uric acid cause the disease gout. Select the TWO true statements below regarding potential treatments for gout.

1. An inhibitor of xanthine oxidase, such as allopurinol, will limit the conversion of hypoxanthine to uric acid, and could be effective in reducing uric acid levels. 2. Supplying humans with the enzyme urate oxidase theoretically could be effective in reducing uric acid levels.

What TWO reasons best explain why fats are the primary energy storage molecule compared to carbohydrates?

1. Carbohydrates have a higher oxidized state than fats, so the energy yield is lower per gram of material. 2. Carbohydrates are hydrated, which adds more wet weight so would be a burden to carry around carbs.

Three Pyrimidines

1. Cytosine 2. Thymine 3. Uracil

Put the correct steps required for absorption of dietary fats into order.

1. Emulsification of fats 2. Lipase cleavage 3. Intracellular synthesis of triacylglycerols 4. Chylomicron assembly 5. Export of lipoprotein particle 6. Entry into the circulatory system

Nucleotides have fourkey roles in biochemical processes:

1. Energy conversion 2. Signal transduction 3. Coenzyme-dependent reactions 4. Genetic Information storage

What is the biochemical role of fulumarate in the so-called Krebs Bicycle? Choose the two BEST answers

1. Fumarate provides the carbon skeleton needed to carry nitrogen from glutamate deamination in the mitochondrial matrix to the urea cycle via aspartate. 2.The nitrogen atom in asparate is carried by arginine in the urea cycle, however without fumarate, the carbon skeleton of aspartate cannot be regenerated by oxaloacetate in the citrate cycle and the urea cycle would stop.

The use of any weight-loss drug must be combined with a reduced-calorie diet to produce effective weight loss. The nutritional composition of the diet can affect weight loss, even in combination with a weight loss drug. Assuming that all diets listed below are of equal caloric value, which would not be very effective in promoting weight loss when combined with orlistat?

1. High carbohydrate, low fat 2. High carbohydrate, high fat

Patients with type 1 diabetes must inject insulin up to several times a day and monitor their carbohydrate intake to regulate their blood glucose level. The amount of insulin injected is based on the carbohydrate content of the diet and so must be adjusted accordingly; otherwise, hypoglycemia will result. Which of the following explain(s) why injecting too much insulin before a meal results in hypoglycemia?

1. Increased glycogen storage 2.Decreased glucose synthesis 3.Increased glucose uptake

Type 1 and Type 2 diabetes are diagnosed with a clinical test following a 12 hour fast. Identify each of the parameters in the graph below by choosing the correct labels for letters A-F.

1. Inject insulin 2. Type 2 diabetes 3. Serum glucose 4. Time (hours) 5. Type 1 diabetes 6. Normal

How does depletion of liver glycogen stores during starvation lead to a shift from glucose utilization to fatty acid oxidation as an energy source for most tissues?

1. Lack of insulin signaling reduces GLUT4 translocation. 2. Lack of liver glycogen stores results in a drop in serum glucose concentration. 3. Glucagon activates hormone-sensitive lipase.

Glycogen can provide fuel for both a short race (e.g., 200 m) and a long race (i.e., 10 K or more). Which of the following best describe(s) the use of muscle and liver glycogen during short and long races?

1. Muscle glycogen can be used to fuel long runs. 2. Liver glycogen can be used to fuel long runs. 3. Muscle glycogen can be used to fuel short runs.

What three mechanisms by which cancer cells become resistant to the drug methotrexate?

1. Overexpression of drug ttransporter 2. DHFR mutation blocking drug binding 3. DHFR gene amplification

G-6-P is a key metabolic intermediate in four major pathways. Describe conditions in which flux through each of these four pathways would be increased as a result of changes in metabolite concentrations or enzyme activities.

1. PPP: flux will be increased when NADPH levels are low 2. Glycolysis: flux will be increased when energy charge in the cell is low (high AMP/ADP) 3. Glycogen Synthesis: flux will be increased when energy charge in the cell is high (high ATP) 4. Glucose export from liver: flux will increase when blood glucose is low in response to glucagon or epinephrine

Undiagnosed phenylketonuria (PKU) can lead to severe neuronal damage and mental incapacity, which is avoidable if the disease is diagnosed at birth. In contrast, albinism is easly diagnosed at birth, but there is no treatment, which leads to an increased risk of skin cancer in these patients. Choose the three statements that BEST explain the reasons for these differences.

1. Phenylalanine is found in many types of food, including aspartame, and therefore by avoiding foods with high amounts of phenylalanine, the build-up pyruvate metabolites can be controlled. In contrast, skin pigments must be made from tyrosine and there is no way to do this if tyrosine metabolism is compromised. 2. PKU is the result of substrate accumulation, which can be avoided using a restricted diet. However, albinism is due to a missing product, which cannot be replaced through injections or dietary regimens. 3. Defects in the enzyme tyrosinase occur in every cell of the body and there is no way to replace this enzyme without embryonic human gene therapy. In contrast, defects in the enzyme phenylalanine hydroxylase can be ameliorated by restricting certain amino acids in the diet.

Four major changes during starvation

1. Release of fatty acids from adipose 2. Increased gluconeogenesis in liver and kidneys 3. Increased ketogenesis in liver 4. Protein degradation in skeletal muscle

Defects in nucleotide metabolism that lead to a buildup of uric acid cause the disease gout. Select the TWO true statements below regarding potential treatments for gout.

1. Supplying humans with the enzyme urate oxidase theoretically could be effective in reducing uric acid levels. 2. An inhibitor of xanthine oxidase, such as allopurinol, will limit the conversion of hypoxanthine to uric acid, and could be effective in reducing uric acid levels.

How does the cell regulate the substrate specificity of ribonucleotide reductase to maintain equal amounts of all four deoxynucleoside triphosphates (dNTPs)?

1. When dTTP s bound to the specificity site, more dGDP is produced. 2. When dGTP is bound to the specificity site, more dADP is produced

Endothelial cells can receive a chemical signal from neurons. This signal is in the form of neuronal (1) release. This chemical messenger binds to a membrane receptor that engages a signal transduction pathway, which increases the cytoplasmic concentration of (2) ions. This ionic secondary messenger then activates endothelial nitric oxide synthase.

1. acetylcholine 2. calcium

Binding of (1) to the active site of ribonucleotide reductase inhibits all activity.

1. dATP

Which of the nucleotides is/are synthesized from a pathway that involves inosine-5'-monophosphate?

1. guanosine Monophosphate 2. adenosine monophosphate

Which cellular change(s) would not encourage the Krebs bicycle and thus the probability of the urea cycle taking place to completion?

1. increased conversion of oxaloacetate into citrate 2. increased aspartate charging to mitochondrial tRNAs

The bioactive 5-fluorouracil (1) a six-atom heterocyclic ring.

1. is

With the Urea cycle, citrulline is produced by orntithine transcarbamylase in the (1) and is consumed in the (2).

1. mitochondria 2. cytoplasm

Describe the three mechanisms by which cancer cells can become resistant to methotrexate treatment; which of these mechanisms might be overcome by developing a methotrexate derivative using structure based drug design?

1. mutation in the target enzyme DHFR so it does not bind the drug but is still active 2. amplification of the DHFR gene to overproduce the enzyme 3. overexpress the multidrug resistance protein to efflux the drug Mechanism 1 (DHFR mutation) is the most likely to be overcome with a new drug design, whereas mechanism 2 and 3 would be unaffected by a new drug.

During IMP biosynthesis, fumarate is released as a byproduct. Fumarate is cleaved from a(n) (1) atom on the developing heterocyclic ring of the purine.

1. nitrogen

Phenylketonuria is caused by a defect in the enzyme (1). The symptoms of PKU are caused by (2) and can be treated by diet which limits (3).

1. phenylalanine hydroxylase 2. substrate accumulation 3. phenylalanine

What cellular condition(s) would cause the upregulation of AMPK activity?

1. phosphorylation of Thr172 in the α subunit 2. low energy charge in muscle cells

Albinism is characterized by white skin and hair due to a defect in the enzyme (1) and is characterized by a substrate (2) mechanism.

1. tyrosinase 2. lack of product

In the proposed mechanism for the E. coli ribonucleotide reductase reaction, a (1) radical in the R2 subunit is reduced by Cys439 in the R1 subunit.

1. tyrosine

Which of the following membranes would have the greatest fluidity based on the following percentage of cholesterol in the membrane?

10%

A man who does not have Lesch-Nyhan syndrome and a woman whose mother and father both had Lesch-Nyhan syndrome have a male child together. What are the chances that this child will have Lesch-Nyhan syndrome? Assume a traditional Mendelian X-linked recessive inheritance pattern for the purpose of this question and assume that all organisms are fertile.

100%

A man who does not have Lesch-Nyhan syndrome and a woman whose mother and father both had Lesch-Nyhan syndrome have a male child together. What are the chances that this child will have Lesch-Nyhan syndrome? Assume a traditional Mendelian X-linked recessive inheritance pattern for the purpose of this question and assume that all organisms are fertile. (Note: Human genetics is very complex, and although genotype follows traditional Mendelian patterns, the organismal phenotype can be quite diverse. Focus on genotypic inheritance to answer this question.)

100%

Consider the following scenario: A man who does not have Lesch-Nyhan syndrome and a woman whose mother and father both had Lesch-Nyhan syndrome have a child together. What are the chances that this child will have Lesch-Nyhan syndrome? Assume a traditional Mendelian X-linked recessive inheritance pattern for the purpose of this question and assume that all organisms are fertile.

100%

Two homozygous parents express the GTA glycosyltransferase. What is the probability of a child with an A blood type?

100%

Two homozygous parents express the GTA glycosyltransferase. What is the probability of a child with an A blood type? 0% 25% 75% 100%

100%

How many ATPs are obtained from the complete beta-oxidation of one mole of stearoyl-CoA (18Cs) compared with three molecules of glucose (3 x 6C = 18Cs)?

120 versus 90

How many ATPs are obtained from the complete mc084-1.jpg-oxidation of one mole of stearoyl-CoA (18Cs) compared with three molecules of glucose (3 mc084-2.jpg 6C = 18Cs)? 106 versus 32 100 versus 100 36 versus 90 120 versus 90

120 versus 90

The Shine-Dalgarno sequence of mRNA base pairs with the __________ rRNA within the prokaryote __________ ribosomal subunit

16S; 30S

The Shine-Dalgarno sequence of mRNA base pairs with the __________ rRNA within the prokaryote __________ ribosomal subunit. 23S; 30S 16S; 50S 16S; 30S 5S; 50S

16S; 30S

Assuming 65% efficiency of converting glucose to ATP by oxidative phosphorylation, how many moles of ATP can be synthesized by consuming a large bag of potato chips containing 56 g of carbohydrate in the form of starch?

18.5 mol of ATP 3.7 kcal/per g of glusoe 4.18 kj/kcal 1 mol ATP/30.5 kJ

How many GTP are hydrolyzed during a single round of translation elongation in a eukaryote?

2

The first two reactions of gluconeogenesis are required to reverse reaction 10 (or the last reaction) of glycolysis. How many ATP equivalents are used by these first two reactions of gluconeogenesis?

2

The first two reactions of gluconeogenesis are required to reverse reaction 10 (or the last reaction) of glycolysis. How many ATP equivalents are used by these first two reactions of gluconeogenesis? 1 2 3 4

2

Phenylketonuria (1) /Albinism (2) is characterized by white skin due to a defect in the enzymetyrosinase, which leads to a block in the pathwayneeded to synthesize adrenaline(3) / melanin (4). Animals with this condition have an increased risk of skin cancer(5)/ mental illness(6). The genetic disease albinism(7) / phenylketonuria(8) is caused by a defect in the enzyme phenylalanine hydroxylase, which leads to the accumulation of tyrosine(9) / phenylalanine(10) derivatives that are toxic metabolites

2 4 5 8 10

What is the overall balanced reaction for the light reactions of photosynthesis?

2 H2O + 8 photons + 2 NADP+ + 3 ADP + 3 Pi --> O2 + 2 NADPH + 3 ATP

What is the overall balanced reaction for the light reactions of photosynthesis? O2 + 2 NADPH + 3 ATP mc004-1.jpg 2 H2O + 2 NADP+ + 3 ADP + 3 Pi 3 CO2 + 6 NADPH + 9 ATP + 6 H2O mc004-2.jpg Glyceraldehyde-3-phosphate + 6 NADP+ + 9 ADP + 9 Pi O2 + 8 photons + NADP+ + 3 ATP mc004-3.jpg 2 H2O + NADP+ + 3 ADP + 3 Pi 2 H2O + 8 photons + 2 NADP+ + 3 ADP + 3 Pi mc004-4.jpg O2 + 2 NADPH + 3 ATP

2 H2O + 8 photons + 2 NADP+ + 3 ADP + 3 Pi mc004-4.jpg O2 + 2 NADPH + 3 ATP

Describe the experimental evidence for a eukaryotic "purinosome" and what the results in figure 18.10 panel F reveal about the purine biosynthetic pathway.

2 human enzymes TrifGART and FGAMS are required for steps 2-5 of the E. Coli purine biosynthetic pathway. Using purine depleted media to induce high levels of purine biosynthesis, it was observed that both of these enzymes co-localise to the same subcellular localization in humans cells grown in culture. These punctate foci were observed using fluorescence microscopy with different fluros for each enzyme (GFP and OFP) in a merged overly( panel F in 19.10) providing strong support for purinosomes.

Noncanonical base pairings of A and I would contain __________ hydrogen bonds. The base pairing of C and I would contain __________ hydrogen bonds.

2, 2

Phenylketonuria (1) / Albinism (2) is characterized by white skin due to a defect in the enzyme tyrosinase, which leads to a block in the pathway needed to synthesize adrenaline (3) / melanin (4). Animals with this condition have an increased risk of skin cancer (5) / mental illness (6). The genetic disease albinism (7) / phenylketonuria (8) is caused by a defect in the enzyme phenylalanine hydroxylase, which leads to the accumulation of tyrosine (9) / phenylalanine (10) derivatives that are toxic metabolites

2, 4, 5, 8, 10

Phenylketonuria (1)/Albinism (2) is characterized by white skin due to a defect in the enzyme tyrosinase, which leads to a block in the pathway needed to synthesize adrenaline (3)/melanin (4). Animals with this condition have an increased risk of skin cancer (5)/mental illness (6). The genetic disease albinism (7)/phenylketonuria (8) is caused by a defect in the enzyme phenylalanine hydroxylase, which leads to the accumulation of tyrosine (9)/phenylalanine (10) derivatives that are toxic metabolites.

2, 4, 5, 8, 10

Albinism is characterized by white skin due to a defect in the enzyme arginase (1) tyrosinase (2), which leads to a block in the pathway needed to synthesize adrenaline (3) / melanin (4). Animals with this condition have a(n) decreased (5) / increased (6) risk of skin cancer. The genetic disease phenylketonuria is caused by a defect in the enzyme tyrosine (7) / phenylalanine (8) hydroxylase, which leads to mental deficiencies caused by a decrease (9) / increase (10) of phenylalanine derivatives that are toxic metabolites

2, 4, 6, 8, 10

) What three biochemical mechanisms determine glucose-6P flux through metabolic pathways.

2. Bioavailability of glucose-6P (concentration) when used as a substrate 4. Activity level of enzymes (allosteric control) for which glucose-6P is a substrate 6. Bioavailability of enzymes (cell localization) for which glucose-6P is a substrate

What three biochemical mechanisms determine glucose-6P flux through metabolic pathways

2. Bioavailability of glucose-6P (concentration) when used as a substrate 4. Activity level of enzymes (allosteric control) for which glucose-6P is a substrate 6. Bioavailability of enzymes (cell localization) for which glucose-6P is a substrate

Aspartate contains four carbons. If all are radioactively labeled, how many carbons of UMP will be radioactively labeled?

3

During intense exercise, muscle ATP stores will be depleted in about

3 seconds

Which four adaptations does the human body make in response to the lack of nutrients during starvation?

3, 4, 7, 8 1. The release of fatty acids from adipose tissue provides ATP for the brain. 2. The liver solely produces glucose to make ATP for the brain. 3. Amino acids from protein degradation of skeletal muscles can be used to produce glucose. 4. Fatty acid release from adipose tissue increases after the first few days to provide ATP for the skeletal muscle and the heart. 5. Ketone body production increases to provide ATP for the heart and skeletal muscle. 6. Protein degradation in the skeletal muscle occurs first during starvation. 7. The kidneys and liver participate in increased gluconeogenesis. 8. Ketone bodies increase via the liver and provide ATP for the heart and brain.

The prokaryotic ribosome contains __________ subunits. The eukaryotic ribosome contains __________ subunits.

30S and 50S; 40S and 60S

The prokaryotic ribosome contains __________ subunits. The eukaryotic ribosome contains __________ subunits. 30S and 70S; 40S and 60S 40S and 60S; 30S and 80S 30S and 50S; 40S and 60S 30S and 60S; 40S and 50S

30S and 50S; 40S and 60S

What percentage of the recommended 2,000 Calories/d is accounted for in a late-night snack consisting of a large bag of potato chips (56 g starch) and a 1-L soda containing 110 g of high-fructose corn syrup? Assume 4 Calories/g of carbohydrate (starch or high-fructose corn syrup).

33%

Thiazolidinedione activates PPAR-gamma in adipose tissue. This results in the upregulation of

35-45 mph

Place the following steps in their proper order starting with dUMP and N5,N10-methylenetetrahydrofolate as substrates. 1. Formation of tetrahydrofolate 2. Formation of N5,N10-methylenetetrahydrofolate 3. Formation of 7,8-dihydrofolate

3; 1; 2

Place the following steps in their proper order starting with dUMP and N5,N10-methylenetetrahydrofolate as substrates. 1. Formation of tetrahydrofolate 2. Formation of N5,N10-methylenetetrahydrofolate 3. Formation of 7,8-dihydrofolate 2; 1; 3 1; 3; 2 1; 2; 3 3; 1; 2

3; 1; 2

How many different forms of pyridoxal phosphate are observed in the aminotransferase reaction?

4

How many total NADH + FADH2s are made in the beta-oxidation pathway from the breakdown of the following fatty acid to all acetyl-CoAs?

4

How many total NADH + FADH2s are made in the mc082-1.jpg-oxidation pathway from the breakdown of the following fatty acid to all acetyl-CoAs? 6 8 4 2

4

Most fad diets do not work because

40% of people do not "stick to it"

The following figure shows the cellular processes that synthesize, secrete, and recognize glycoconjugates in eukaryotic cells. Identify the cell surface glycoconjugates.

5

The following figure shows the cellular processes that synthesize, secrete, and recognize glycoconjugates in eukaryotic cells. Identify the cell surface glycoconjugates. 1 2 5 6

5

Purine biosynthesis takes place in two stages. The first stage begins with ribose-5-phosphate and ends with

5-aminoimidazole ribonucleotide.

Purine biosynthesis takes place in two stages. The first stage begins with ribose-5-phosphate and ends with 5-phosphoriboxylamine. 5-aminoimidazole ribonucleotide. inosine-mc127-1.jpg-monophosphate. 5-aminoimidazole-4-carboxamide ribonucleotide.

5-aminoimidazole ribonucleotide.

_____ Deoxynucleoside diphosphate is released from the enzyme and C225-C462 disulfide bond is reduced by thioredoxin _____ A water molecule is removed in the reaction leaving behind a cation on C-2' that is stabilized by the radical on C-3' _____ Cation on C-2' is reduced by C462 creating the deoxynucleotide and the radical at C-3' is reduced by C439 _____ A thiyl free radical removes a hydrogen atom from the C-3' position of the ribose ring to generate a radical _____ The hydroxyl group on C-2' of the ribose ring gains a proton from the C225 residue of the enzyme _____ A free radical at Y122 abstracts a hydrogen atom from C439 to form a cysteine thiyl radical

6, 4, 5, 2, 3, 1

The formation of one net glyceraldehyde-3-phosphate during the net Calvin cycle reactions requires __________ NADPH and __________ ATP.

6, 9

How many of the 10 glycolysis reactions use the same enzyme in the gluconeogenesis pathway?

7

How many of the 10 glycolysis reactions use the same enzyme in the gluconeogenesis pathway? 10 7 5 3

7

How many passes through the fatty acid oxidation pathway are required to degrade palmitic acid?

7

Why are cancer patients given high doses of 5-fluorouracil?

80% of this compound is metabolized by normal dihydropyrimidine dehydrogenase activity.

Where would an amino acid be attached to the tRNA below?

A

tRNA interacts with ribosomes at one of three sites: the E site, the P site, and the __________ site.

A

tRNA interacts with ribosomes at one of three sites: the E site, the P site, and the __________ site. T M SD A

A

Which of the following sequences of enzymatic activity would produce one of the nucleotides that is used as a substrate by a polymerase?

A guanylate kinase reaction, then a ribonucleotide reductase reaction, and finally a nucleoside diphosphate kinase reaction.

How would the 6-mercaptopurine treatment regimen need to be modified if the leukemia patient had a deficiency in xanthine oxidase? What other nucleotide metabolic disease would this be similar to, with regard to modifying a standard disease treatment on the basis of a patient's genetic profile?

A leukemia patient with xanthine oxidase deficiency should be treated with a reduced dose of 6-mercaptopurine to avoid possible toxic side effects. Addition of allopurinol to this treatment would likely have no effect and should not be done. This is a similar problem to treating a cancer patient who has a deficiency in dihydropyrimidine dehydrogenase with 5-fluorouracil.

A) what is the biochemical basis for gout? B) What is the most common symptom of gout?

A) Gout is caused by a buildup of uric acid owing to at least three factors B) pain in the big toe

Type 1 and Type 2 diabetes are diagnosed with a clinical test following a 12 hour fast. Identify each of the parameters in the graph below by choosing the correct labels for letters A-F.

A) Inject insulin B) Type 2 diabetes C) Serum glucose D) Time (hours) E) Type 1 diabetes F) Normal

1. (6 pts) Based on the function of PYY3-36, why should you eat quickly if you are trying to gain weight?

A. Because PYY3-36 signals food is in small intestine, so need to fill up quickly before feeling too full.

11. (6 pts) Based on the function of PYY3-36, why should you eat quickly if you are trying to gain weight?

A. Because PYY3-36 signals food is in small intestine, so need to fill up quickly before feeling too full.

Why does benzoic acid therapy lower blood ammonia levels in patients with deficiencies in the urea cycle considering that benzoic acid reacts with glycine to form hippuric acid, which is rapidly excreted?

A. Loss of glycine by hippuric acid excretion, drives more NH4+ into glycine biosynthesis to replace it.

Uric acid is derived from both purine biosynthesis and by diet. What accounts for high levels of 15N in excreted urine after feeding a patient 15N-glycine to diagnose the cause of gout? Explain.

A. The cause of the gout is excess production of uric acid because glycine is a precursor to purines

(6 pts) Uric acid is derived from both purine biosynthesis and by diet. What accounts for high levels of 15N in excreted urine after feeding a patient 15N-glycine to diagnose the cause of gout? Explain.

A. The cause of the gout is excess production of uric acid because glycine is a precursor to purines.

Inhibition of which of the following processes would disrupt the regulation of metabolic homeostasis?

A.insulin release from the pancreas B.glucagon release from the pancreas C.somatostatin release from the pancreas D.PPAR nuclear receptor activation

Neuronal samples from wild-type mice were monitored in the absence and presence of leptin via Western blotting. The data are shown below. Choose the answer that correctly identifies the NPY/AGRP neuronal samples obtained in the absence and then the presence of leptin. B; C D; B A; C C; A

A; C

Neuronal samples from wild-type mice were monitored in the absence and presence of leptin via Western blotting. The data are shown below. Choose the answer that correctly identifies the NPY/AGRP neuronal samples obtained in the absence and then the presence of leptin.

A;C

A full catalytic cycle of an aminoacyl-tRNA synthetase generates __________ as a product.

AMP

A full catalytic cycle of an aminoacyl-tRNA synthetase generates __________ as a product. ATP ADP H2O AMP

AMP

Which of the following molecules would activate phosphofructokinase 1 (PFK1)?

AMP

Activation of glucagon receptors in liver and adipose tissue increases levels of cyclic __, which activates __ signaling, which phosphorylates the enzyme __, leading to increased flux through __ and decreased flux through __.

AMP PKA PFK-2/FBPase-2 Gluconeogeneis Glycolysis

Analysis of AMP and GMP synthesized in E. coli grown in medium containing 15N-labeled aspartate would show that

AMP contained twice as much 15N than GMP.

According to the thrifty gene hypothesis, which of the following gene varients could be candidate thrifty genes whose expression would increase the chances that an individual would develop insulin resistance?

AMPK with a lower affinity for AMP

To study the allosteric regulation of E. coli ribonucleotide reductase in a cell-free in vitro system, which of the following would be a good choice to add to the system?

ATP

Which of the following molecules would inhibit the last reaction of glycolysis?

ATP

Which of the following molecules would inhibit the last reaction of glycolysis? ATP glucose AMP NAD+

ATP

What precursors are required for the breakdown of palmitate?

ATP and CoA

In the five steps used by photosynthetic organisms to convert solar energy into chemical energy, which molecules store the solar energy before sugar synthesis?

ATP and NADPH

In the five steps used by photosynthetic organisms to convert solar energy into chemical energy, which molecules store the solar energy before sugar synthesis? chlorophylls NADH and FADH2 H2O and O2 ATP and NADPH

ATP and NADPH

The Calvin cycle requires which species from the light reactions?

ATP and NADPH

ADA-SCID is caused by mutations in the enzyme adenosine deaminase leading to accumulation of deoxyadenosine (dA). Excess dA increases production of dATP and inhibition of DNA replication in developing immune cells, causing the SCID immunodeficiency phenotype. Which combination of allosteric effectors bound to the specificity site and the activity site of ribonucleotide reductase provides the BEST biochemical explanation for the ADA-SCID phenotype?

ATP bound to the specificity site; dATP bound to the activity site of ribonucleotide reductase.

Which of the following is a symptom of metabolic syndrome?

Abdominal obesity (*Individuals with metabolic syndrome have three or more symptoms from the following list. Symptoms include abdominal obesity, insulin resistance, high blood pressure (hypertension), hyperlipidemia, and high risk for cardiovascular disease)

Nitrogen atoms removed from proteins during protein catabolism are least likely to be conserved in the structure of which of the following?

Acetyl-CoA

Which FIVE of the following gene variant are good candidates to function as Thrifty Genes?

Acetyl-CoA carboxylase with high affinity for citrate Glucokinase with decreased affinity for glucose Decreased catalytic activity of fatty acyl-CoA synthetase Hexokinase with decreased affinity for fructose Acetyl-CoA carboxylase with low affinity for palmitoyl-CoA

Which FIVE of the following gene variants are good candidates to function as Thrifty Genes?

Acetyl-CoA carboxylase with low affinity for palmitoyl-CoA Decreased catalytic activity of fatty acyl-CoA synthetase Glucokinase with increased affinity for glucose. Acetyl-CoA carboxylase with high affinity for citrate Insulin receptor with increased affinity for insulin.

An effect of elevated levels of TNF-alpha in serum on adipocytes is:

Activation of NFKB

What is the probable diagnosis for Case Study 2.2 "It is good to be king?"

Acute intermittent porphyria

Cultured cells can be prevented from initiating DNA replication by adding excess thymidine to the growth media. Propose a mechanism to account for this on the basis of the specificity and activity of ribonucleotide reductase under different physiologic conditions.

Addition of high amounts of thymidine to cultured cells leads to the accumulation of dTTP through the salvage pathway. dTTP binding to the specificity site of ribonucleotide reductase inhibits reduction of CDP to dCDP, thus inhibiting DNA replication due to insufficient substrate

Cultured cells can be prevented from initiating DNA replication by adding excess thymidine to the growth media. Propose a mechanism to account for this "thymidine block" on the basis of the specificity and activity of ribonucleotide reductase under different physiologic conditions.

Addition of high amounts of thymidine to cultured cells leads to the accumulation of dTTP through the salvage pathway. dTTP binding to the specificity site of ribonucleotide reductase inhibits reduction of CDP to dCDP, thus inhibiting DNA replication due to insufficient substrate.

Cells defective in thymidylate synthase are able to undergo cell division if methotrexate and thymidine are oth provided: however, cells with fully functional thymidylate synthase cannot divide under these same culture conditions. What is the biochemical basis for this observation?

Addition of methotrexate inhibits dihydrofolate reductase, blocking the production of new tetrahydrofolate, a critical metabolite for numerous pathways. Because thymidylate synthase is active, the remaining tetrahydrofolate pool is quickly depleted. Adding thymidine to the culture does not rescue the cells because it is the low levels of tetrahydrofolate that cause cell death, not lack of thymidine. In contrast, thymidylate synthase-deficient cells do not use up the tetrahydrofolate pool when methotrexate is added, so that other tetrahydrofoloate-dependent pathways can function for a time. Moreover, added thymidine rescues the thymdylate synthase deficiency, so these cells are able to survive longer than normal cells.

Which of the following is the first source of energy in muscle when running a 100 yard dash?

Adenosine triphosphate [ATP] in the muscle

Which of the following is directly affected by thiazolidinediones?

Adipose tissue

The brain requires a constant input of glucose. The metabolic activity of which organ(s) is/are necessary to maintain the concentration of glucose required by the brain when dietary glucose is low?

Adipose tissue, kidney, liver

The brain requires a constant input of glucose. The metabolic activity of which organ(s) is/are necessary to maintain the concentration of glucose required by the brain when dietary glucose is low?

Adipose tissue, liver, kidney

What is the probable diagnosis for Case Study 1.3 "Squidward Q. Tentacles?"

Alkaptonuria

What is the probable diagnosis for Case Study 3.1 "Squidward Q. Tentacles?"

Alkaptonuria

Ribonucleotide reductase is a key enzyme that mediates the synthesis of deoxyribonucleotides, the DNA precursors. When dATP binds to the activity site and ATP binds to the specificity site, which of the following would likely occur in the catalytic site?

All dNDP production is inhibited.

Which enzyme class best describes the activity of a urease?

Amidohydrolase

What is the biochemical explanation for why rapid weight loss is reported with protein-rich diets?

Amino acid degradation results in urea production to remove excess NH4+, so mostly water weight loss

What is the biochemical explanation for why rapid weight loss is reported with protein-rich diets?

Amino acid degradation results in urea production to remove excess NH4+, so mostly water weight loss.

What is the biochemical explanation for why rapid weight loss is reported with protein-rich diets?

Amino acid degradation results in urea production to remove excess NH4+, so mostly water weightloss

Upon deamination of an amino acid, which component becomes substrate for the urea cycle?

Ammonia

What three properties of enzymes determine glucose-6P flux through metabolic pathways?

Amount of enzyme controlling a particular pathway Activity of enzyme controlling a particular pathway. Location of enzyme controlling a particular pathway.

How does a defect in adenosine deaminase (ADA) lead to inhibition of DNA synthesis?

An ADA (adenosine deaminase) deficiency increases dATP levels, which inhibits ribonucleotide reductase and DNA synthesis

How does a defect in adenosine deaminase (ADA) lead to inhibition of DNA synthesis?

An ADA deficiency increases dATP levels, which inhibits ribonucleotide reductase and DNA synthesis

How does a defect in adenosine deaminase (ADA) lead to inhibition of DNA synthesis?

An ADA deficiency increases dATP levels, which inhibits ribonucleotide reductase and DNA synthesis.

Why is de novo biosynthesis of purines markedly elevated in patients with a deficiency in hypoxanthine-guanine phosphoribosyl transferase (HGPRT)?

An HGPRT deficiecy prevents salvage biosynthesis of purines: therefore, de novo synthesis is stimulated in order to maintain sufficient purine levels

Why is de novo biosynthesis of purines markedly elevated in patients with a deficiency in HGPRT?

An HGPRT deficiency prevents salvage biosynthesis of purines; therefore, de novo synthesis is stimulated in order to maintain sufficient purine levels.

Individuals with enzyme deficiencies in the urea cycle have elevated blood ammonia levels, which can be life threatening. One useful therapy is to feed them benzoic acid, which reacts with glycine to form hippuric acid which can then be excreted in urine. Why does benzoic acid therapy lower ammonia levels?

An alternative way to excrete nitrogen-rich products is glycine depletion by formation of hippuric acid. This process drives ammonia into amino acid synthesis reactions to replace the lost glycine, thereby lowering the level of free ammonia

Individuals with enzyme deficiencies in the urea cycle have elevated blood ammonia levels, which can be life threatening. One useful therapy is to feed these individuals benzoic acid, which reacts with glycine to form hippuric acid. This compound can be excreted in the urine. Why does benzoic acid therapy lower blood ammonia levels in these individuals?

An alternative way to excrete nitrogen-rich products is glycine depletion by formation of hippuric acid. This process drives ammonia into amino acid synthesis reactions to replace the lost glycine, thereby lowering the level of free ammonia

a newborn infant with highly elevated levels of blood ammonia was diagnosed to have a urea cycle enzyme defect in either carbamoyl phosphate synthetase or arginase. The clinical lab was able to measure the concentrations of alanine, glutamine, and arginine in the blood, but the enzyme assays were not available. How would your knowledge of abnormal levels of alanine, glutamine, or arginine in the blood be useful to distinguish between an enzyme defect between the two?

An enzyme defect in carbamoyl phosphate synthetase would block the urea cycle at the first step of citrulline production from ornithine and carbamoyl phosphate. This would cause higher than normal alanine and glutamine concentrations because they cannot be metabolized and lower than normal arginine concentration because the urea cycle is not functioning (no arginine produced by argininosuccinase). However, a defective arginase would cause arginine to accumulate, and alanine and glutamine concentrations would be closer to normal, assuming excess nitrogen is excreted as argininosuccinate and arginine

A newborn infant with highly elevated levels of blood ammonia was diagnosed to have a urea cycle enzyme defect in either carbamoyl phosphate synthetase or arginase. The clinical lab was able to measure the concentrations of alanine, glutamine, and arginine in the blood, but enzyme assays were not available. How would knowledge of abnormal levels of alanine, glutamine, or arginine in the blood be useful to distinguish between an enzyme defect in carbamoyl phosphate synthetase or arginase?

An enzyme defect in carbamoyl phosphate synthetase would block the urea cycle at the first step of citrulline production from ornithine and carbamoyl phosphate. This would cause higher than normal alanine and glutamine concentrations because they cannot be metabolized and lower than normal arginine concentration because the urea cycle is not functioning (no arginine produced by argininosuccinase). However, a defective arginase would cause arginine to accumulate, and alanine and glutamine concentrations would be closer to normal, assuming excess nitrogen is excreted as argininosuccinate and arginine.

How does the fat storage set point relate to changes in the fatty acid metabolism of an individual?

An increase in adipose tissue concentrations above the set point activates fatty acid oxidation, whereas a decrease in adipose tissue below the set point activates fatty acid synthesis pathways.

An increase in SAICAR lyase activity would most directly cause which of the following?

An increase in the production of malate by way of the citrate cycle.

Why wouldn't an individual show signs of Lesch-Nyhan syndrome?

An individual with a single copy of HGPRT is sufficient to remove excess metabolir intermediates and avoid symptoms.

Defects in nucleotide metabolism that lead to a buildup of uric acid cause the disease gout. Select the TWO true statements below regarding potential treatments for gout.

An inhibitor of xanthine oxidase, such as allopurinol, will limit the conversion of hypoxanthine to uric acid, and could be effective in reducing uric acid levels. Supplying humans with the enzyme urate oxidase theoretically could be effective in reducing uric acid levels.

Which if the following three statements are true regarding neuropeptides and appetite control?

Anorexigenic neurons send out signals that increase energy expenditure Orexigenic neurons increase fat stores. Ghrelin stimulates orexigenic signaling by binding to NPY/POMC neurons

Nitric oxide (NO) is a potent vasodilator in humans. Synthesis of NO occurs by a two-step enzymatic reaction that is catalyzed by nitric oxide synthase. Which amino acid(s) can serve as substrate for this enzyme?

Arginine

Choose the ONE best explanation for why ingesting large amounts of L-arginine is a viable treatment for the urea cycle disorder argininosuccinase deficiency.

Arginine is a source of Ornithine, which provides the urea cycle intermediates needed to transfer nitrogen from amino acid degradation pathways to argininosuccinate.

Why is ingesting large amounts of L-arginine a viable treatment for arginine a viable treatment for argininosuccinase deficiency?

Arginine is converted to ornithine, which provides urea cycle intermediates needed for nitrogen removal.

Why is ingesting large amounts of L-arginine a viable treatment for argininosuccinase deficiency

Arginine is converted to ornithine, which provides urea cycle intermediates needed for nitrogen removal.

Why is ingesting large amounts of L-arginine a viable treatment for argininosuccinase deficiency?

Arginine is converted to ornithine, which provides urea cycle intermediates needed for nitrogen removal.

Why is ingesting large amounts of L-arginine a viable treatment for argininosuccinase deficiency?

Arginine s converted to ornithine, which provides urea cycle intermediates needed for nitrogen removal.

Why do angina patients carry nitroglycerin with them?

As a rapid source of NO for blood vessel dilation

Why do angina patients carry nitroglycerine with them?

As a rapid source of NO for blood vessel dilation

Why do angina patients carry nitroglycerine with them?

As a rapid source of NO for blood vessel dilation.

What amino acid feed into Oxaloacetate? Citrate Cycle

Asparagine

Which amino acid provides the majority of carbons in the pyrimidine ring system?

Aspartate

Which reaction explains why animals fed 15N-Asp will accumulate 15N-labeled proteins?

Aspartate + a -keto acid ® oxaloacetate + a -amino acid

How many rounds of ubiquitination must a condemned protein undergo to facilitate proteasomal recognition?

At least three times is sufficient.

Which of the diets below relies on high fat and low carbohydrates to stimulate ketogenesis?

Atkins diet

A person who only expresses the GTB enzyme and not the GTA enzyme will have which blood type?

B

A person who only expresses the GTB enzyme and not the GTA enzyme will have which blood type? O A B AB

B

Which of the following is an intermediate in the conversion of AMP to uric acid but NOT an intermediate in the conversion of GMP to uric acid?

B

Which of the following is an intermediate in the conversion of AMP to uric acid but NOT an intermediate in the conversion of GMP to uric acid? A B C D

B

3. (6 pts) Phenylketonuria is caused by a defect in the enzyme phenylalanine hydroxylase (1) / phenylalanine aminotransferase (2). The symptoms of the disorder are caused by substrate accumulation (3) / lack of product (4) and can be treated by diet which limits phenylalanine (5) / tyrosine (6). Albinism is characterized by white skin and hair due to a defect in the enzyme arginase (7) / tyrosinase (8) and is characterized by a substrate accumulation (9) / lack of product (10) mechanism.

B. 1, 3, 5, 8, 10

Phenylketonuria is caused by a defect in the enzyme phenylalanine hydroxylase (1) / phenylalanine aminotransferase (2). The symptoms of the disorder are caused by substrate accumulation (3) / lack of product (4) and can be treated by diet which limits phenylalanine (5) / tyrosine (6). Albinism is characterized by white skin and hair due to a defect in the enzyme arginase (7) / tyrosinase (8) and is characterized by a substrate accumulation (9) / lack of product (10) mechanism.

B. 1, 3, 5, 8, 10

14. (6 pts) What is the biochemical mechanism by which proteins are digested in the small intestine?

B. Enteropeptidase cleaves trypsin, which initiates a proteolytic cascade

A nutritional deficiency in Vitamin D causes rickets and is best described as:

B. Environment effect

15. (6 pts) A nutritional deficiency in Vitamin D causes rickets and is best described as

B. Environment effect.

24. (6 pts) What are two problems that have been identified owing to overuse of glyphosate in agriculture?

B. Native plants become glyphosate-resistance and increased human health problems, including cancer.

The human genome encodes three types of enzymes in the ubiquitinating pathway, all with multiple variant gene copies; 11 E1 variant genes, 26 E2 variant genes and over 500 E3 variant genes. What does this tell you about the biochemistry of the ubiquitination process?

B. That recognition of distinct target proteins requires specific E3 sequences; therefore many E3 proteins.

To determine the role of gluteredoxin in the generation of deoxyribonucleotides by ribonucleotide reductase, which organisms would be the best to study?

Bacteria

Why does it makes sense that ghrelin stimulates orexigenic neurons (eat more/metabolize less)?

Because gherlin is released from stomach cells when your stomach is empty.

Why does it make sense that ghrelin stimulates orexigenic neurons (eat more/metabolize less)?

Because ghrelin is released from stomach cells when your stomach is empty

Why does it makes sense that ghrelin stimulates orexigenic neurons (eat more/metabolize less)?

Because ghrelin is released from stomach cells when your stomach is empty

Why does it make sense that ghrelin orexigenic neurons (eat more/metabolize less)?

Because ghrelin is released from stomach cells when your stomach is empty.

Why does it makes sense that ghrelin stimulates orexigenic neurons (eat more/metabolize less)?

Because ghrelin is released from stomach cells when your stomach is empty.

Why aren't PKU patients albinos since they cannot synthesize tyrosine from phenylalanine?

Because they obtain tyrosine in their diet and make melanin

How do the effects of leptin and insulin signaling on anorexigenic and orexigenic neurons compare?

Both activate anorexigenic neurons and inhibit activation of orexigenic neurons.

How are anorexigenic and orexigenic neurons effected by leptin and insulin signaling?

Both hormones activate anorexigenic neurons and inhibit activation of orexigenic neurons.

In an experiment designed to monitor amino acid metabolism, cats were starved for 12 hours and then fed a meal containing either all 20 amino acids (control cats) or a meal containing only 19 amino acids lacking arginine (experimental cats). What was observed two hours after feeding?

Both sets of cats produce NH4+, but only the control cats were able to remove NH4+ by the urea cycle

Skeletal muscle proteins are degraded.

Brain cells begin using fatty acids as an energy source.

Which of the following does NOT occur during starvation?

Brain cells begin using fatty acids as an energy source.

Identify the carbon atom that is attacked by Cys439 in the first step of the E. coli ribonucleotide reductase mechanism.

C

Identify the carbon atom that is attacked by Cys439 in the first step of the E. coli ribonucleotide reductase mechanism. A B C D

C

If HCO3 that is radioactively labeled with 13C is available, which carbon atom in the molecule below would ultimately be expected to be radioactive?

C

If HCO3- that is radioactively labeled with 13C is available, which carbon atom in the molecule below would ultimately be expected to be radioactive?

C

If HCO3 that is radioactively labeled with 13C is available, which carbon atom in the molecule below would ultimately be expected to be radioactive?

C (Amide)

In the elongation of glycogen, activated glucose units are attached to which hydroxyl of the terminal residue of the growing glycogen chain?

C-4

In the elongation of glycogen, activated glucose units are attached to which hydroxyl of the terminal residue of the growing glycogen chain? C-1 C-3 C-4 C-6

C-4

) What explains dramatic weight loss in people who ingest dinitrophenol (DNP) as a diet strategy?

C. DNP depletes ATP levels in the liver leading to increased rates of fatty acids derived from fat cells

8. (6 pts) Glucagon circulates throughout the body, but has no signaling effect in brain and muscle. Why?

C. Energy produced in muscle and brain is only utilized by these tissues themselves, and is never exported, so there is no need for these tissues to express glucagon receptors.

12. (6 pts) A deficiency in the enzyme G6PD causes favism and is described as

C. Genes --->Environment effect.

A deficiency in the enzyme G6PD causes favism and is best described as:

C. Genes to Environment effect.

10. (6 pts) Explain why glyphosate works faster in the summer after rain than after two weeks without rain.

C. It works faster after it rains because the plants grow faster and run out of amino acids quicker.

What is the name of the syndrome characterized by a decreased amount of pigment in hair, skin, and eyes; if a change in diet leads to increased pigmentation, what was missing from the diet?

C. Kwashiorkor; tyrosine

Five nucleotide bases are found in nucleic acids. Name a purine/pyrimidine in that order.

C. adenine/uracil

Which of the following amino acids is a starting material for the synthesis of heme?

C. glycine

Which group is responsible for activating phosphatidic acid for the synthesis of phospholipids?

CTP

Which group is responsible for activating phosphatidic acid for the synthesis of phospholipids? CoA ATP CO2 CTP

CTP

Which of the following statements is true regarding the regulation of pyrimidine biosynthesis in animal cells?

CTP negatively regulates CTP synthase, and therefore CTP negatively regulates its own biosynthesis.

Which of the following statements is true regarding the regulation of pyrimidine biosynthesis in E. coli?

CTP negatively regulates carbamoyl aspartate synthesis.

CTP synthetase is inhibited by __________ and activated by __________.

CTP; GTP

CTP synthetase is inhibited by __________ and activated by __________. CTP; GTP CTP; UMP ATP; GTP GTP; UMP

CTP; GTP

Which of the following is FALSE concerning dihydropyrimidine dehydrogenase?

Cancer patients with dihydropyrimidine dehydrogenase deficiencies do not face tissue toxicity when treated with standard amount of 5-fluorouracil

Which of the following is FALSE concerning dihydropyrimidine dehydrogenase?

Cancer patients with dihydropyrimidine dehydrogenase deficiencies do not face tissue toxicity when treated with standard amount of 5-fluorouracil.

Which of the following substances is an intermediate produced in both pyrimidine biosynthesis and the urea cycle? Only one answer is correct.

Carbamoyl phosphate

Of the three diets listed below, which would provide the most complete nutrition? 1. Carbs + protein 2. Carbs + fats 3. Proteins + fats

Carbs + protein Because proteins are required and fats can be synthesized from carbohydrates, but carbs can't be synthesized from fats.

What would a possible outcome be if all eukaryotic cells had telomerase activity?

Cells would live indefinitely.

What would a possible outcome be if all eukaryotic cells had telomerase activity? Cells would die sooner. Cells would have increased synthesis. The rate of synthesis would increase. Cells would live indefinitely.

Cells would live indefinitely.

What is the probable diagnosis for Case Study "It is Good to be King?"

Cheilosis

Nucleotides have four key roles in biochemical processes. Name TWO of the possible four.

Coenzyme-dependent reactions and energy conversion.

Compared to other chemotherapy drugs, cancer patients are commonly administered high doses of 5-fluorouracil. When individuals with a dihydropyrimidine dehydrogenase deficiency are unknowingly given these same high doses of 5-fluorouracil, dangerous tissue toxicity arises because most of the 5-fluorouracil remains in the circulatory system rather than being degraded. Cancer patients being considered for 5-fluorouracil treatment are now routinely screened for dihydropyrimidine dehydrogenase deficiencies.

Compared to other chemotherapy drugs, cancer patients are commonly administered high doses of 5-fluorouracil. When individuals with a dihydropyrimidine dehydrogenase deficiency are unknowingly given these same high doses of 5-fluorouracil, dangerous tissue toxicity arises because most of the 5-fluorouracil remains in the circulatory system rather than being degraded. Cancer patients being considered for 5-fluorouracil treatment are now routinely screened for dihydropyrimidine dehydrogenase deficiencies.

There are many diseases collectively called porphyrias that are characterized by the accumulation of heme precursors in the blood and liver. Which type of porphyria is characterized by low uroporphyrinogen III synthesis?

Congenital erythropoietic porphyria

What is the name of the disease that causes the following symptoms; red urine, reddish brown teeth fluorescent teeth, sensitivity to sunlight, and anemia

Congenital erythropoietic porphyria

The function of islets of Langerhans in the pancreas is to:

Control blood glucose levels by releasing insulin from beta cells and glucagon from alpha cells.

The wobble hypothesis was first proposed by

Crick

The wobble hypothesis was first proposed by Leder. Nirenberg. Holley. Crick.

Crick.

The standard change in free energy for the glutamate dehydrogenase reactions is +30 kJ/mol. The reaction shown in figure 17.11 would therefore be unfavorable. Under what conditions might the assimilation of NH4+ into glutamate be favorable? Explain your answer by referring to the mass action ratio

Crop plants are often exposed t very high levels of (NH4)2SO4 (ammonium sulfate), which will lead to a more favorable delta G 0 because the mass action ratio [P]/[S] will be a number much less than 1. The natural log (ln) of a small number, is a (-) number, making the actual change in free energy (delta G) more favorable even with an unfavorable delta G 0 of 30 kJ/mol.

The importance of stable free radicals in the mechanism of E. coli ribonucleotide reductase was identified in the early 1970s. The existence of stable radicals in a protein had not been previously observed. Which of the following amino acids can exist as a stable radical in ribonucleotide reductase?

Cys

Which reaction explains why animals fed 15N-Asp will accumulate 15N-labeled proteins?

D. Aspartate + a -keto acid ® oxaloacetate + a -amino acid

9. (6 pts) Which of these three diets provides the most complete nutrition and why?Diet 1: carbohydrate + protein Diet 2: carbohydrate + fats Diet 3: protein + fats

D. Diet 1; because it contains essential amino acids and fats are synthesized from carbohydrates.

Why is de novo biosynthesis of purines markedly elevated in patients with a deficiency in hypoxanthine-guanine phosphoribosyl transferase (HGPRT)?

D. HGPRT deficiency prevents purine salvage, leading to increased purine biosynthesis to make up for it.

which mechanism might be overcome by developing a methotrexate derivative using structure based drug design

DHFR mutation is most likely to be overcome with new drug design

Which of the following would cause a genetic mutation?

DNA damage is not corrected

Which of the following would cause a genetic mutation? DNA makes too many copies in the cell. DNA synthesis does not occur. DNA damage is not corrected. Death of a cell occurs.

DNA damage is not corrected.

Which of the following is a specialized direct DNA repair process?

DNA photolyase system

Which of the following is a specialized direct DNA repair process? DNA photolyase system nucleotide excision repair base excision repair DNA mismatch repair

DNA photolyase system

Which enzyme synthesizes the RNA primer for DNA replication?

DNA primase

Which enzyme synthesizes the RNA primer for DNA replication? DNA polymerase DNA primase DNA polymerization DNA Pol III

DNA primase

What do sigma factors bind to in RNA synthesis?

DNA promoters and RNA polymerase

The definition of a retrotransposon is a segment of

DNA that goes through an RNA intermediate to be moved.

The definition of a retrotransposon is a segment of DNA that goes through an RNA intermediate to be moved. RNA that goes through a DNA intermediate to be moved. DNA that needs to be excised. DNA that is folded incorrectly.

DNA that goes through an RNA intermediate to be moved.

What explains dramatic weight loss in people who ingest dinitrophenol (DNP) as a diet strategy?

DNP depletes ATP levels in the liver leading to increased rates of fatty acids derived from fat cells.

Explain how a deficiency in the enzyme dihydropyrimidine dehydrogenase (DPD) could lead to severe side effects in cancer treatments using 5-fluorouracil (5-FU) as a chemotherapeutic agent.

DPD degrades 5-FU so high doses need to be given, if DPD is deficient, then high 5-FU doses are toxic.

Explain how a deficiency in the enzyme dihydropyrimidine dehydrogenase (DPD) could lead to severe side effects in cancer treatments using 5-fluorouracil (5-FU) as a chemotherapeutic agent.

DPD degrades 5-FU so high doses need to begiven, if DPD is deficient, then high 5-FU doses are toxic

The pyrimidine thymine is only found in DNA, whereas the pyrimidine uracil is on found in RNA. What is the one BEST explanation for this observation?

Deamination of cytosine generates uracil, which will result in a G-C to A-T mutation in DNA if uracil is allowed to persist. Evolution solved this problem by selecting for enzymes that constantly remove uracil from DNA

The pyrimidine thymine is only found in DNA, whereas the pyrimidine uracil is on found in RNA. What is the one BEST explanation for this observation?

Deamination of cytosine generates uracil, which will result in a G-C to A-T mutation in DNA if uracil is allowed to persist. Evolution solved this problem by selecting for enzymes that constantly remove uracil from DNA.

Undiagnosed phenylketonuria (PKU) can lead to severe neuronal damage and mental incapacity, which is avoidable if the disease is diagnosed at birth. In contrast, albinism is easly diagnosed at birth, but there is no treatment, which leads to an increased risk of skin cancer in these patients. Choose the THREE statements that best explain the reasons for these differences.

Defects in the enzyme tyrosinase occur in every cell of the body and there is no way to replace this enzyme without embryonic human gene therapy. In contrast, defects in the enzyme phenylalanine hydroxylase can be ameliorated by restricting certain amino acids in the diet. Phenylalanine is found in many types of food, including aspartame, and therefore by avoiding foods with high amounts of phenylalanine, the build-up pyruvate metabolites can be controlled. In contrast, skin pigments must be made from tyrosine and there is no way to do this if tyrosine metabolism is compromised. PKU is the result of substrate accumulation, which can be avoided using a restricted diet. However, albinism is due to a missing product, which cannot be replaced through injections or dietary regimens.

What is the biochemical basis for Case Study 1.2 "More Gin, less tonic - please?"

Deficiency in the enzyme glucose-6P dehydrogenase

What is the biochemical basis for case study 1.2 "More Gin, Less Tonic - Please?"

Deficiency in the enzyme glucose-6P dehydrogenase

What is the biochemical basis for Case Study 3.1 "Squidward Q. Tentacles?"

Deficiency in the enzyme homogentisate 1,2-dioxygenase

What is the biochemical basis for Case Study 3.2 "From Russia Needs Love?"

Deficiency in the enzyme phenylalanine hydroxylase

What is the biochemical basis for Case Study 2.2 "It is good to be king?"

Deficiency in the enzyme porphobilinogen deaminase

What is the biochemical basis for Case Study 2.2 "It is Good to be King?"

Deficiency in the enzyme prophobilinogen deaminase

What is the biochemical basis for Case Study 2.1 "Bah, Humbug?"

Deficiency in vitamin D

9. (6 pts) Which of these three diets provides the most complete nutrition and why?

Diet 1: carbohydrate + protein D. Diet 1; because it contains essential amino acids and fats are synthesized from carbohydrates.

Of the three diets below, which would provide the most complete nutrition and why? 1. carbohydrate and protein 2. carbohydrate and fat 3. protein and fat

Diet no. 1 is the most complete because proteins are required (nine essential amino acids), and fats can be synthesized from carbohydrates, but carbohydrates cannot be synthesized from fats

What explains the dramatic weight loss associated with dinitrophenol pills? What are the possible dire consequences of using dinitrophenol pills to lose weight?

Dinitrophenol can lead to weight loss because it mimics the "starved" state and fat is catobilized at a high rate, but long-term use causes fatal liver damage

What explains the dramatic weight loss associated with dinitrophenol pills? What are the possible dire consequences of using these pills to lose weight?

Dinitrophenol is an uncoupler of mitochondrial ATP synthesis, leading to high rates of fatty acid oxidation to compensate for the resulting low energy charge. Dinitrophenol can lead to weight loss because it mimics the "starved" state and fat is catabolized at a high rate, but it has been reported that long-term dinitrophenol use causes fatal liver damage due to hepatocyte apoptosis

Which catecholamine is made by the action of tyrosine hydroxylase and aromatic amino acid decarboxylase? (Assume that a biologically relevant molarity of all substrates for both tyrosine hydroxylase and aromatic amino acid decarboxylase are available.

Dopamine

17. (6 pts) Nucleotides have four key roles in biochemical processes. Name TWO of the possible four.

E. Coenzyme-dependent reactions and energy conversion

17. (6 pts) Nucleotides have four key roles in biochemical processes. Name TWO of the possible four.

E. Coenzyme-dependent reactions and energy conversion.

23. (6 pts) What are three mechanisms by which cancer cells become resistant to the drug methotrexate?

E. DHFR mutation blocking drug binding, Overexpression of drug transporter, DHFR gene amplification

Which of the following best explains why nucleotide salvage pathways evolved?

E. It is energetically expensive for the cell to solely rely on de novo biosynthetic pathways

13. (6 pts) Which of the following best explains why nucleotide salvage pathways evolved?

E. It is energetically expensive for the cell to solely rely on de novo biosynthetic pathways.

To purify glutathione reductase to carry out a kinetics analysis, which organism would be the best source of the enzyme?

E. coli

To purify glutathione reductase to carry out a kinetics analysis, which organism would be the best source of the enzyme? cod fish chimpanzee E. coli turtle

E. coli

After the initiation complex is complete, what occurs next in the process of translation?

EF-Tu•GTP AA-tRNAAA binding

After the initiation complex is complete, what occurs next in the process of translation? peptide bond formation EF-G•GTP binding EF-Tu•GTP AA-tRNAAA binding RF2 binding

EF-Tu•GTP AA-tRNAAA binding

Endothelial cells can receive a chemical signal from neurons. This signal is in the form of neuronal (A) acetylcholine release. This chemical messenger binds to a membrane receptor that engages a signal transduction pathway, which increases the cytoplasmic concentration of (B) calcium ions. This ionic secondary messenger then activates endothelial nitric oxide synthase.

Endothelial cells can receive a chemical signal from neurons. This signal is in the form of neuronal (A) acetylcholine release. This chemical messenger binds to a membrane receptor that engages a signal transduction pathway, which increases the cytoplasmic concentration of (B) calcium ions. This ionic secondary messenger then activates endothelial nitric oxide synthase.

Cells cannot store amino acids that accumulate as a result of protein degradation. The carbon skeletons that remain

Enter the citrate cycle

What is the biochemical mechanism by which proteins are digested in the small intestine?

Enteropeptidase cleaves trypsin, which initiates a proteolytic cascade

Extreme weight loss and liver failure owing to ingesting dinitrophenol (DNP) is best described as

Environment

Extreme weight loss and liver failure owing to ingesting dinitrophenol (DNP) is best described as:

Environment

Glucose-6P is critical in metabolic homeostasis, which mechanism controls metabolic flux?

Enzyme levels

Sort the amino acids as to whether they are considered essential or nonessential amino acids in humans. Drag the amino acid name into the appropriate bin.

Essential- Valine, threonine, lysine Nonessential-glycine, cysteine

Why was gout associated with ethanol and meat consumption?

Ethanol interferes with excretion of purine breakdown chemicals and some foods contain high levels of purines.

A purinosome, a multi-enzyme complex, is part of purine biosynthesis in which of the following?

Eurkaryotes only

Using nitrogenase to reduce N2 should require 2 ATP to be invested, but it actually takes 16 ATP. Why?

Extra energy is required to produce H2.

Using nitrogenase to reduce N2 should require 2 ATP to be invested, but it actually takes 16 ATP. Why? It is harder to break the nitrogen triple bond than expected and requires more energy input. Extra energy is required to produce H2. To regenerate the MoFe protein requires the input of ATP. To keep the nitrogenase complex associated requires an input of 4 ATP

Extra energy is required to produce H2.

Which of the following is a coenzyme that is derived from ATP?

FAD

Which of the following is a coenzyme that is derived from ATP? coenzyme Q FAD heme Fe4-S4 cluster

FAD

Which enzyme is involved in Stage 1 of IMP biosynthesis from FGAR to FGAM?

FGAM synthetase

How does ephedrine work to stimulate weight loss?

Fatty acid oxidation is increased

How does ephedrine work to stimulate weight loss?

Fatty acid oxidation is increased.

What is the genetic sex of the cell donor in Lesch-Nyhan?

Female because it is an X-linked recessive genetic disorder

Which of the following is true concerning the glycemic index?

Foods with a high glycemic index lead to a refractory period where blood glucose levels drop below the normal fasting state (*The glycemic index for a food is the ratio of blood glucose levels for the test food divided by blood glucose levels for pure glucose. High glycemic index foods lead to a sudden rise in blood glucose levels, followed by a refractory period. Low glycemic index foods lead to a more gradual increase in blood glucose and insulin levels)

Which of the following is true concerning the glycemic index?

Foods with a high glycemic index lead to a refractory period where blood glucose levels drop below the normal fasting state.

The same four reactions that are central to the beta-oxidation pathway are also present in the fatty acid synthesis pathway, except they are reversed. What is a key difference between the four reactions in these two pathways?

Four enzymes are used in one, whereas one enzyme is used in the other.

In the Krebs bicycle, which of the following citrate cycle intermediates links to the urea cycle via argininosuccinate?

Fumarate

If a cell has many ROS species, what is the most likely reaction with DNA?

G-C to T-A replacement

If a cell has many ROS species, what is the most likely reaction with DNA? Nothing, because ROS do not react with DNA. G-C to T-A replacement O-alkylation of thymine single point excision

G-C to T-A replacement

In an iteration of the Nirenberg-Leder experiment to assign triplet codons to specific amino acids, radioactively labeled aminoacyl-tRNA with the anticodon of CUG was used. The radioactivity was retained on the filter at the end of the experiment. Which mRNA was used in this iteration of the experiment?

GAC

In an iteration of the Nirenberg-Leder experiment to assign triplet codons to specific amino acids, radioactively labeled aminoacyl-tRNA with the anticodon of CUG was used. The radioactivity was retained on the filter at the end of the experiment. Which mRNA was used in this iteration of the experiment? CAG GAC CTG GTC

GAC

An inhibitor of __________ would specifically prohibit the release of Ran from importin

GAP

An inhibitor of __________ would specifically prohibit the release of Ran from importin. GTP-GDP exchange in Ran exportin the signal peptide peptidase GAP

GAP

Which enzyme is involved in Stage 1 of IMP biosynthesis from 5PRA to glycinamide (GAR)?

GAR synthetase

Which enzyme is involved in Stage 1 of IMP biosynthesis from glycinamide (GAR) to FGAR?

GAR transformylase

A person wishes to determine the role of PRPP synthetase in a newly discovered organism. They decide that a good way to do this would be to see what happens when PRPP synthetase is inhibited. Which of the following could be added to a cell from this organism to inhibit PRPP synthetase?

GDP

If a mutation of the decarboxylase enzyme that functions in the biosynthetic pathway of UMP caused the pathway to halt at that step, which of the following would no longer occur?

Generation of CO2

If a mutation of the decarboxylase enzyme that functions in the biosynthetic pathway of UMP causes the pathway to halt at that step, which of the following would no longer occur?

Generation of CO2

A deficiency in the enzyme tyrosinase causes albinism and is best described as

Genes effect.

Which of the following statements is FALSE concerning the effects of insulin and glucagon?

Glucagon stimulates glucose export in skeletal muscle

Which of the following statements is FALSE concerning the effects of insulin and glucagon?

Glucagon stimulates glucose export in skeletal muscle.

The Thrifty Gene Hypothesis states that humans have survival gene variants that provide protection against a "feast or famine" lifestyle, which cause obesity in societies where it is always feast and never famine. Which is the ONE best candidate for a thrifty gene variant based on its likely function?

Glucokinase with increased affinity for glucose

Insulin signals for an increase in glucose uptake in both skeletal muscle and adipose tissue. Which of the following correctly describes how these organs use this glucose?

Glucose in muscle cells is completely oxidized or stored as glycogen, depending on the ATP demand. In adipose tissue, glucose is used to synthesize fatty acids and glycerol.

Symptoms: 1. a young woman receives sulfadiazine treatment for urinary infection 2. After 3 weeks of antibiotics, patient is lethargic but has no infection 3. Leg cramps and an irritable demeanor 4. Yellow epidermis and sclera and brown urine 5. low erythrocytes and high bilirubin in the blood 6. Father is from Kenya and maternal grandparents are from Italy

Glucose-6P dehydrogenase deficiency

Within the image, identify the enzymes involved in Stage 1 of IMP biosynthesis by dragging the enzyme name to its target.

Glutamine-PRPP amidotransferase -> Gar synthetase-> GAR transformylase -> FGAM synthetase -> AIR synthetase

What amino acid feed into Pyruvate to acetyl-CoA? Citrate cycle

Glycine

Which of the following amino acids are required in the first step of heme biosynthesis?

Glycine

Which of the following contributes both carbon and nitrogen to the purine ring system?

Glycine

Which of the following is a starting material for the synthesis of heme

Glycine

Which of the following is a starting material for the synthesis of heme?

Glycine

Which of the following is inactivated by AMPK?

Glycogen synthase

AMP-dependent protein kinase (AMPK) is activated by low energy charge in the cell, which is the result of direct AMP binding to the g subunit and phosphorylation of Thr172. Name TWO enzymes that are inhibited by AMPK activation and thereby lead to increased ATP synthesis and a higher energy charge?

Glycogen synthase and acetyl-CoA carboxylase

AMP-dependent protein kinase (AMPK) is activated by low energy charge in the cell, which is the result of direct AMP binding to the g subunit and phosphorylation of Thr172. Name TWO enzymes that are inhibited by AMPK activation and therby lead to increased ATP synthesis and a higher energy charge?

Glycogen synthase and acetyl-CoA carboxylase.

What is the mechanism by which Roundup works?

Glyphosate is a competitive inhibitor to plant EPSP synthase

What is the mechanism by which Roundup works?

Glyphosate is a competitive inhibitor to plant EPSP synthase.

A person unable to produce new melanocytes is likely to have

Gray Hair

Which of the following statements correctly describes an interaction between the ATP and GTP biosynthetic pathways?

Greater ATP production will positively regulate GTP synthesis because ATP is involved in the conversion of xanthosine-5'-monophosphate into guanosine-5'-monophosphate.

What molecules are missing from boxes in the gluconeogenesis reaction shown below?

H2O, Pi

Nonprimate mammals convert uric acid to allantoin. This reaction, catalyzed by urate oxidase, also generates which of the following products?

H2O2

What molecules are missing from boxes in the gluconeogenesis reaction shown below?

H2O; Pi

What molecules are missing from boxes in the gluconeogenesis reaction shown below? Ser-Pi; Ser-Pi H2O; Pi ADP; ATP ATP; ADP

H2O; Pi

An enzyme panel analysis of a patient reveals a lack of __________, which is an indication of Lesch-Nyhan syndrome.

HGPRT

Why is de novo biosynthesis of purines markedly elevated in patients with a deficiency in hypoxanthine-guanine phosphoribosyl transferase (HGPRT)?

HGPRT deficiency prevents purine salvage, leading to increased purine biosynthesis to make up for it.

The conversion of hypoxanthine to IMP is catalyzed by __________, which is deficient in __________.

HGPRT; Lesch-Nyhan syndrome

Glycan arrays can be coupled with __________ to qualitatively compare fractionated cell extracts from different sources.

HPLC

Why do helicase and gyrase need to work together?

Helicase unwinds DNA and gyrase relieves the torsional strain.

Why do helicase and gyrase need to work together? Gyrase adds the RNA primer and helicase removes it. Helicase unwinds DNA and gyrase relieves the torsional strain. Helicase synthesizes DNA and gyrase prevents helicase from dissociating. Gyrase synthesizes RNA primers and helicase.

Helicase unwinds DNA and gyrase relieves the torsional strain.

The enzyme pyruvate carboxylase is expressed at abnormally high levels.

Higher blood glucose levels

The glucagon receptor is always active, even in the absence of glucagon.

Higher blood glucose levels

Which amino acid is NOT produced from the shikimate pathway product chorismate?

Histidine

Match the patient symptoms on the left with the most likely biochemical basis on the right. Chip was home schooled from K-12 and then went away to college Although he lived a sheltered life, he enjoyed the idea of meeting people First night in college he got drunk and urinated in the swimming pool People cleared the pool when a cloud of blackish water surrounded Chip Chip was surprised that people outside his family have yellow urine Chip's frat house nicknamed him Squidward - for a very good reason

Homogentisate 1,2-dioxygenase deficiency

Symptoms: 1. Home schooled boy now at college 2. Enjoyed the "idea" of meeting people 3. First night of college, got drunk and peed in the pool 4. The water turned black where he urinated and he was surprised to learn other people have yellow urine

Homogentisate 1,2-dioxygenase deficiency

Which of the following best defines nitrogen balance?

Human daily intake of nitrogen, primarily in the form of protein, equals the amount of nitrogen lost by excretion in the feces and urine.

Which of the following best defines nitrogen balance? A. Human daily intake of nitrogen, primarily in the form of protein, equals the amount of nitrogen lost by excretion in the feces and urine. B. Bacterial fixation of nitrogen, primarily in the form of ammonia, equals the amount of nitrification that takes place of Nitrobacter. C. The rate of atmospheric nitrogen fixation is equal to the rate of biological nitrogen fixation, which is equal to the rate of industrial nitrogen fixation. D. The rate of denitrification by Pseudomonas is balanced with the rate of all forms of nitrogen fixation minus the rate of nitrification by Nitrosomonas.

Human daily intake of nitrogen, primarily in the form of protein, equals the amount of nitrogen lost by excretion in the feces and urine.

Individuals who develop type 2 diabetes are often overweight or obese (i.e., exhibit hyperlipidemia), although there is no correlation between weight and the development of type 1 diabetes. Why does obesity contribute to the development of type 2 diabetes?

Hyperlipidemia reduces adiponectin release from adipose tissue.

Activation of __________ in __________ leads to downregulation of adiponectin.

IKK; adipocytes

Which protein will be phosphorylated in a muscle cell in an obese individual with elevated serum levels of free fatty acids?

IRS1

Which protein will be phosphorylated in a muscle cell in an obese individual with elevated serum levels of free fatty acids? protein kinase C insulin receptor IRS1 GLUT1

IRS1

Kwashiorkor Syndrome is a dietary deficiency characterized by decreased pigment in the skin and hair. What would you conclude if adding back this missing food group had not effect on the skin and hair condition?

If adding back protein (tyrosine) does not result in coloration of skin and hair after an extended period of time, then there is a defect in the pigment biosynthetic pathway converting tyrosine to pigments.

Gout is due to a build-up of uric acid crystals, which can be due to either excess production of uric acid by the purine biosynthetic pathway (Figure 18.17), or or inability to excrete uric acid leading to a build-up. Considering that uric acid is derived from both purine biosynthesis and by diet, explain how feeding a patient 15N-glycine and measuring the amount of 115N in excreted urine can distinguish between these two causes of gout?

If there is 15N in the urine that most likely a result of excess production of uric acid since 15N-glycine will incorporate 15N into purines and then uric acid that is excreted because in excess (cause of the gout). If however, there is very little 15N in the urine, then likely inability to break down or excrete uric acid since most of the uric acid normally comes from the diet and it would not therefore be 15N labeled.

Gout can be due to either excess production of purines by de novo synthesis (defect A) or inability to excrete excess dietary uric acid properly (defect B). Explain how feeding a patient 15N-glycine, and determining the amount of 15N in their excreted uric acid, could be used to distinguish between defect A and defect B.

In defect A, large amounts of 15N are incorporated in uric acid because glycine contribute a nitrogen to the purine ring in de novo synthesis. in defect B, uric acid should be relatively free of 15N because much of the excess uric acid is due to purines in the diet, not as a result of de novo purine biosynthesis.

Gout can be due to either excess production of purines by de novo synthesis (defect A) or inability to excrete excess dietary uric acid properly (defect B). Explain how feeding a patient 15N-glycine and determining amount of N15 in their excreted uric acid could be used to distinguish between A and B defects.

In defect A, large amounts of 15N are incorporated in uric acid because glycine contributes a nitrogen to the purine ring in de novo synthesis. In defect B, uric acid should be relatively free of 15N because much of the excess uric acid is due to purines in the diet, not as a result of de novo purine biosynthesis

Kwashiorkor is a dietary deficiency disease characterized by decreased pigment in the skin and hair. What would you conclude if adding this missing food group to the diet had no effect on the skin and hair condition?

In such a case, the individual may have a defect in an enzyme required to convert tyrosine to melanin.

Hyperglycemia can lead to hyperlipidemia, which reinforces the hyperglycemic state. Place the steps in order to describe the process by which hyperlipidemia reinforces the hyperglycemic state.

Increased TNF-a expression Reduced adiponectin expression Reduced AMPK activation Reduced GLUT4 expression

Why is ketogenesis stimulated during starvation?

Increased gluconeogenesis and increased fatty acid oxidation

Why is ketogenesis stimulated during starvation?

Increased gluconeogenesis and increased fatty acid oxidation.

Why is ketogenesis stimulated during starvation? Choose the one BEST answer

Increased gluconeogenesis and increased fatty acid oxidation.

Why is ketogenesis stimulated during starvation? Choose the one BEST answer.

Increased gluconeogenesis and increased fatty acid oxidation.

Why is ketogenesis stimulated during starvation? Choose the one best answer.

Increased gluconeogenesis and increased fatty acid oxidation.

Patients with type 1 diabetes must inject insulin up to several times a day and monitor their carbohydrate intake to regulate their blood glucose level. The amount of insulin injected is based on the carbohydrate content of the diet and so must be adjusted accordingly; otherwise, hypoglycemia will result. Which of the following explain(s) why injecting too much insulin before a meal results in hypoglycemia?

Increased glucose uptake, Increased glycogen storage, Decreased glucose synthesis

What explains the observation that individuals with phenylketonuria have pigmented skin and hair considering that phenylalanine hydroxylase catalyzes the conversion of phenylalanine to tyrosine and tyrosine is the precursor to melanin pigments?

Individuals with phenylketonuria (PKU) have pigmented skin and hair because they obtain tyrosine in the diet. This dietary tyrosine is converted to melanin pigments. It is true however that children with PKU are often light skinned and have a higher risk of skin cancer because dietary tyrosine may not be sufficient.

What explains the observation that individuals with phenylketonuria have pigmented skin and hair considering that phenylalanine hydroxylase catalyzes the conversion of phenylalanine to tyrosine and tyrosine is the precursor to melanin pigments?

Individuals with phenylketonuria (PKU) have pigmented skin and hair because they obtain tyrosine in the diet. This dietary tyrosine is converted to melanin pigments. It is true however that children with PKU are often light skinned and have a higher risk of skin cancer because dietary tyrosine my not be sufficient

Explain why individuals with phenylketonuria are not complete albinos, even though they cannot synthesize tyrosine from phenylalanine.

Individuals with phenylketonuria have tyrosinase and obtain tyrosine in their diet.

Why do individuals with both type 1 and type 2 diabetes have elevated blood glucose levels but respond differently to the same dose of injected insulin?

Individuals with type 1 diabetics are insulin deficient, but the intracellular insulin signaling pathways are inhibited in individuals with type 2 diabetes.

What explains the smell of acetone on the breath of dieters following the Atkins diet?

Insufficient carbohydrates to maintain flux through the citrate cycle leads to increased ketogenesis

What explains the smell of acetone on the breath of dieters following the Atkins diet?

Insufficient carbohydrates to maintain flux through the citrate cycle leads to increased ketogenesis.

Normal blood glucose concentration is considered to be 4.5 mM. Changes in blood glucose can lead to changes such that the level of insulin is greater than the level of glucagon (insulin>glucagon) or the level of insulin is less than that of glucagon (insulin<glucagon).Which statement correctly describes the ratio of insulin to glucagon release by the pancreas and how this ratio is affected by changes in serum glucose?

Insulin and glucagon are released simultaneously; insulin>glucagon when glucose in the blood exceeds 4.5 mM, whereas insulin < glucagon if this concentration drops below 4.5 mM

normal blood glucose concentration is considered to be 4.5 mM. Changes in blood glucose can lead to changes such that the level of insulin is greater than the level of glucagon (insulin>glucagon) or the level of insulin is less than that of glucagon (insulin<glucagon).Which statement correctly describes the ratio of insulin to glucagon release by the pancreas and how this ratio is affected by changes in serum glucose?

Insulin and glucagon are secreted continuously at different levels; insulin>glucagon when glucose in the blood exceeds 4.5 mM, whereas insulin<glucagon if this concentration drops below 4.5 mM.

Pancreatic β cells release insulin from intracellular vesicles. This release is regulated in part by the activity of glucokinase. How would insulin release be affected if pancreatic β cells were mutated to express hexokinase rather than glucokinase?

Insulin release from pancreatic β cells would be constant if hexokinase was expressed rather than glucokinase, and expression of insulin would not be affected by changes in the serum glucose.

How do we deal with the double bonds during the degradation of unsaturated fatty acids?

Isomerize the position of the double bonds to converge them with the beta-oxidation pathway.

Which of the following is true of the glyoxylate cycle?

It allows plants to produce glucose from fats and two-carbon molecules like acetate

Which of the following best explains why nucleotide salvage pathways evolved?

It is energetically expensive for the cell to solely rely on de novo biosynthetic pathways.

You breathe fast after short-term vigorous exercise (sprint), which provides O2 for oxidative phosphorylation and ATP synthesis. What is this newly synthesized ATP used for in liver and muscle cells?

It is used in the liver cells to convert lactate into glucose in gluconeogenesis. It is used in muscle cells to phosphorylate creatine to replenish it.

Explain why glyphosate works faster in the summer after rain than after two weeks without rain

It works faster after it rains because plants take up glyphosate, which quickly inhibits EPSP synthase.

Explain why glyphosate works faster in the summer after rain than after two weeks without rain?

It works faster after it rains because the plants grow faster and run out of amino acids quicker

What might occur if a mutation in the leptin receptor causes it to be always activated, even in the absence of leptin?

JAK2 will be activated.

What might occur if a mutation in the leptin receptor causes it to be always activated, even in the absence of leptin? JAK2 will be activated. a-MSH secretion will be low. NPY secretion will be high. AGRP secretion will be high.

JAK2 will be activated.

Kwashiorkor Syndrome is a dietary deficiency characterized by decreased pigment in the skin and hair. Which nutrient is likely missing from the diet? Explain your answer

Kwashiorkor Syndrome is due to a deficiency in the amino acid tyrosine, which would occur if the diet is lacking in sufficient protein. Tyrosine is the precursor to all major skin and hair pigments.

Kwashiorkor is a dietary deficiency disease characterized by decreased pigment in the skin and hair. Which major food group is likely missing from the diet? Justify your answer.

Kwashiorkor is caused by a lack of protein in the diet. The decreased pigmentation is caused by a deficiency in tyrosine, which is the precursor to melanin pigments. Moreover, because phenylalanine is converted to tyrosine by phenylalanine hydroxylase, phenylalanine deficiency also contributes to the skin and hair condition.

What is the name of the syndrome characterized by a decreased amount of pigment in hair, skin, and eyes;; if a change in diet leads to increased pigmentation, what was missing from the diet?

Kwashiorkor; tyrosine

Which of the following is the reaction for nitric oxide synthase?

L-arginine + 1.5 NADP+ + H+ + 2O2 citrulline + NO + 1.5 NADPH

Which lipid is thought to give rise to the plaques that can clog arteries?

LDL

How does depletion of liver glycogen stores during starvation lead to a shift from glucose utilization to fatty acid oxidation as an energy source for most tissues?

Lack of insulin signaling reduces GLUT4 translocation, Lack of liver glycogen stores results in a drop in serum glucose concentration, Glucagon activates hormone-sensitive lipase.

Genetic studies with OB and DB mice identified leptin as a possible regulator of obesity. Which observations in humans indicated that the relationship between leptin and obesity in mice could not be used as a model to study obesity in humans?

Leptin levels are elevated in obese humans and leptin receptor mutations are rare.

What is true of Lesch-Nyhan syndrome?

Lesch-Nyhan syndrome is a X-linked recessive disease that affects males at a higher frequency than females

The transformation of ribose-5-phosphate to AIR in the first stage of purine biosynthesis in E. coli consumes the equivalent of 5 ATP. The second stage consumes

Less than 5 ATP (3 ATP, making the total ATP consumption 8 ATP for both stages)

The Biosphere 2 project, in Tucson, Arizona, was an experiment involving a large sealed terrarium with humans and photosynthetic plants living in balance. Why did the project have to be interrupted after only a few months?

Levels of CO2 rose to dangerous levels

The brain requires a constant input of glucose. The metabolic activity of which organ(s) is/are necessary to maintain the concentration of glucose required by the brain when dietary glucose is low?

Liver + Adipose Tissue + kidney

Glycogen can provide fuel for both a short race (e.g., 200 m) and a long race (i.e., 10 K or more). Which of the following best describe(s) the use of muscle and liver glycogen during short and long races?

Liver and Muscle for long runs muscle for short runs

What is newly synthesized ATP used for in the liver and muscle cells following short-term vigorous exercise?

Liver: support gluconeogenesis to convert lactate to pyruvate to glucose Muscle: phosphorylation creatine and replenish phosphocreatine

Order the following steps involving the regeneration of ribonucleotide reductase that occurs in most animals so that it may carry out the formation of deoxyribonucleotides. (Note that not all steps are shown.) 1. Reduction of thioredoxin 2. Reduction of ribonucleotide reductase 3. Oxidation of thioredoxin reductase 4. Reduction of thioredoxin reductase

Look this up because 1,2,3,4; 4,1,2,3; 3,4,1,2; or 2,1,3,4 are all not right

Which of the following is used to induce weight loss by suppressing appetite?

Lorcaserin (*several drugs have been developed to stimulate weight loss. Ephedrine stimulates adrenergic receptor signaling, while lorcaserin targets appetite by binding to a specific serotonin receptor. Orlistat inhibits pancreatic lipase to prevent adsorption of dietary fats, while olestra serves as a fat substitute that is not metabolized)

Why does benzoic acid therapy lower blood ammonia levels in patients with deficiencies in the urea cycle considering that benzoic acid reacts with glycine to form hippuric acid, which is rapidly excreted?

Loss of glycine by hippuric acid excretion, drives more NH4+ into glycine biosynthesis to replace it.

Individuals wishing to lose weight are often advised to eat foods with a low glycemic index (GI). How could glycemic index affect weight loss?

Low GI foods can decrease the rate of fatty acid synthesis.

wishing to lose weight are often advised to eat foods with a low glycemic index (GI). How could glycemic index affect weight loss?

Low GI foods can decrease the rate of fatty acid synthesis.

Which of the following statements is FALSE concerning adiponectin?

Low levels of high-molecular-weight adiponectin leads to insulin sensitivity.

Glycogen debranching enzyme does not function during intense exercise.

Lower blood glucose levels

Pancreatic alpha cells do not secrete functional glucagon.

Lower blood glucose levels

Which of the following sequences is most likely a nuclear localization signal?

Lys-Lys-Arg-Gly-Arg

What amino acid feed into Acetoacetyl-CoA leading to ketogenesis? Citrate Cycle

Lysine

Which of the following statements concerning lysosomes is FALSE? A. The interior of a lysosome has a low pH. B. Proteases within the lysosome are non-specific. C. Lysosomes degrade a variety of macromolecules. D. Lysosomal degradation of proteins in ATP-dependent.

Lysosomal degradation of proteins in ATP-dependent.

Which of the following statements concerning lysosomes is FALSE?

Lysosomal degradation of proteins is ATP-dependent.

Other than the proteasome, there is a second organelle mechanism of protein degradation within the eukaryotic cell. Which of the following organelles is also involved in protein degradation

Lysosome

Other than the proteasome, there is a second organelle mechanism of protein degradation within the eukaryotic cell. Which of the following organelles is also involved in protein degradation?

Lysosome

Anorexigenic neurons express the __________ receptor.

MC4

Anorexigenic neurons express the __________ receptor. MC4 Y1/Y5 Y2 NPY

MC4

Which of the following is NOT a cellular role for nucleotides?

Membrane formation

Which of the following is NOT a component that is released from the ribosome on translation termination?

Met-tRNAfMet

What is an advantage of using methicillin compared with penicillin? Penicillin is susceptible to inactivation by mutant transpeptidase enzymes. Methicillin is resistant to b-lactamase activity. Penicillin does not bind transpeptidase as well as methicillin. Methicillin does not bind transpeptidase.

Methicillin is resistant to b-lactamase activity.

What is an advantage of using methicillin compared with penicillin?

Methicillin is resistant to beta-lactamase activity.

In a eukaryotic cell, deamination of amino acids takes place most extensively in the

Mitochondrial matrix

Why is it important that muscle cells are unresponsive to glucagon?

Muscle cells store glycogen for its own energy needs and not the needs of other tissues.

The DNA alkylation of guanine is possible because the __________ of guanine is __________ and reacts rapidly with __________ alkylating agents.

N-7; nucleophilic; electrophilic

The carbon atoms of a purine originate from glycine and

N10-formyl-THF

Which of the following provides the methyl group for conversion of dUMP to dTMP?

N5,N10-methylenetetrahydrofolate

Which of the following is the net reaction of the urea cycle?

NH+ + HCO3- + aspartate + 3ATP --> urea + fumerate + 2 ADP + 2Pi + AMP + PPi

Which of the following is the net reaction of the urea cycle?

NH+ + HCO3- + aspartate + 3ATP INTTO urea + fumerate + 2 ADP + 2Pi + AMP + PPi

Which of the following is the net reaction of the urea cycle? Urea + Fumerate + 2 ADP + 2Pi + AMP + PPi mc114-1.jpg NH+ + HCO3- + aspartate + 3ATP NH+ + HCO3mc114-2.jpg + aspartate + 3ATP mc114-3.jpg urea + fumerate + 2 ADP + 2Pi + AMP + PPi NH+ + HCO3mc114-4.jpg + fumerate + 3ATP mc114-5.jpg urea + aspartate + 2 ADP + 2Pi + AMP + PPi NH+ + HCO3mc114-6.jpg + aspartate + 2 ADP + 2Pi + AMP + PPi mc114-7.jpg urea + fumerate + 3ATP

NH+ + HCO3mc114-2.jpg + aspartate + 3ATP mc114-3.jpg urea + fumerate + 2 ADP + 2Pi + AMP + PPi

If GMP radiolabeled at the nitrogen atom indicated by the * were metabolized to uric acid, which of the following molecules along the pathway would also be radioactive?

NH4+

If GMP radiolabeled at the nitrogen atom indicated by the * were metabolized to uric acid, which of the following molecules along the pathway would also be radioactive? uric acid guanine NH4+ xanthine

NH4+

Which of the following represent the reactants of the net equation of the urea cycle?

NH4+, ATP, aspartate, and HCO3-

Which of the following represent the reactants of the net equation of the urea cycle? A. NH4+, urea, aspartate, and HCO3- B. NH4+, ATP, fumarate, and HCO3- C. NH4+, AMP, aspartate, and urea D. NH4+, ATP, aspartate, and HCO3-

NH4+, ATP, aspartate, and HCO3-

Which of the following statements describes nitric oxide in humans?

Nitric oxide is formed from arginine in a reaction catalyzed by nitric oxide synthase.

Nitrogen in biochemical compounds ultimately comes from what source?

Nitrogen gas

Nitrogen in biological compounds ultimately comes from what sources?

Nitrogen gas

A hyperactive hormone-sensitive lipase on lipid droplets in adipose cells. Thrifty?

No, continually hydrolized triacylglycerol in adipose tissue and make it difficult to accumulate stored fat

High basal levels of uncoupling protein in liver cells. Thrifty?

No, increases thermogenesis and raises BMR

Terri and Ronise are both runners. Terri is a short-distance runner for the university track team and competes primarily in 100-meter and 200-meter sprints. Ronise competes in local 10K races and has even completed a half-marathon. Terri notices that Ronise brings glucose gels with her on her longer training runs. Terri asks Ronise if the glucose gels could give her a boost if she ate one halfway through her 200-meter sprint. What should Ronise tell her?

No, ingesting a glucose gel during a 200-meter sprint will not provide useful fuel, and may even decrease the efficiency of ATP synthesis.

Would 5-fluorouracil be suitable for a patient that has demonstrated dihydropyrimidine dehydrogenase hyperactivity?

No, the drug would probably not remain at high enough molarity to be bioactive.

Which of the biomolecules shown at the right directly contribute nitrogen and/or carbon atoms to purine synthesis?

None of the above

Choose the one best answer that contains the correct answers to BOTH of the genetic probability questions in parts "a" and "b" below. a) What is the probability of having a child with albinism, an autosomal recessive trait, if both parents are carriers of the gene defect but neither parent is an albino? b) What is the probability that a child with acute intermittent porphyria has at least one parent with the disease?

None of the choices have both correct answers.

Choose the one best answer that contains the correct answers to BOTH of the genetic probability questions in parts "a" and "b" below. a) What is the probability of having a child with albinism, and autosomal recessive trait, if both parents are carriers of the gene defect but neither parent is an albino? b) What is the probability that a child with acute intermittent porphyria has at least one parent with the disease?

None of the choices have both correct answers. a) 25% b) 100%

Explain why glyphosate works faster in the summer after rain than after two weeks without rain.

None of these answers are correct

Order the following steps involving the regeneration of ribonucleotide reductase that occurs in most animals so that it may carry put the formation of deoxyribonucleotides. (Note that not all steps are shown.) 1. Reduction of thioredoxin 2. Reduction of ribonucleotide reductase 3. Oxidation of thioredoxin reductase 4. Reduction of thioredoxin reductase

None of these answers are correct 4, 3, 1, 2

Explain why glyphosate works faster in the summer after rain than after two weeks without rain.

None of these answers are correct.

Explain why glyphosate works faster in the summer after rain than after two weeks without rain.

None of these answers are correct. Rain does not affect it.

Which of the following is a starting material for the synthesis of heme?

None of these answers is correct

Which of the following is starting material for the synthesis of heme?

Not phenylalanine, alanine, or histidine

A person who has alkaptonuria is easily diagnosed from his or her black urine. What causes the black color

Not reduction of homogentisate, reduction of acetoacetyl-CoA, oxidation of phenylananine, or dehydration

The glycan group on glycoproteins and glycolipids on the red blood cells all contain the glycan subgroup(s)

O

Leptin is an adipocyte hormone that sends signals to the brain to "eat less" and "metabolise more." What is the explanation for why leptin injections cause weight loss in the strain of OB mutant mice but leptin injections have no effect on the majority of obese people? Is this observation analogous to type 1 or type 2 diabetes?

OB mutant mice lack leptin, and leptin injections enable them to better maintain energy balance and metabolic homeostasis. Most obese humans have chronically high leptin levels rather than a mutation in the leptin gene like the OB mice. In fact, most obese individuals are leptin insensitive. This is analogous to type 2 diabetics at initial diagnosis, who often have high levels of insulin but are insulin insensitive

What is the explanation for why leptin injections cause weight loss in the strain of OB mutant mice, but leptin injections have no effect in the majority of obese people? Is this observation analogous to type 1 or type 2 diabetes?

OB mutant mice lack leptin, and leptin injections enable them to better maintain energy balance and metabolic homeostasis. Most obese humans have chronically high leptin levels rather than a mutation in the leptin gene like the OB mice. In fact, most obese individuals are leptin insensitive. This is analogous to type 2 diabetics at initial diagnosis, who often have high levels of insulin but are insulin insensitive.

Why does feeding patients uridine or cytidine help alleviate the symptoms of orotate phosphoriosyl transferase anemia?

OMP is not produced, so UTP and CTP aren't. Uridine and Cytidine can go through an alternate pathway to make UTP and CTP. This alleviates anemia and provides feedback inhibition to decrease orotic acid excretion

What explains acetone breath on dieters following a strict low carb diet such as atkins?

On a low-carbohydrate diet, oxaloacetate, α-ketoglutarate, and pyruvate come from amino acid degradation. The acetone breath indicates that these reactions are not sufficient to keep the citrate cycle functioning at full capacity and acetyl-CoA builds up, leading to the production of ketone bodies and acetone

Which of the following diet relies on high fiber to slow down digestion?

Ornish Diet

Which of the following diets relies on high fiber to slow down digestion?

Ornish Diet (*Most diets work to reduce calories and shift the macronutrient profile so fatty acid oxidation is stimulated. The Ornish diet relies on low fat and a high fiber content to slow down absorption of food and make an individual feel fuller longer)

Which of the following diets relies high on fiber to slow down digestion?

Ornish diet

Which of the following diets relies on high fiber to slow down digestion?

Ornish diet

What are three mechanisms by which cancer cells become resistant to the drug methotrexate?

Overexpression of drug transporter, DHFR mutation blocking drug binding, DHFR gene amplification

25-Hydroxyvitamin D3 is converted to 1,25-hydroxyvitamin D3 by

P450C1 hydroxylase

Ribosomal rRNA interacts with all EXCEPT __________ during the process of translation.

PABP

Ribosomal rRNA interacts with all EXCEPT __________ during the process of translation. mRNA tRNA ribosomal proteins PABP

PABP

What is the identity of the protein labeled with the question mark in the eukaryotic preinitiation complex shown below?

PABP

Undiagnosed phenylketonuria (PKU) can lead to severe neuronal damage and mental incapacity, which is avoidable if the disease is diagnosed at birth. In contrast, albinism is easily diagnosed at birth, but there is no treatment, which leads to an increased risk of skin cancer in these patients. Choose the three statements that BEST explain the reasons for these differences.

PKU is the result of substrate accumulation, which can be avoided using a restricted diet. However, albinism is due to a missing product, which cannot be replaced though injections or dietary regimes. Defects in the enzyme tyrosinase occur in every cell of the body and there is no way to replace this enzyme without embryonic human gene therapy. IN contrast, defects in the enzyme phenylalanine hydroxylase can be ameliorated by restricting certain amino acids in the diet. Phenylalanine is found in many types of food, including aspartame, and therefore by avoiding foods with high amounts of phenylalanine, the build-up of pyruvate metabolites can be controlled. In contrast, skin pigments must be made from tyrosine and there is no way to do this is tyrosine metabolism is compromised.

Undiagnosed phenylketonuria (PKU) can lead to severe neuronal damage and mental incapacity, which is avoidable if the disease is diagnosed at birth. In contrast, albinism is easly diagnosed at birth, but there is no treatment, which leads to an increased risk of skin cancer in these patients. Choose the THREE statements that best explain the reasons for these differences.

PKU is the result of substrate accumulation, which can be avoided using a restricted diet. However, albinism is due to a missing product, which cannot be replaced through injections or dietary regimens. Defects in the enzyme tyrosinase occur in every cell of the body and there is no way to replace this enzyme without embryonic human gene therapy. In contrast, defects in the enzyme phenylalanine hydroxylase can be ameliorated by restricting certain amino acids in the diet.

Which of the following is NOT one of the three peroxisome proliferator-activated receptor nuclear receptor proteins that function in metabolic homeostasis?

PPAR-beta

A mutation in the gene encoding which of the following enzymes would affect the synthesis of both AMP and GMP?

PRPP synthetase

A mutation in the gene encoding which of the following enzymes would affect the synthesis of both AMP and GMP? adenylosuccinate synthetase PRPP synthetase IMP dehydrogenase GMP synthase

PRPP synthetase

based on the function of PYY3-36, why does it make sense to eat slowly if you are trying to reduce calorie intake

PYY3-36 signals that your small intestine has food, so by inhibiting ghrelin activation of NPY/AGRP neurons, it blocks orexigenic signaling to decrease appetite

Which of the following statements is a correct comparison of patients with type 1 versus patients with type 2 diabetes?

Patients with Type 1 diabetes are generally of normal weight whereas patients with type 2 diabetes are often obese

Which of the following statements is a correct comparison of patients with type 1 versus patients with type 2 diabetes?

Patients with Type 1 diabetes are generally of normal weight whereas patients with type 2 diabetes are often obese.

Reducing power from which of the following would help drive the nitric oxide synthase reaction?

Pentose phosphate pathway

Reducing power from which of the following would help drive the nitric oxide synthase reaction? Choose one or more

Pentose phosphate pathway

Compare the function of pepsin with that of secretin

Pepsin cleaves polypeptide bonds, whereas secretin neutralizes pH back to 7

Compare the function of pepsin with that of secretin

Pepsin cleaves polypeptide bonds, whereas secretin neutralizes pH back to 7.

Compare the function of pepsin with that of secretin.

Pepsin cleaves polypeptide bonds, whereas secretin neutralizes pH back to 7.

Compare the function of pepsin with that of secretin

Pepsin cleaves polypeptide bonds;; secretin neutralizes pH.

Match the patient symptoms on the left with the most likely biochemical basis on the right. Female 6 yrs old, adopted from Russia, blue eyes, blonde hair Guthrie test positive, urinary pterin analysis normal v Diet is normal, follows recommended nutritional guidelines Mental deficiencies, functions at the level of a 18 month old No family history is available Peculiar body odor

Phenylalanine hydroxylase deficiency

Undiagnosed phenylketonuria (PKU) can lead to severe neuronal damage and mental incapacity, which is avoidable if the disease is diagnosed at birth. In contrast, albinism is easily diagnosed at birth, but there is no treatment, which leads to an increased risk of skin cancer in these patients. Choose the statement that best explains the reasons for these differences.

Phenylalanine is found in many types of food, and therefore by avoiding foods with high amounts of phenylalanine, the build-up pyruvate metabolites can be controlled. In contrast, skin pigments must be made from tyrosine and there is no way to do this if tyrosine metabolism is compromised.

What is the probable diagnosis for Case Study 3.2 "From Russia Needs Love?"

Phenylketonuria

Which correctly describes the synthesis of phosphatidylethanolamine?

Phosphatidylserine is decarboxylated.

Which correctly describes the synthesis of phosphatidylethanolamine? Activated ethanolamine is added to a phosphatidic acid precursor. Phosphatidylserine is decarboxylated. Diacylglycerol is activated by CTP. Triacylglycerol is phosphorylated with ATP.

Phosphatidylserine is decarboxylated.

AMPK activation during short periods of intense exercise can promote ATP synthesis from glucose. Which of the following AMPK activities explains why this can occur?

Phosphorylation of glycogen synthase and phosphofructokinase-2

Which of the following is NOT involved in the feedback regulation of purine biosynthesis?

Pi

HIVRT is closely related to which DNA polymerase?

Pol I

HIVRT is closely related to which DNA polymerase? Pol I Pol II Pol III Pol IV

Pol I

Symptoms: 1. healthy male 25 yrs old has acute sharp and stabbing abdominal pain 2. sometimes anxious and fidgety for no reason 3. Sometimes the pain leads to vomiting, which can be bloody 4. no evidence of infection 5. BP of 180/120 mmHG but CT scan is negative 6. Parents immigrated from Romania

Porphobilinogen Deaminase Deficiency

What explains the observation that some forms of porphyria are associated with jaundice while others are not?

Porphyrias are genetically inherited diseases caused by defects in enzymes for heme biosynthesis. Acute intermittent porphyria is a genetic defect in porphobilinogen deaminase causing a buildup of bilirubin, which leads to yellow skin, or jaundice. Because some of the heme biosynthesis intermediates are chemically related to heme degradation products, defects in heme biosynthesis can lead to a buildup of substrates converted to bilirubin to cause jaundice. Not all porphyrias lead to an accumulation of bilirubin, therefore not all porphyrias are associated with jaundice.

What is the probable diagnosis for Case Study 1.2 "More gin, less tonic - please?"

Primaquine-induced anemia

A deficieny in the enzyme tyrosinase causes albinism and is best described as

Primarily a genes effect

Producing an iPS cell is remarkable because the pathway

Producing an iPS cell is remarkable because the pathway

When IPTG is added to a cell, what is the predicted outcome?

Protein overproduction occurs.

Which of the following enzymes is stimulated by insulin in liver cells only?

Protein phosphatase 2A

Which of the following enzymes is stimulated by insulin in liver cells only?

Protein phosphatase 2A (Insulin has similar effects on skeletal muscle and liver cells. A key difference has to do with fatty acid synthesis. Skeletal muscle does not participate in this pathway and insulin stimulates pathways involved in glycolysis and glycogen synthesis. Fatty acid synthesis does take place in the liver and insulin facilitates that pathway by activating protein phosphatase 2A)

Purines (2) & Pyrimidines (3)

Purines: A (adenine) & G (guanine) Pyrimidines: T (thymine) & C (cytosine) & U (uracil)

Nucleotide salvage pathways most commonly act upon

RNA

Nucleotide salvage pathways most commonly act upon ___

RNA

Prokaryotic transcription could terminate by

RNA with GC stem loop structures

Prokaryotic transcription could terminate by association of a mc204-1.jpg factor with the DNA. RNA with GC stem loop structures. a G-rich region on the DNA. RNA polymerase without the elongation factor.

RNA with GC stem loop structures.

Which enzyme is important in the processing of tRNA and mRNA from prokaryotes?

RNaseP

Which enzyme is important in the processing of tRNA and mRNA from prokaryotes? RNaseP snRNA reverse transcriptase ribozyme

RNaseP

Why doesn't glyphosate kill Round-Up Ready (RUR) crops? Choose the ONE best answer.

RUR crops are resistant to glyphosate because they are transgenic plants with a bacterial EPSP synthase gene, which is not inhibited by the herbicide

Consider the following scenario. Dopamine ß-hydroxylase and phenylethanolamine N-methyltransferase participate in a linear biochemical cascade. Sort the following chemicals into their appropriate bins in terms of their relevance to the reaction cascade that involves dopamine ß-hydroxylase and phenylethanolamine N-methyltransferase enzymatic activity.

Reactant: Dopamine Intermediate: Norepinephrine Product: Epinephrine Not part of: Dopaquinone

When DNA is damaged, what protein is activated?

RecA

When DNA is damaged, what protein is activated? LexA repressor RecA SOS operon lac operon

RecA

The human genome contains 11 E1 genes, 26 E2 genes and over 500 E3 genes. What does this tell you about the biochemistry of the ubiquitination process?

Recognition of distinct target proteins requires specific E3 sequences, so many E3 proteins.

Which of the following is a benefit of nucleotide salvage pathways compared with de novo synthesis?

Reduced energy expenditure

Which of the following is a benefit of the nucleotide salvage pathways compared with de novo synthesis?

Reduced energy expenditure

Which of the following is correct concerning ribonucleotide reductase?

Ribonucleotide reductase in most eukaryotes contains a dinuclear Fe3+ center in the R2 subunit

Which of the following is correct concerning ribonucleotide reductase?

Ribonucleotide reductase in most eukaryotes contains a dinuclear Fe3+ center in the R2 subunit.

What is the probable diagnosis for Case Study 1.2 "More Gin, Less Tonic - Please?"

Rickets disease

What is the probable diagnosis for Case Study 2.1 "Bah, Humbug?"

Rickets disease

Why do plant root nodules in leguminous plants contain high levels of glutamine synthetase?

Root nodules assimilate NH4+ into glutamine

Describe the biochemistry behind RoundUp Ready crops, i.e. what is a RoundUp Ready plant, why does it yield more crop per acre, and what are the potential problems with such a strategy with regard to overuse of glyphosate?

RoundUp Ready crops are resistant to the herbicide RoundUp (glyphosate) because they carry a bacterial gene encoding a variant of the enzyme ESPS synthase, a required enzyme in amino acid biosynthesis. RoundUp Ready crops can be sprayed with the herbicide and survive, but encroaching weeds will die, thus leading to higher crop yields. Some data suggest that excessive use of glyphosate could be harmful to animals and moreover, glyphosate-resistant native plants will emerge

Describe the biochemistry behind RoundUp Ready crops, i.e., what is a RoundUp Ready plant, why does it yield more crop per acre, and what are the potential problems with such a strategy with regard to overuse of glyphosate?

Roundup ready crops are resistant to the herbicide Roundup (glyphosate) because they carry a bacterial gene encoding a variant of the enzyme EPSP synthase, a required enzyme in amino acid biosynthesis. RoundUp Ready crops can be sprayed with the herbicide and survive, but encroaching weeds will die, thus leading to high crop yields. Some data suggest that excessive use of glyphosate could be harmful to animals and moreover, glyphosate-resistant native plants will emerge.

Consider a cell that is using protein as a primary carbon/energy source. Which of the following amino acids would be most relevant for this cell in terms of glycogen synthesis?

Serine

Elevated levels of free fatty acids and elevated levels of TNF-α can both result in a decrease in sensitivity to insulin. What do the mechanisms of both have in common?

Serine phosphorylation on the insulin receptor substrate protein

Insulin signaling activates glucose uptake in skeletal and adipose tissue. Place the steps of this process in the order that they would occur in both tissues.

Serum glucose concentration > 4.5 mM Activity of glucokinase in pancreatic beta cells increases Release of insulin from pancreatic intracellular vesicles Insulin binds to insulin receptor on the cell surface Translocation of GLUT4 to cell surface Increased glucose uptake by skeletal muscle and adipose tissue

More ATP per carbon is produced from oxidation of fatty acids as Which of the following offers support for the thrifty gene hypothesis compared to glucose. However, fatty acids are used primarily to provide fuel for ATP synthesis during long periods of exercise. Why can't fatty acids be used to fuel short, intense periods of exercise?

Short, intense periods of exercise do not provide sufficient time for oxidative phosphorylation to occur.

More ATP per carbon is produced from oxidation of fatty acids as compared to glucose. However, fatty acids are used primarily to provide fuel for ATP synthesis during long periods of exercise. Why can't fatty acids be used to fuel short, intense periods of exercise?

Short, intense periods of exercise do not provide sufficient time for oxidative phosphorylation to occur.

How does regulation of metabolic homeostasis by insulin and glucagon signalling differ from regulation by activation of peroxisome proliferator-activated receptor (PPAR) nuclear receptors?

Signalling mediated by activation of PPAR nuclear receptor proteins is initiated in response to different metabolic signals than insulin and glucagon signalling.

How does regulation of metabolic homeostasis by insulin and glucagon signalling differ from regulation by activation of peroxisome proliferator-activated receptor (PPAR) nuclear receptors?

Signalling mediated by activation of PPAR nuclear recpetor proteins is initiated in response to different metabolic signals than insulin and glucagon signalling.

Why is there no glucagon signaling in brain and muscle considering it is a circulating hormone?

Skeletal muscle and brain cells contain insulin receptors, but not glucagon receptors

AMPK activation increases ATP production by which three mechanisms:

Stimulating flux through glycolysis and fatty acid oxidation, as well as increasing oxidative phosphorylation.

Many popular diets exist, each with a different "eating plan" designed to promote weight loss. Common diets include the Atkins diet, Weight Watchers, the Zone diet, and the Ornish diet. Although these diets can be effective, there does not appear to be an advantage to any particular diet. In addition, many individuals do not lose weight when following these diets. What could be a reason that weight loss does not occur when someone follows one of these fad diets?

Strict adherance to the eating plan is required for significant weight loss to occur.

Many popular diets exist, each with a different "eating plan" designed to promote weight loss. Common diets include the Atkins diet, Weight Watchers, the Zone diet, and the Ornish diet. Although these diets can be effective, there does not appear to be an advantage to any particular diet. In addition, many individuals do not lose weight when following these diets. What could be a reason that weight loss does not occur when someone follows one of these fad diets?

Strict adherence to the eating plan is required for significant weight loss to occur

Explain the biochemical rationale behind the use of sulfonylureas (stimulates insulin secretin) and metaformin (inhibits gluconeogenesis for type 2 diabetes treatment

Sulfonylurea increases blood insulin which increases insulin binding to the receptor and upregulation of blood glucose uptake. Metoformin is used to lower blood glucose levels.

Treatment of type II diabetes to lower blood glucose includes the use of sulfonylureas which stimulate insulin secretion, and metformin which inhibits gluconeogenesis. Explain the rationale behind each drug.

Sulfonylurea is used on the assumption that the insulin receptor in a diabetic individual is less sensitive to insulin; therefore, increasing the insulin concentration should increase binding to the receptor and upregulate blood glucose uptake. Metformin is used on the assumption that decreased flux through gluconeogenesis lowers blood glucose level

Defects in nucleotide metabolism that lead to a buildup of uric acid cause the disease gout. Select TWO true statements below regarding potential treatments for gout

Supplying humans with the enzyme urate oxidase theoretically could be effective in reducing uric acid levels. An inhibitor of xanthine oxidase, such as allopurinol, will limit the conversion of hypoxanthine to uric acid, and could be effective in reducing uric acid levels.

Defects in nucleotide metabolism that lead to a buildup of uric acid cause the disease gout. Select the TWO true statements below regarding potential treatments for gout.

Supplying humans with the enzyme urate oxidase theoretically could be effective in reducing uric acid levels. An inhibitor of xanthine oxidase, such as allopurinol, will limit the conversion of hypoxanthine to uric acid, and could be effective in reducing uric acid levels.

Which of the following reactions or pathways is/are activated by insulin signalling in the liver but are not affected in the muscle under the same conditions?

Synthesis of glucose-6-phosphate, Synthesis of malonyl-CoA

Acetyl-CoA carboxylase (ACC) is a key regulated enzyme in fatty acid synthesis. Refer to the figure below and choose T (true) or F (false) for each statement; record your answer. 1. T F Insulin stimulates ACC activity. 2. T F Citrate inhibits ACC activity. 3. T F Phosphorylation activates ACC activity. 4. T F Glucagon inhibits ACC activity.

T, F, F, T

Why does it make metabolic sense that tadpoles have low levels of the enzyme arginase, but after their metamorphosis into frogs their arginase levels increase dramatically?

Tadpoles can excrete ammonia directly into the water, but frogs need to conserve water and excrete ammonia as urea. Arginase is essential for the production of urea from arginine.

Why does it make metabolic sense that tadpoles (which live in water) have low levels of the enyzme arginase, but after their metamorphosis into frogs (which spend time on land) their arginase levels increase dramatically?

Tadpoles can excrete ammonia directly into the water, but frogs that live on land need to conserve water and excrete ammonia as urea. Arginase is essential for the production of urea from arginine

Why does it make sense that tadpoles (which live in water) have low levels of the enzyme arginase, but after metamorphosis into frogs (which live on land), their arginase levels increase?

Tadpoles living in water can excrete ammonia directly, but once on land, they need to excrete urea.

Why does it make sense that tadpoles (which live in water) have low levels of the enzyme arginase, but after metamorphosis into frogs (which live on land), thier arginase levels increase?

Tadpoles living in water can excrete ammonia directly, but once on land, they need to excrete urea.

The human genome encodes three types of enzymes in the ubiquitinating pathway, all with multiple variant gene copies; 11 E1 variant genes, 26 E2 variant genes and over 500 E3 variant genes. What does this tell you about the biochemistry of the ubiquitination process?

That recognition of distinct target proteins requires specific E3 sequences; therefore many E3 proteins.

What are the different roles played the acyl carrier protein (ACP) and the ketoacyl synthase (KS) domain during fatty acid synthesis?

The ACP carries the growing fatty acid chain more than the KS domain.

A plausible biochemical explanation for why increased consumption of fructose could potentially lead to higher rates of obesity than would be expected if an equimolar amount of sucrose were the sweetener.

The Km of fructokinase for fructose is 20 times lower than the Km of glucokinase for glucose, so fructose is efficiently converted to fructose-1-P in liver cells and converted to acetyl-CoA, the substrate for fatty acid synthesis in the liver. In contrast, glucokinase requires more glucose to achieve the same flux to acetyl-CoA, and some glucose is converted to glucose-6-P, which can be used for glycogen synthesis (after conversion to glucose-1-P). An equimolar amount of sucrose contains 50% fructose and 50% glucose, which is 5% less fructose than that in high-fructose corn syrup.

How is penicillin inactivated by penicillin-resistant bacteria? Transpeptidase binds to methicillin. The carbonyl carbon of the b-lactam ring binds to transpeptidase. The b-lactam ring in penicillin is hydrolyzed. The serine in transpeptidase binds to penicillin.

The b-lactam ring in penicillin is hydrolyzed.

How is penicillin inactivated by penicillin-resistant bacteria?

The beta-lactam ring in penicillin is hydrolyzed.

The bioactive 5-fluorouracil is a six-atom heterocyclic ring.

The bioactive 5-fluorouracil is a six-atom heterocyclic ring.

High protein diets involve ingesting milk protein hydrolysates as a primary source of nutrients followed by aerobic exercise. What is the biochemical explanation for why rapid weight loss with such diets is mostly water rather than fatty acid oxidation?

The carbon skeleton from the aa's is used as a carbon source to supply ATP for muscle contraction during exercise rather than carbs in the diet. AA degredation results in NH4+ production and urea synthesis which requires large amounts of water for excretion. The initial weight loss is mostly water from excessive urea excretion.

Why is less ATP obtained from the average carbon in a sugar molecule than from a carbon in a fat molecule?

The carbons in fatty acids are a more reduced form of carbon than those in sugars.

Why is less ATP obtained from the average carbon in a sugar molecule than from a carbon in a fat molecule? The sugar carbons require more water weight during cellular storage. The carbons in fatty acids are a more reduced form of carbon than those in sugars. The sugar carbons yield more NADH than ATP. The sugar carbons are derived from CO2.

The carbons in fatty acids are a more reduced form of carbon than those in sugars.

Penicillin kills bacteria by inhibiting the biosynthesis of the cell wall. How does penicillin cause the inhibition? The transpeptidase of the bacteria binds to methicillin. The carbonyl carbon of the b-lactam ring of penicillin binds to transpeptidase. The b-lactam ring in penicillin is hydrolyzed. The glycine in transpeptidase binds to penicillin.

The carbonyl carbon of the b-lactam ring of penicillin binds to transpeptidase

Penicillin kills bacteria by inhibiting the biosynthesis of the cell wall. How does penicillin cause the inhibition?

The carbonyl carbon of the beta-lactam ring of penicillin binds to transpeptidase.

Penicillin kills bacteria by inhibiting the biosynthesis of the cell wall. How does penicillin cause the inhibition?

The carbonyl carbon of the mc037-1.jpg-lactam ring of penicillin binds to transpeptidase

Uric acid is derived from both purine biosynthesis and by diet. What accounts for high levels of 15N in excreted urine after feeding a patient 15N-glycine to diagnose the cause of gout? Explain

The cause of the gout is excess production of uric acid because glycine is a precursor to purines.

What would a possible outcome be if no primer could be added at the end of a linear chromosome during replication?

The chromosome would be shortened every subsequent replication.

What would a possible outcome be if no primer could be added at the end of a linear chromosome during replication? DNA synthesis would restart on the same strand. DNA synthesis would be enhanced. The chromosome would be shortened every subsequent replication. The chromosome would be elongated every subsequent replication.

The chromosome would be shortened every subsequent replication.

What enzyme is defective in Lesch-Nyhan syndrome, and why was amount of radioactive DNA in this cell culture assay about 50% of normal?

The defective enzyme is HGPRT and the about 50% level of radioactive DNA suggests this individual is heterozygous for HGPRT; that is, he or she is an asymptomatic carrier of the HGPRT defect.

Considering the physiology of thymidylate synthase, why is Tomudex an effective anticancer drug?

The drug is a noncompetitive inhibitor that binds to thymidylate synthase and decreases enzymatic activity. This causes an imbalance in the quantities of dNTPs that are available for DNA replication.

Which diet would likely be proven to work the best over 12 months for losing the most weight?

The economic diet; less money, less food, less calories

Explain how an enzyme deficiency in dihydropyrimidine dehydrogenase could lead to severe side effects in a cancer treatment regimen using 5- fluorouracil; what would be the alternative treatment?

The enzyme dihydropyrimidine dehydrogenase (DPD) modifies 5- fluorouracil (5-FU), which leads to its degradation and inactivation as a cancer drug inhibiting DNA synthesis. Because of this, high doses of 5-FU must be given to reach an effective dose in the patient (80% is degraded). However, in patients with a DPD deficiency, these high doses of 5-FU can be very toxic because most is not degraded. The alternative treatment in patients with DPD deficiency is to lower the dose of 5-FU.

Explain how an enzyme deficiency in dihydropyrimidine dehydrogenase could lead to severe side effects in a cancer treatment regimen using 5- fluorouracil; what would be the alternative treatment?

The enzyme dihydropyrimidine dehydrogenase (DPD) modifies 5- fluorouracil (5-FU), which leads to its degradation and inactivation as a cancer drug inhibiting DNA synthesis. Because of this, high doses of 5-FU must be given to reach an effective dose in the patient (80% is degraded). However, in patients with a DPD deficiency, these high doses of 5-FU can be very toxic because most is not degraded. The alternative treatment in patients with DPD deficiency is to lower the dose of 5-FU. (See pages 922-923)

Explain how an enzyme in dihydropyrimidine dehydrogenase could lead to severe side effects in a cancer treatment regimen using 5-fluorouracil; what would be the alternative treatment?

The enzyme dihydropyrimidine dehydrogenase (DPD) modifies 5-fluorouracil (5-FU) which leads to its degredation and inactivation as a cancer drug inhibiting DNA synthesis. Because of this, high doses of 5-FU must be given to reach an effective dose in the patient (80% degraded). However, in patients with a DPD deficienccy, these high doses of 5-FU can be very toxic because most is not degraded. The alternative treatment in patients with DPD deficiency is to lower the dose of 5-FU.

Explain how an enzyme deficiency in dihydropyrimidine dehydrogenase could lead to severe side effects in a cancer treatment regimen using 5-fluorouracil; what would be the alternative treatment?

The enzyme dihydropyrimidine dehydrogenase (DPD) modifies 5-fluorouracil (5-FU), which leads to dehydration and inactivation as a cancer drug inhibiting DNA synthesis. Because of this, high doses of 5-FU must be given to reach an effective dose in the patient (80% is degraded). However, in patients with a DPD deficiency, these high doses of 5-FU can be very toxic because most of it is not degraded. The alternative treatment in patients with DPD deficiency is to lower the dose of 5-FU.

Explain why patients with phenylketonuria can be spared from many of the deleterious effects of the disease, whereas there is no feasible treatment for albinism.

The level of phenylalanine in the body can be controlled by diet. There is no safe way to continually add pigments to cells through diet or drugs.

Which of the following statements is FALSE concerning leptin and leptin signaling?

The majority of obesity and diabetes cases in humans is attributed to leptin deficiencies or mutations in leptin or leptin receptor genes.

You breathe fast after short term vigorous exercise (sprint) which provides O2 for oxidative phosphorylation and ATP synthesis. What is this newly synthesized ATP used for in liver and muscle cells?

The newly synthesized ATP in the liver is used to support gluconeogenesis to convert lactate to pyruvate (Cori cycle). In muscle cells, the ATP is used to phosphorylate creatine and replenish levels of phosphocreatine.

Which of the following offers support for the thrifty gene hypothesis?

The obesity rates of genetically similar individuals are affected by diet and lifestyle.

Given that there are no known regulatory mechanisms controlling muscle protein hydrolysis, how can you explain the pattern in protein hydrolysis across 7 days?

The pattern results from the maintenance of blood glucose levels during the first days of the fast. Glucose comes initially from glycogen degradation, but once glycogen is depleted, gluconeogenesis in the liver and kidneys occurs using amino acids from protein hydrolysis as the source of carbon. After the brain adapts to using ketones as a component of its energy needs, flux through gluconeogenesis decreases, and protein is spared until survival is the highest priority.

Which of the following is consistent with the data presented in the feedback inhibition figure?

The product of the guanylate kinase reaction will inhibit the activity of PRPP synthetase

Which of the following is consistent with the data presented in the feedback inhibition figure?

The product of the guanylate kinase reaction will inhibit the activity of PRPP synthetase.

In the figure, the change in the concentrations of the three biomolecules is different, with two showing a similar pattern while the third increases rapidly as starvation conditions continue. What is the explantion for this biological phenomenon? Label the graph in Part 1 and base your answer to this question on your placement of those labels.

The rapid increase in gluconeogenic flux early in starvation leads to a corresponding increase in ketone body synthesis.

Insulin-stimulated glucose uptake and utilization by skeletal muscle and liver cells results in a reduction in the blood glucose level. Which organ is primarily responsible for the decrease in serum glucose, and why?

The skeletal muscle is responsible for the majority of glucose uptake, as the total mass of skeletal muscle is greater than the mass of liver cells.

Uracil is not found in DNA, so why does it make sense that ribonucleotide reductase converts UDP to dUDP?

The substrate for thymidylate synthase is dUMP, which is derived from dUDP.

Uracil is not found in DNA, so why does it make sense that ribonucleotide reductase converts UDP to dUMP?

The substrate for thymidylate synthase is dUMP, which is derived from dUDP.

After covalent modification of a target protein with the ubiquitin that is shown in the molecular structure, what is the most likely fate of the ubiquitin chain?

The target protein, without the polyubiquitin attachment, will be accepted into the proteasome and digested; ATP will be consumed during this process. The ubiquitin concentration in the cytoplasm will remain stable.

After covalent modification of a target protein with the ubiquitin that is shown in the molecular structure, what is the most likely fate of the ubiquitin chain? Choose one: A. The target protein, along with the polyubiquitin attachment, will be accepted into the proteasome and digested; ATP will be produced during this process. The ubiquitin concentration in the cytoplasm will decrease accordingly. B. The target protein, without the polyubiquitin attachment, will be accepted into the proteasome and digested; ATP will be consumed during this process. The ubiquitin concentration in the cytoplasm will remain stable. C. The target protein, without the polyubiquitin attachment, will be accepted into the proteasome and digested; ATP will be produced during this process. The ubiquitin concentration in the cytoplasm will remain stable. D. The target protein, along with the polyubiquitin attachment, will be accepted into the proteasome and digested; ATP will be consumed during this process. The ubiquitin concentration in the cytoplasm will decrease accordingly.

The target protein, without the polyubiquitin attachment, will be accepted into the proteasome and digested; ATP will be consumed during this process. The ubiquitin concentration in the cytoplasm will remain stable.

Describe the three mechanisms by which cancer cells can become resistant to methotrexate treatment; which of these mechanisms might be overcome by developing a methotrexate derivative using structure based drug design

The three mechanisms by which cancer cells can become resistant to drug treatment are 1) mutation in the target enzyme DHFR so it does not bind the drug but is still active, 2) amplification of the DHFR gene to overproduce the enzyme, and 3) overexpress the multidrug resistance protein to efflux the drug. Mechanism 1 (DHFR mutation) is the most likely to be overcome with a new drug design, whereas mechanisms 2 and 3 would be unaffected by a new drug.

Describe the three mechanisms by which cancer cells can become resistant to methotrexate treatment; which of these mechanisms might be overcome by developing a methotrexate derivative using structure based drug design?

The three mechanisms by which cancer cells can become resistant to drug treatment are 1) mutation in the target enzyme DHFR so it does not bind the drug but is still active, 2) amplification of the DHFR gene to overproduce the enzyme, and 3) overexpress the multidrug resistance protein to efflux the drug. Mechanism 1 (DHFR mutation) is the most likely to be overcome with a new drug design, whereas mechanisms 2 and 3 would be unaffected by a new drug.

Which of the following is correct and most accurately describes the complete urea cycle of the eukaryotic cell?

The urea cycle occurs between the mitochondrial matrix and the cytoplasm of the eukaryotic cell

Which of the following is correct and most accurately describes the complete urea cycle of the eukaryotic cell?

The urea cycle occurs between the mitochondrial matrix and the cytoplasm of the eukaryotic cell.

Which of the following is correct and most accurately describes the complete urea cycle of the eukaryotic cell? A. The urea cycle occurs between the mitochondrial matrix and the cytoplasm of the eukaryotic cell. B. The urea cycle occurs completely within the mitochondrial matrix of the eukaryotic cell. C. The urea cycle occurs completely within the cytoplasm of the eukaryotic cell. D. The urea cycle occurs between the inner mitochondrial membrane and the matrix of the eukaryotic cell.

The urea cycle occurs between the mitochondrial matrix and the cytoplasm of the eukaryotic cell.

There are eight α helices that comprise the secondary structures of thymidylate synthase, and some of these α helices comprise the active site of thymidylate synthase.

There are eight α helices that comprise the secondary structures of thymidylate synthase, and some of these α helices comprise the active site of thymidylate synthase.

For a heterozygous dominant individual carrying the Lesch-Nyhan syndrome gene, what would you recommend on the possibility their child will develop it?

There is a 50% chance a daughter would be a carrier and a 50% chance a son would have the syndrome

The figure below is a fluorescence readout from a lectin array. What can be deduced from B?

There is a glycan-lectin interaction in the well.

The figure below is a fluorescence readout from a lectin array. What can be deduced from B? There is no interaction between the glycan and the lectin in the well. There is a glycan-lectin interaction in the well. There is an interaction, but there is no way to determine the species that are interacting. Glycan cleavage from the glycoconjugate has been achieved.

There is a glycan-lectin interaction in the well.

The figure below is a fluorescence readout from a lectin array. What can be deduced from A?

There is no interaction between the glycan and the lectin in the well.

The figure below is a fluorescence readout from a lectin array. What can be deduced from A? There is no interaction between the glycan and the lectin in the well. There is a glycan-lectin interaction in the well. There is an interaction, but there is no way to determine the species that are interacting. Glycan cleavage from the glycoconjugate has been achieved.

There is no interaction between the glycan and the lectin in the well.

What is a possible outcome of the urea cycle if the cell is unable to produce enough aspartate?

There would be a buildup of citrulline

Which of the following does occurs during starvation?

There's an increase of fatty acids released from adipose tissue. There's an increase of gluconeogenesis in the liver and kidneys. Skeletal muscle proteins are degraded.

Which is true of ribozymes?

They have conserved secondary and tertiary structures.

How would individuals with decreased levels of the pentose phosphate enzyme glucose-6-phosphate dehydrogenase respond to oxidative stress?

They would not have the ability to regenerate reduced glutathione as rapidly.

How would individuals with decreased levels of the pentose phosphate enzyme glucose-6-phosphate dehydrogenase respond to oxidative stress? Higher than normal levels of NADPH would accumulate. They would not have the ability to regenerate reduced glutathione as rapidly. They would rapidly neutralize cellular levels of H2O2 and other reactive oxygen species. They would compensate with higher than normal levels of pentose phosphate pathway activity.

They would not have the ability to regenerate reduced glutathione as rapidly.

How does a defect in adenosine deaminase lead to inhibition of DNA synthesis, and why does this have an effect only on certain types of human cells?

This defect leads to an accumulation of dATP, which inhibits ribonucleotide reductase and thus deoxynucleotide biosynthesis. This is a problem for rapidly dividing cells such as B and T cells

Insulin stimulates both glucose uptake and fatty acid synthesis. With this in mind, why is it a bad idea to eat three Milky Way chocolate bars before a 60 mile bike race to get energy for this 3 hour event?

This is because the milky way bar contains too many carbohydrates and that leads to being too much glucose in the blood stream too quickly. This isn't good because having too high blood sugar levels lead to fatigue and can cause headaches.

Which of the following statements is FALSE concerning the thrifty gene hypothesis?

Thrifty genes are beneficial for the more sedentary lifestyle of modern day humans.

When amino acids from dietary proteins enter a cell, how are they able to enter the urea cycle?

Through conversion to aspartate

Which enzyme activity would be directly reduced deoxyuridine--monophosphate is present?

Thymidylate synthase

Which of the following best describes how trans- and cis-acting factors operate?

Trans-acting factors can bind to specific DNA sequences whereas cis-acting sites are DNA sequences.

which two enzymes are required for steps 2-5 of the E Coli purine biosynthetic pathway

TrifGART and FGAMS

Describe the experimental evidence for eukaryotic "purinosome" and what the result in Figure 18.10 panel F reveal about the purine biosynthetic pathway.

Two human enzymes TrifGART and FGAMS are required for steps 2-5 of the E. coli purine biosynthetic pathway. Using purine-depleted media to induce high levels of purine biosynthesis. It was observed that both of these enzymes co-localize to the same subcellular localization in human cells grown in culture. These punctuate foci were observed using fluorescence microscopy with different fluors for each enzyme (GFP and OFP) in a merged overly (panel F in Figure 18.10), providing strong support for purinosomes

In the figure below of the E. coli ribonucleotide reductase enzyme, which amino acid is important in the R2 subunit?

Tyrosine

Which amino acid is the precursor of catecholamines?

Tyrosine

Animals cannot synthesize tyrosine from shikimate-3P because they lack EPSP synthase. However, tyrosine is listed as a non-essential amino acid in most tables. What is the explanation?

Tyrosine can be synthesized from phenyalanine, and since phenylalanine is essential, tyrosine is not.

Animals cannot synthesize tyrosine from shikimate-3P because they lack EPSP synthase. However, tyrosine is listed as a non-essential amino acid in most tables. What is the explanation?

Tyrosine can be synthesized from phenylalanine, and since phenylalanine is essential, tyrosine is not.

Animals cannot synthesize tyrosine from shikimate-3P because they lack the EPSP synthase. However, tyrosine is listed as a non-essential amino acid in most tables. What is the explanation?

Tyrosine can be synthesized from phenylaline, and since phenylalanine is essential, tyrosine is not.

Which of the following mRNA codons would NOT be recognized by a tRNA that is charged with an amino acid?

UAA

In the glycogenesis pathway, what is the leaving group in the last step where glucose is added to the growing glycogen chain?

UDP

7-Dehydrocholesterol is converted to cholecalciferol (Vitamin D3) by

UV light

Which pathway does NOT provide metabolic precursors for amino acid synthesis?

Urea Cycle

Which of the following is NOT a fate of the carbon atoms from amino acid catabolism?

Urea cycle

Which pathway does NOT provide metabolic precursors for amino acid synthesis?

Urea cycle

What purine metabolite is the likely cause of gout?

Uric acid

A person who has alkaptonuria is easily diagnosed from his or her black urine. What causes the black color?

Urine of these individuals turns black when exposed to air as a result of oxidation of homogentisate.

What amino acid feed into succinyl-CoA? Citrate Cycle

Valine

Modify the schematic below by properly labeling the four neurons (W, X, Y, and Z).

W = anorexigenic; X = orexigenic; Y = POMC; Z = NPY/AGRP

Modify the schematic below by properly labeling the four neurons (W, X, Y, and Z). W = orexigenic; X = anorexigenic; Y = POMC; Z = NPY/AGRP W = anorexigenic; X = orexigenic; Y = NPY/AGRP; Z = POMC W = anorexigenic; X = orexigenic; Y = POMC; Z = NPY/AGRP W = anorexigenic; X = orexigenic; Y = NPY/AGRP; Z = POMC

W = anorexigenic; X = orexigenic; Y = POMC; Z = NPY/AGRP

How does the cell regulate the substrate specificity of ribonucleotide reductase to maintain equal amounts of all four deoxynucleoside triphosphates (dNTPs)?

When dTTP is bound to the specificity site, more dGDP is produced. When dGTP is bound to the specificity site, more dADP is produced.

Why does it make sense that citrate stimulates acetyl-CoA carboxylase activity? Only ONE answer is correct.

When the citrate cycle is inhibited by high energy charge, and glycolysis is activated by insulin signaling, then the excess citrate is transported to the cytosol where it can stimulate fatty acid synthesis.

Explain why cells with defects in thymidine kinase grow in media containing high amounts of 5-bromodeoxyuridine [BrdU], whereas cells with normal thymidine kinase will die when exposed to BrdU?

When thymidine kinase is lacking, BrdU does not become phosphorylated and hence cannot be incorporated into DNA.

Explain why cells with defects in thymidine kinase grow in media containning high amounts of 5-bromodeoxyuridine [BrdU], whereas cells with normal thymidine kinase will die when exposed to BrdU?

When thymidine kinase is lacking, BrdU does not become phosphorylated and hence cannot be incorporated into DNA.

Within the urea cycle, citrulline is produced by ornithine transcarbamylase in the mitochondria and is consumed in the cytoplasm

Within the urea cycle, citrulline is produced by ornithine transcarbamylase in the mitochondria and is consumed in the cytoplasm

The graph below shows the relative changes in concentration of glucose, fatty acids, and ketone bodies during 40 days of starvation. Choose the answer that correctly labels the data on the graph.

X = ketone bodies; Y = glucose; Z = fatty acids

There are many diseases collectively called porphyrias that are characterized by the accumulation of heme precursors in the blood and liver. Which type of porphyria is characterized by low uroporphyrinogen III synthesis?

X-linked dominant protoporphyria

Which of the following is a common breakdown product between AMP and GMP degradation to uric acid?

Xanthine

Explain the improved efficacy of 6-mercaptopurine when combined with allopurinol.

Xanthine oxidase inactivates 6-mercaptopurine by converting it to 6-thiouric acid, which lowers its effective concentration as a competitive inhibitor of adenylosuccinate synthetase and IMP dehydrogenase in leukemic cells. Therefore, by including allopurinol in the treatment, inactivation of 6-mercaptopurine by xanthine oxidase is repressed, leading to a higher effective concentration

Insulin receptor that has an increased affinity for insulin. Thrifty?

Yes, fatty acid synthesis and lipogenesis would increase fat stores

Acetyl-CoA carboxylase that is insensitive to feedback inhibition by palmitoyl-CoA. Thrifty?

Yes, malonyl-CoA would be synthesized at higher levels and thereby increase the total amount of stored triacylglycerol in adipose tissue

A hyperactive lipoprotein lipase on the plasma membrane in adipose cells. Thrifty?

Yes, scavenge as much lipid as possible from circulating lipoprotein particles to increase fat storage in adipose tissue.

Do PPARα-specific agonists theoretically offer more advantages for lowering cholesterol as compared to traditional statin drugs?

Yes, treatment with PPARα-specific agonists affects cholesterol transport and uptake as well as synthesis, whereas statins affect synthesis only.

Peroxisome proliferator-activated receptor (PPAR) nuclear receptor agonists represent an attractive class of protein targets for the development of pharmaceutical drugs for treating human metabolic disease. These drugs can be used to treat high cholesterol and type 2 diabetes. PPARα activation is known to induce expression of HDL-specific proteins, leading to an increase in HDL concentration and reverse cholesterol transport. PPARα activation also inhibits sterol response element binding protein-2 (SREBP2), a transcripton factor that activates synthesis of HMG-CoA reductase and the LDL receptor.

Yes, treatment with PPARα-specific agonists affects cholesterol transport and uptake as well as synthesis, whereas statins affect synthesis only.

Based on the premise of the thrifty gene hypothesis, predict if the following hypothetical gene variants would be candidate thrifty genes. Explain your answer. a. Insulin receptor that has an increased affinity for insulin b. High basal levels of uncoupling protein in liver cells c. A hyperactive lipoprotein lipase on the plasma membrane in adipose cells

Yes. Even at low levels both lipogenesis and fatty acid synthesis increase fat stores. b. No. Uncoupling protein overexpression in liver cells would raise basal metabolic rate by increasing thermogenesis and high rates of fatty acid oxidation. c. Yes. It would take up any circulating lipoprotein particles that it could and increase fat storage in adipose tissue.

Which of the following diets relies on high protein and low carbohydrate consumption to increase fat loss?

Zone diet

AMPK is a serine/threonine kinase that is highly conserved in eukaryotes. The AMP nucleotide associates with the regulatory subunit of the protein, and another subunit that has multiple α helices and β sheets is the catalytic subunit. In the molecular structure, the kinase inhibitor staurosporine is bound to the active site in the catalytic α subunit. What is staurosporine, shown in blue?

a competitive inhibitor that blocks the active site access to ATP and/or a target protein serine/threonine residue

A patient is found to have a mutated enzyme that results in fewer branch points in glycogen. Which one of the following would best characterize the way in which the patient's glycogen is altered relative to normal glycogen?

a decreased ratio of nonreducing to reducing ends

A patient is found to have a mutated enzyme that results in fewer branch points in glycogen. Which one of the following would best characterize the way in which the patient's glycogen is altered relative to normal glycogen? an increase in the proportion of mc061-1.jpg(1,6) linkages relative to mc061-2.jpg(1,4) linkages a change in the stereochemical configuration at the reducing end some β(1,4) linkages in place of mc061-3.jpg(1,4) linkages a decreased ratio of nonreducing to reducing ends

a decreased ratio of nonreducing to reducing ends

OB gene of mice results in obesity and symptoms of type 2 diabetes due to

a deficiency in the adipocyte hormone leptin.

To detect lectins on the surface of pathogenic and nonpathogenic bacteria, __________ would be used.

a glycan array

Resonance energy transfer from one chlorophyll in the photosystems results in

a neighboring chlorophyll in an excited state.

Resonance energy transfer from one chlorophyll in the photosystems results in a reduced neighboring pheophytin molecule. a neighboring chlorophyll in an excited state. heat release. the release of a photon.

a neighboring chlorophyll in an excited state.

a) What is the biochemical basis for gout? b) what is the most common symptom of gout?

a) Gout is caused by a buildup of uric acid owing to at least three factors b) pain in the big toe

a) What is the biochemical basis for gout; b) what is the most common symptom of gout?

a) Gout is caused by a buildup of uric acid owing to at least three factors; b) pain in the big toe.

Individuals lacking orotate phosphoribosyl transferase excrete high levels of orotic acid in their urine and develop anemia. When patients are fed uridine or cytidine, the anmeia is reduced and levels of orotic acid in the urine decline. a) identify the metabolic pathway that requires orotate phosphoribosyl transferase and write the reaction it catalyzes. b) why does feeding patients uridine or cytidine help alleviate the symptoms of anemia and high levels of orotic acid in the urine?

a) It is in the de novo pyrimidine biosynthesis pathway and the reaction it catalyzes is orotate + PRPP → OMP + PPi (b) OMP is the precursor to UTP and CTP, which are not produced when orotate phosphoribosyl transferase is defective. However, the salvage pathway can use uridine and cytidine to make UTP and CTP to provide nucleotides for the synthesis of blood cell precursors to alleviate anemia. A decrease in orotic acid excretion may occur because the fed nucleotides inhibit de novopyrimidine biosynthesis by feedback inhibition, preventing further accumulation of orotate

Kwashiorkor Syndrome is a dietary deficiency characterized by decreased pigment in the skin and hair. a) Which nutrient is likely missing from the diet? Explain your answer. b) What would you conclude if adding back this missing food group had not effect on the skin and hair condition?

a) Kwashiorkor Syndrome is due to a deficiency in the amino acid tyrosine, which would occur if the diet is lacking in sufficient protein. Tyrosine is the precursor to all major skin and hair pigments. b) If adding back protein (tyrosine) does not result in coloration of skin and hair after an extended period of time, then there is a defect in the pigment biosynthetic pathway converting tyrosine to pigments.

In a case study of Lesch-Nyhan syndrome in the 1990s, an asymptomatic individual wanted to know if they carried the genetic defect. Lots of tests were performed and showed the incorporation of radioactive hypoxanthine into DNA was about half of what would be expected in an individual with normal pyrimidine metabolism. a) what enzyme is defective in Lesch-Nyhan and why was the amount of radioactive DNA in this cell culture assay about 50%? b) what is the genetic sex of the cell donor? explain. c) why doesn't this individual show signs of Lesch-Nyhan? d) what advice would you give to this individual regarding the probability their child will develop the syndrome?

a) The defective enzyme is HGPRT and the ∼50% level of radioactive DNA suggests this individual is heterozygous for HGPRT; that is, he or she is an asymptomatic carrier of the HGPRT defect. (b) Female. Lesch-Nyhan syndrome is an X-linked recessive genetic disorder, so females can be heterozygous asymptomatic, whereas males are either normal (for a male) or symptomatic for Lesch-Nyhan syndrome. (c) A single copy of HGPRT, such as males have (with one X chromosome), is sufficient to remove excess metabolic intermediates and avoid symptoms of Lesch-Nyhan syndrome. (d) For an X-linked recessive disorder such as Lesch-Nyhan syndrome, a daughter will not develop Lesch-Nyhan syndrome, but has a 50% chance of being a carrier; a son has a 50% chance of developing Lesch-Nyhan syndrome.

The enzyme adenosine deaminase converts adenosine and deoxyadenosine into inosine and deoxyinosine. a) what is the name of the human disease characterized by an adenosine deaminase deficiency and what are the symptoms? b) how does a defect in the enzyme lead to inhibition of DNA synthesis and why does this have an effect only on certain types of human cells?

a) The disease ADA-SCID (ADA: adenosine deaminase; SCID: severe combined immunodeficiency disease) is characterized by a defective immune system, resulting from extremely low levels of infection-fighting B and T cells. (b) This defect leads to accumulation of dATP, which inhibits ribonucleotide reductase, and thus deoxynucleotide biosynthesis. This is only a problem in rapidly dividing cell types, such as immune cells that need to proliferate quickly in response to antigen-mediated stimulation (B and T cells)

Individuals with the disease phenylketonuria lack the enzyme phenylalanine hydroxylase which is required to convert phenylalanine to tyrosine. Albinism is a disease characterized by lack of skin pigment due to deficiency in the enzyme tyrosinase. a) explain why patients with phenylketonuria can be spared from many of the deleterious effects of the disease, whereas albinism has no feasible treatment. b) explain why individuals with pheynlketonuria are not complete albinos even though they cannot synthesize tyrosine from phenylalanine.

a) The level of phenylalanine in the body can be controlled by diet. There is no safe way to continually add pigments to cells through diet or drugs. (b) Individuals with phenylketonuria have tyrosinase and obtain tyrosine in their diet.

On the basis of the thrify gene hypothesis, predict if the following hypothetical gene variants would be candidate thrifty genes. a) acetyl CoA carboxylase that is insensitive to feedback inhibition by palmitoyl-CoA b) insulin receptor that has an increased affinity for insulin c) high basal levels of uncoupling protein in liver cells d) a hyperactive lipoprotein lipase on the plasma membrane in adipose cells e) a hyperactive hormone-sensitive lipase on lipid droplets in adipose cells

a) Yes, this would be a candidate because malonyl-CoA would be synthesized to higher levels than normal (no feedback inhibition) and thereby increase the total amount of stored triacylglycerol in adipose tissue. (b) Yes, this would be a candidate because even at low insulin levels, fatty acid synthesis and lipogenesis would increase fat stores. (c) No, this would not be a candidate because uncoupling protein over-expression would lead to more thermogenesis and high rates of fatty acid oxidation, raising basal metabolic rates. (d) Yes, this would be a candidate because it would scavenge as much lipid as possible from circulating lipoprotein particles to increase fat storage in adipose tissue. (e) No, this would not be a candidate because it would continually hydrolyze triacylglycerol in adipose tissue and make it difficult to accumulate stored fat.

A helix-turn-helix motif can best be described as a(n) __________ followed by a(n) __________. a-helix; b-helix a-helix;a-helix b-helix;b-helix b-helix; a-helix

a-helix;a-helix

What would the products be for the reaction below? a-keto acid and glutamate oxaloacetate and glutamate pyruvate and glutamate histidine and glutamate

a-keto acid and glutamate

Provide the following for answers for Case Study 1.1 "This Really Bites" presented in Topic 29 using the pptx slides and list of possible human disease conditions posted on the content page for Module 9: a. The probable diagnosis b. The biochemical mechanism c. Answer the Diet, DNA, or Dudes question.

a. Lesch-Nyhan disease b. There is a defect in HGPRT caused by a mutation in the DNA. c. Gene

2. (6 pts) Put the following steps in the correct order for the ribonucleotide reductase reaction mechanism. Beginning with "a" at the top of the list, and "e" at the bottom of the list, which numbered sequence in the answers given below correctly puts Steps 1-5 in order. Step 4 is provided for you

a. Step _1_ A free radical at Y122 abstracts a hydrogen atom from C439 to form a cysteine thiyl radical. b. Step _4_ Cation on C-2' is reduced by C462 creating the deoxynucleotide. c. Step _5_ Deoxynucleoside diphosphate is released and a disulfide bond is reduced by thioredoxin. d. Step _3_ The hydroxyl group on C-2' of the ribose ring gains a proton from C225 residue. e. Step _2_ A thiyl free radical removes a hydrogen atom from the C-3' position of the ribose ring.

Which of the following is a symptom of metabolic syndrome?

abdominal obesity

Approximately how many light-harvesting chromophores are found per photosystem enzyme, such as in PSI or PSII?

about 100 to 200

Nitrogen atoms that are removed from proteins during protein catabolism are least likely to be conserved in the structure of which of the following?

acetyl-CoA

Nitrogen atoms that are removed from proteins during protein catabolism are least likely to be conserved in the structure of which of the following? A. urea B. nucleotides C. biological amines D. acetyl-CoA

acetyl-CoA

Which molecule allosterically activates pyruvate carboxylase?

acetyl-CoA

What enzyme catalyzes the following reaction in the first step of fatty acid synthesis?

acetyl-CoA carboxylase

What enzyme catalyzes the following reaction in the first step of fatty acid synthesis? fatty acyl-CoA synthetase acetyl-CoA carboxylase fatty acid synthase fatty acyl-CoA dehydrogenase

acetyl-CoA carboxylase

Bromodomain-containing complexes bind to __________ lysines in histones.

acetylated

What molecule signals the endothelial cell to produce NO?

acetylcholine

What molecule signals the endothelial cell to produce NO? dopamine phosphate acetylcholine citrulline

acetylcholine

Endothelial cells can receive a chemical signal from neurons. This signal is in the form of neuronal ___________ release. This chemical messenger binds to a membrane receptor that engages a signal transduction pathway, which increases the cytoplasmic concentration of ___________ This ionic secondary messenger then activates endothelial nitric oxide synthase.

acetylcholine; calcium ions

One of the functions of simple sugars includes

acting as metabolic intermediates in energy conversion pathways.

An effect of elevated levels of TNF-alpha in serum on adipocytes

activation of NFκB

An effect of elevated levels of TNF-α in serum on adipocytes is

activation of NFκB.

Eukaryotic RNA polymerase II facilitates the

addition of 7-methylguanylate cap

The C-terminal domain (CTD) of the eukaryotic RNA polymerase II is involved in the transcriptional process of

addition of a poly(A) tail.

The C-terminal domain (CTD) of the eukaryotic RNA polymerase II is involved in the transcriptional process of addition of a poly(A) tail. removal of exons. directing RNA to the cytoplasm. product phosphorylation.

addition of a poly(A) tail.

The five-step mechanism of ribulose bisphosphate carboxylase (rubisco)

adds a CO2 molecule to a C5 ribulose sugar, yielding an unstable C6 molecule that cleaves into two C3 molecules

The five-step mechanism of ribulose bisphosphate carboxylase (rubisco) is an overall thermodynamically unstable reaction. involves C1, C3, C4, C5, and C6 molecules in the enzyme mechanism intermediates. adds a CO2 molecule to a C5 ribulose sugar, yielding an unstable C6 molecule that cleaves into two C3 molecules. involves the incorporation of 3 CO2 molecules to yield one 3-phosphoglycerate molecule.

adds a CO2 molecule to a C5 ribulose sugar, yielding an unstable C6 molecule that cleaves into two C3 molecules.

) Five nucleotide bases are found in nucleic acids. Name a purine/pyrimidine in that order.

adenine/uracil

Five nucleotide bases are found in nucleic acids. Name a purine/pyrimidine in that order.

adenine/uracil

Which answer INCORRECTLY pairs a substrate and subsequent product of dihydropyrimidine dehydrogenase?

adenine; dihydroadenine

A person is carrying out in vitro synthesis of uric acid from GMP. He or she wants to include all substrates and enzymes necessary. All EXCEPT __________ should be included in the reaction mixture.

adenosine deaminase

A person is carrying out in vitro synthesis of uric acid from GMP. He or she wants to include all substrates and enzymes necessary. All EXCEPT __________ should be included in the reaction mixture. adenosine deaminase purine nucleoside phosphorylase guanine deaminase xanthine oxidase

adenosine deaminase

The conversion of adenosine to inosine is catalyzed by __________, which is deficient in __________.

adenosine deaminase; ADA-SCID

The conversion of adenosine to inosine is catalyzed by __________, which is deficient in __________. HGPRT; Lesch-Nyhan syndrome adenosine deaminase; ADA-SCID xanthine oxidase; Lesch-Nyhan syndrome purine nucleoside phosphorylase; ADA-SCID

adenosine deaminase; ADA-SCID

Which of the nucleotides is/are synthesized from a pathway that involves inosine-5'-monophosphate?

adenosine monophosphate; guanosine monophosphate

Analysis of enzyme activities in a cell in the absence and presence of AMP and GMP is completed. Which enzyme would show lower activity when AMP is added but not when GMP is added?

adenylosuccinate synthase

leptin is a

adipocyte peptide hormone

Which of the following is secreted by visceral fat?

adipokines

Ephedrine stimulates

adrenergic receptor signaling through enhanced release of noradrenaline to increase metabolic rate.

When do somatic mutations occur?

after zygote formation

When do somatic mutations occur? G1 phase S phase G2 phase after zygote formation

after zygote formation

Which enzyme is involved in Stage 1 of IMP biosynthesis from FGAM to AIR?

air synthetase

During periods of starvation, gluconeogenesis increases in the liver. Which of the following is a major substrate for glucose biosynthesis under these conditions?

alanine

During periods of starvation, gluconeogenesis increases in the liver. Which of the following is a major substrate for glucose biosynthesis under these conditions? glutamate alanine urea fatty acids

alanine

What nitrogen source is used to produce carbamoyl phosphate?

alanine

What nitrogen source is used to produce carbamoyl phosphate? alanine urea ammonia glutamine

alanine

Which of the following is an autosomal recessive disease caused by mutations in the gene encoding tyrosinase?

albinism

A patient is having problems regulating blood pressure and has poor kidney function. Which steroid could be responsible for this?

aldosterone

A patient is having problems regulating blood pressure and has poor kidney function. Which steroid could be responsible for this? progesterone cortisol aldosterone estradiol

aldosterone

The onset of ___ is due to a recessive mutation in the gene encoding homogentisate-1,2-dioxygenase, an enzyme in the phenylalanine and tyrosine degradation pathways. A. alkaptonuria B. phenylketonuria C. both D. neither

alkaptonuria

The onset of __________ is due to a recessive mutation in the gene encoding homogentisate-1,2-dioxygenase, an enzyme in the phenylalanine and tyrosine degradation pathways.

alkaptonuria

The following pairs each match a uric acid degradation product and the animal that excretes that product. Which one is an glyoxylate - amphibians incorrect match?

allantoic acid-bird

The pentose phosphate pathway

allows 5C sugars to converge with or diverge from the glycolysis pathway

The pentose phosphate pathway

allows 5C sugars to converge with or diverge from the glycolysis pathway.

The pentose phosphate pathway resembles the TCA cycle in that it couples the loss of CO2 with the formation of NADH. allows 5C sugars to converge with or diverge from the glycolysis pathway. contains 2C, 3C, 4C, 5C, 6C, and 7C sugar molecules. enables the production of ATP from glucose.

allows 5C sugars to converge with or diverge from the glycolysis pathway.

A helix-turn-helix motif can best be described as a(n) __________ followed by a(n) __________.

alpha helix, alpha helix

The product Beano contains __________, which helps humans digest raffinose-series oligosaccharides.

alpha-galactosidase

The product Beano contains __________, which helps humans digest raffinose-series oligosaccharides. alpha-galactosidase beta-galactosidase lactase amylase

alpha-galactosidase

A helix-turn-helix motif can best be described as a(n) __________ followed by a(n) __________.

alpha-helix; alpha-helix

What would the products be for the reaction below?

alpha-keto acid and glutamate

Chlorophyll absorbs light energy efficiently because of the

alternating double and single bonds

AMPK

alters metabolic flux through energy conversion pathways to increase ATP production. This regulatory enzyme is a heterotrimeric serine/threonine kinase that is highly conserved in eukaryotes. Under conditions of high [AMP], AMP binding to the AMPK g subunit induces a conformational change that partially activates the enzyme. This conformational change facilitates phosphorylation of Thr172 on the asubunit by the upstream AMPK kinase to fully activate AMPK. Activated AMPK leads to a net increase in [ATP] by three mechanisms: 1) stimulating flux through glycolysis; 2) stimulating flux through fatty acid oxidation; 3) increased oxidative phosphorylation.

Which enzyme class best describes the activity of a urease?

amidohydrolase

Which of the following catalyzes the formation of a bond between an amino acid and a tRNA?

aminoacyl-tRNA synthetase

The process of nitrogen fixation reduces N2 to

ammonia

Upon deamination of an amino acid, which component becomes substrate for the urea cycle?

ammonia

Upon deamination of an amino acid, which component becomes substrate for the urea cycle? A. a carbon skeleton B. carbon dioxide C. a carboxylic acid D. ammonia

ammonia

The process of nitrogen fixation reduces N2 to NAD+. nitrate. ammonia. nitrite.

ammonia.

Samples from a bird, reptile, amphibian, and primate were analyzed by Western blotting for the expression of allantoinase, the enzyme that converts allantoin to allantoic acid. Unfortunately the researcher forgot the order that the samples were loaded into the gel. Luckily, only a single sample indicated allantoinase. Which sample contained allantoinase?

amphibian

The energy derived from photosynthesis and used to make ATP is manifested in what form?

an acidic thylakoid lumen

The energy derived from photosynthesis and used to make ATP is manifested in what form? an acidic stroma an electron gradient in the thylakoid membrane an acidic thylakoid lumen an acidic cytoplasm

an acidic thylakoid lumen

Under what condition is the Cori cycle most active?

anaerobic metabolism in muscle cells

The importance of V(D)J recombination is that it gives rise to new

antibodies

The importance of V(D)J recombination is that it gives rise to new viruses being formed. DNA combinations. antibodies. RNA combinations.

antibodies.

What type of antigen(s) is/are expressed on the red blood cell of a personal with type A blood?

antigen A

What type of antigen(s) is/are expressed on the red blood cell of a personal with type A blood? antigen A antigen B both antigen A and antigen B neither antigen A or antigen B

antigen A

When LexA is autocleaved by RecA*, the SOS genes

are activated

When LexA is autocleaved by RecA*, the SOS genes are repressed. are activated. are unaffected. produce operons.

are activated.

Eukaryotic promoters, but NOT prokaryotic promoters,

are located after the transcription start site.

Eukaryotic promoters, but NOT prokaryotic promoters, are located before the transcription start site. bind transcription factors. are located after the transcription start site. assist in activating transcription

are located after the transcription start site.

Nitric oxide (NO) is a potent vasodilator in humans. Synthesis of NO occurs by a two-step enzymatic reaction that is catalyzed by nitric oxide synthase. Which amino acid(s) can serve as substrate for this enzyme?

arginine

What amino acid feed into a-ketoglutarate? Citrate Cycle

arginine

Which intermediate is shared by the urea cycle and the citrate cycle?

argininosuccinate

Why do angina patients carry nitroglycerine with them?

as a rapid source of NO for blood vessel dilation

A person who has phenylketonuria must avoid which food additive?

aspartame

The atom referenced in Part 1 would have come from which of the following?

aspartate

Which amino acid provides the majority of carbons in the pyrimidine ring system?

aspartate

What role do BRCA1 and BRCA2 play in DNA repair?

assist with strand invasion

What role do BRCA1 and BRCA2 play in DNA repair? find the double-strand break assist with strand invasion repair pyrimidine dimers synthesize new DNA bases

assist with strand invasion

How many rounds of ubiquitination must a condemned protein undergo to facilitate proteasomal recognition? A. Once is sufficient B. Twice is sufficient C. At least three times is sufficient

at least three times is sufficient

To determine the role of glutaredoxin in the generation of deoxyribonucleotides by ribonucleotide reductase, which organisms would be the best to study?

bacteria

During the process of saponification, fatty acid molecules are released from triacylglycerols by treatment(s) with a strong

base and heat

During the process of saponification, fatty acid molecules are released from triacylglycerols by treatment(s) with a strong base. acid. base and heat. acid and heat.

base and heat.

What is the mechanism by which replication errors are fixed in E. coli?

base excision

Based on the function of PYY3-36 why does it make sense to eat slowly if you are trying to reduce caloric intake?

because PYY3-36 signals that your small intestine has food, so by inhibiting ghrelin activvation of NPY/AGRP neurons, it blocks orexigenic signaling to decrease appetite.

At what point during transcription does the mc203-1.jpg factor dissociate from the RNA polymerase? in the initiation phase in the termination phase before the elongation phase during the elongation phase

before the elongation phase

At what point during transcription does the sigma factor dissociate from the RNA polymerase?

before the elongation phase

Thrifty genes are __________ during times of starvation and __________ when physical activity is low.

beneficial; detrimental

Which of the following is a photosynthetic light harvesting pigment?

beta carotene

The image below shows a(n) __________ glycosidic bond.

beta-1,4-

Which enzyme is produced by penicillin-resistant bacteria?

beta-lactamase

Which enzyme is produced by penicillin-resistant bacteria? beta-lactamase peptidase transpeptidase a-galactosidase

beta-lactamase

During heme catabolism, which enzyme participates in heme breakdown while also serving an NADPH oxidase role in the reaction?

biliverdin reductase

What cofactor, common to carboxylase enzymes, is used by acetyl-CoA carboxylase?

biotin

The role of CDK in DNA synthesis is to

block further production of pre-RCs while converting pre-RC to RPC.

Uric acid is processed to allantoic acid, which is then secreted. In which of the following organisms does this occur?

bony fish

15N-labeled aspartate is provided to bacteria during the biosynthesis of pyrimidine trinucleotides. The radiolabeled nitrogen would ultimately be found in

both UTP and CTP.

What type of antibody or antibodies is/are found in the plasma of a person with type O blood?

both anti-A and anti-B

What type of antibody or antibodies is/are found in the plasma of a person with type O blood? anti-A anti-B neither anti-A or anti-B both anti-A and anti-B

both anti-A and anti-B

The phenotypes of OB and DB mice are very similar in that

both are grossly overweight and have elevated glucose and insulin due to insulin resistance.

In the glycogenesis pathway, which enzyme catalyzes the formation of alpha(1,6) linkages?

branching enzyme

If no NAD+ was available, the group 2 amino acid degradation pathway would experience a

buildup of glutamate

Which cell type in C4 plants is isolated from O2 exposure as they perform Calvin cycle reactions?

bundle sheath cells

The glyoxylate cycle enzymes

bypass the decarboxylation steps of the TCA cycle, resulting in the net synthesis of a sugar molecule from acetyl-CoA

The glyoxylate cycle enzymes are active at night when sugar reserves are high and lipid synthesis is activated. bypass the decarboxylation steps of the TCA cycle, resulting in the net synthesis of a sugar molecule from acetyl-CoA. replace the mitochondrial TCA cycle enzymes. require ATP and NADPH from the light reactions.

bypass the decarboxylation steps of the TCA cycle, resulting in the net synthesis of a sugar molecule from acetyl-CoA.

The molecule below is a precursor to the formation of which of the following second messengers?

cAMP

The molecule below is a precursor to the formation of which of the following second messengers? -Adenine RNA-

cAMP

The molecule below is a precursor to the formation of which of the following second messengers? (ATP)

cAMP

RNA is a highly dynamic biomolecule in that it

can fold and hydrogen bond with itself, DNA, proteins, or small molecules to adopt largely modified tertiary structures.

RNA is a highly dynamic biomolecule in that it can fold and hydrogen bond with itself, DNA, proteins, or small molecules to adopt largely modified tertiary structures. is less capable of forming stable H-bonds than DNA. is often shorter than DNA. is largely composed of a phosphate backbone.

can fold and hydrogen bond with itself, DNA, proteins, or small molecules to adopt largely modified tertiary structures.

Which functional group on penicillin forms a complex with transpeptidase of the bacterial wall?

carbonyl

Which functional group on penicillin forms a complex with transpeptidase of the bacterial wall? sulfide amine hydroxyl carbonyl

carbonyl

why is the rapid weight loss associated with high protein diets mostly water rather than fatty acid oxidation

carbs from amino acids are used for muscle contraction (ATP) rather than from diet, that leads to a higher demand for water and therefore initial weight loss is water from excessive urea excretion

Which molecule is used to transport fatty-acyl groups into the mitochondria?

carnitine

Which enzyme catalyzes the modification of a fatty acid, so that it can be transported into the mitochondria for oxidation?

carnitine acyltransferase I

The branching enzyme couples the formation of glycogen with UDP loss from UDP-glucose. makes a(1,4) linkages with the transfer of seven glucose units. breaks a(1,6) branch points and remakes mc058-3.jpg(1,4) linkages. catalyzes the cleavage of a(1,4) links and the formation of a(1,6) links.

catalyzes the cleavage of a(1,4) links and the formation of a(1,6) links.

The branching enzyme

catalyzes the cleavage of alpha(1,4) links and the formation of alpha(1,6) links.

The primary difference between ATP synthesis during the photosynthetic light reactions and ATP synthesis in the mitochondrial electron transport chain is the

cellular location of the proton motive force

tRNA must be __________ before binding to the ribosome to allow for translation to occur.

charged with an amino acid

tRNA must be __________ before binding to the ribosome to allow for translation to occur. charged with a codon charged with an anticodon charged with an amino acid bound by ATP

charged with an amino acid

What is a major function of prokaryotic DNA polymerase I besides replication?

checking for exonuclease activity

What is a major function of prokaryotic DNA polymerase I besides replication? very slow polymerization rate (1-2 nucleotides) to prevent mistakes DNA translation unlimited processivity checking for exonuclease activity

checking for exonuclease activity

What method is used to release the O-linked glycan groups from the membrane-associated glycoconjugates shown in 2 in the figure below? enzymatic cleavage using PNGaseF chemical cleavage using a b-elimination reaction chemical cleavage using a Schiff base intermediate enzymatic cleavage using mc040-3.jpg-galactosidase

chemical cleavage using a b-elimination reaction

What method is used to release the O-linked glycan groups from the membrane-associated glycoconjugates shown in 2 in the figure below?

chemical cleavage using a beta-elimination reaction

why is pigmentation defect not observable in children

children are often starving from birth so not observable until food is available

In what location of the cell does the Calvin cycle occur?

chloroplast stroma

In addition to triacylglycerols, which other lipid synthesized in the liver is transported by VLDLs?

cholesterol

All of the following genes are activated by the interaction of the SREBP with the SRE, EXCEPT for

cholesterol esterase.

All of the following genes are activated by the interaction of the SREBP with the SRE, EXCEPT for HMG-CoA reductase. LDL receptor. cholesterol esterase.

cholesterol esterase.

Which particles transport triacylglycerols to adipose tissue for storage?

chylomicrons

Predict the fatty acid with the lowest melting point. trans 14:1 (D7) cis 14:2 (D7,9) trans 14:2 (D7,9) cis 14:3 (D7,9,11)

cis 14:3 (D7,9,11)

Predict the fatty acid with the lowest melting point.

cis 14:3 (delta 7,9,11)

Using common nomenclature, name the following fatty acid. trans 16:2 (mc062-2.jpg11,13) cis 16:2 (mc062-3.jpg11,13) trans 16:2 (mc062-4.jpg3,5) cis 16:2 (D3,5)

cis 16:2 (D3,5)

Using common nomenclature, name the following fatty acid.

cis 16:2 (delta 3,5)

Predict the essential w-6 fatty acid. trans 18:2 (D9,12) cis 18:2 (D9,12) trans 18:2 (D6,9) cis 18:2 (D6,9)

cis 18:2 (D9,12)

Predict the essential omega-6 fatty acid.

cis 18:2 (delta 9,12)

Predict the essential w-6 fatty acid

cis 18:2 (delta 9,12)

The eve stripe 2 enhancer can be defined as multiple __________ sites for at least four __________.

cis-binding; transcription factor proteins

Which of the following combinations accurately represents the final products of the two-step nitric oxide synthase reaction?

citrulline, nitric oxide, and water

A beta-elimination reaction is used in glycan characterization to

cleave the O-linked glycans.

A mc041-1.jpg-elimination reaction is used in glycan characterization to label the antibody arrays. fluorescently label glycoproteins. cleave the O-linked glycans. cleave the N-linked glycans.

cleave the O-linked glycans.

In the duodenum, enteropeptidase

cleaves trypsinogen to form trypsin

In a leucine zipper, the leucine resides are found every seventh amino residue. This forces the supersecondary structure to be a(n)

coiled-coil

The 10 amino acids that animals need to take in through their diet are called the __________ amino acids.

common

The shikimate pathway can be defined as the pathway that involves the

condensation of phosphoenolpyruvate and erythrose-4-phosphate.

There are many diseases collectively called porphyrias that are characterized by the accumulation of heme precursors in the blood and liver. Which type of porphyria is characterized by low uroporphyrinogen III synthesis?

congenital erythropoietic porphyria

Identify the wax.

connected thru ester linkage

The gluconeogenesis pathway

converts two 3C sugars into a 6C sugar.

The gluconeogenesis pathway converts NADH into NAD+ so that glycolysis can continue. provides energy for the cell. increases the ratio of ATP to ADP in the cell. converts two 3C sugars into a 6C sugar.

converts two 3C sugars into a 6C sugar.

Which antibiotic functions by inhibiting DNA gyrase?

coumermycin

The function of an insulator sequence in DNA is to

counteract the action of enhncers

Glycoconjugates are linked to proteins or lipids via

covalent bonds

Glycoconjugates are linked to proteins or lipids via London forces. ionic bonds. covalent bonds. hydrogen bonds.

covalent bonds.

The Swi-Snf complex affects chromatin by

creating a nucleosome-free region near the promoter

The Swi-Snf complex affects chromatin by creating extra nucleosomes near the promoter. being a promoter. creating a nucleosome-free region near the promoter. transcribing the chromatin.

creating a nucleosome-free region near the promoter.

The function of DNase I is that it

cuts double-stranded DNA by cutting phosphodiester bonds.

The function of DNase I is that it cuts DNA whenever DNA binding proteins are present. cuts double-stranded DNA by cutting phosphodiester bonds. cuts DNA binding proteins. makes an RNA copy of DNA.

cuts double-stranded DNA by cutting phosphodiester bonds.

The nitrogen atoms of a purine originate from all of the following amino acids EXCEPT

cysteine

In the ER, prenylation can occur. Prenylation is the attachment of an isoprenoid group to a __________ residue via a(n) __________.

cysteine, thioester

The nitrogen atoms of a purine originate from all of the following amino acids EXCEPT glycine. aspartate. cysteine. glutamine.

cysteine.

Binding of __________ to the active site of ribonucleotide reductase inhibits all activity.

dATP

ADA-SCID is caused by mutations in the enzyme adenosine deaminase, which leads to the accumulation of deoxyadenosine (dA). Excess dA increases production of dATP and subsequent inhibition of DNA replication in developing immune cells, causing the SCID phenotype. Which combination of allosteric effectors bound to the substrate specificity site, and the regulatory site, of ribonucleotide reductase provides the BEST biochemical explanation for the ADA-SCID phenotype?

dATP bound to substrate specificity site;; dATP bound to regulatory site of ribonucleotide reductase.

beta-aminoisobutyrate is a breakdown product of

dTMP

b-aminoisobutyrate is a breakdown product of dATP. dUMP. dCMP. dTMP.

dTMP.

Contrast the binding of nucleotides to the activity site and specificity site on the R1 subunit of E. coli ribonucleotide reductase by choosing the nucleotide that can bind to the specificity site but NOT the activity site.

dTTP

Contrast the binding of nucleotides to the activity site and specificity site on the R1 subunit of E. coli ribonucleotide reductase by choosing the nucleotide that can bind to the specificity site but NOT the activity site. ATP dTTP dATP dCDP

dTTP

In this molecular structure, thymidylate synthase is shown associating with which of the following?

dUMP and raltitrexed

Identify the meabolic pathway that requires orotate phosphoribosyl transferae, and write the reaction.

de novo pyrmidine biosynthesis pathway and the reaction is: Orotate + PRPP leads to OMP +PPi

Which of the following is the most common type of DNA damage?

deamination of cytosine

Which of the following is the most common type of DNA damage? deamination of cytosine deamination of uracil methylation of adenine methylation of tyrosine

deamination of cytosine

Which enzyme catalyzes the following reaction?

debranching enzyme

Which enzyme catalyzes the following reaction? debranching enzyme glycogen phosphorylase branching enzyme glycogen synthase

debranching enzyme

In the nitrogen cycle, one main method for ammonia entering is

decomposition of organic material by invertebrates.

In the nitrogen cycle, one main method for ammonia entering is decomposition of organic material by invertebrates. from the atmosphere. through denitrification. by leghemoglobin.

decomposition of organic material by invertebrates.

Reduced levels adiponectin in obese individuals results in

decreased activation of AMPK resulting in: 1) reduced expression of the GLUT4 glucose transporter protein 2) higher acetyl-CoA carboxylase activity and malonyl-CoA, which inhibits fatty acid oxidation. Net result of low adiponectin is increased serum glucose and fatty acids is insulin resistance

PPARs have a variety of functions in lipid metabolism that include all EXCEPT

decreasing insulin sensitivity

What is the biochemical basis for Case Study 2.1 "Bah, Humbug?"

deficiency of vitamin D

What enzyme class catalyzes the following reaction? (Note that there may be some missing reactants and products.)

dehydrogenase

A possible outcome if E. coli was depleted in NADPH would be

depletion of methionine.

A possible outcome if E. coli was depleted in NADPH would be buildup of lysine. depletion of asparagine. buildup of aspartate. depletion of methionine.

depletion of methionine.

kwashiorkor

dietary deficiency characterized by distended abdomen, but also loss of pigment and hair

how were the punctate foci observed using fluorescence miscroscopy

different fluors for each enzyme (GFP and OFP) in a merged overly (panel F) provided a strong support for purinosomes

Reduction in dTMP biosynthesis is NOT dependent upon __________.

dihydrofolate

Reduction in dTMP biosynthesis is NOT dependent upon __________. methotrexate aminopterin raltitrexed dihydrofolate

dihydrofolate

Which enzyme activity would be directly inhibited by the addition of methotrexate?

dihydrofolate reductase

Carbamoyl phosphate and aspartate are substrates of aspartate transcarbamoylase. Aspartate transcarbamoylase synthesizes a product that becomes substrate for which enzyme?

dihydroorotase

The chemotherapy agent 5-fluorouracil is usually degraded by which of the following enzymes, which requires administration of higher doses?

dihydropyrimidine dehydrogenase

The chemotherapy agent 5-fluorouracil is usually degraded by which of the following enzymes, which requires administration of higher doses? mc137-1.jpg-ureidopropionase thymidine phosphorylase dihydropyrimidine dehydrogenase dihydropyrimidinase

dihydropyrimidine dehydrogenase

When trp operon is attenuated, what does that mean for the trp operon?

disruption of transcriptional elongation by RNA polymerase

Which catecholamine is made by the action of tyrosine hydroxylase and aromatic amino acid decarboxylase? (Assume that a biologically relevant molarity of all substrates for both tyrosine hydroxylase and aromatic amino acid decarboxylase are available.)

dopamine

Identify the w-3 fatty acid.

double bond 3 C's away from end methyl group

Predict the structure of a fatty acid that is trans 16:1 (delta9).

double bond starts on C6

Homologous recombination repairs what kind of DNA damage?

double strand break

To purify glutathione reductase to carry out a kinetics analysis, which organism would be the best source of the enzyme?

e coli

The eukaryotic 48S preinitiation complex is purified and the components are identified. Which of the following would be found in this complex?

eIF4

The eukaryotic 48S preinitiation complex is purified and the components are identified. Which of the following would be found in this complex? 60S ribosomal subunit Met-rRNAMet EF-Tu eIF4

eIF4

Describe the biochemistry behind RoundUp Ready crops, i.e., what is a RoundUp Ready plant, why does it yield more crop per acre, and what are the potential problems with such a strategy with regard to overuse of glyphosate?

ecause they carry a bacterial gene encoding a variant of the enzyme EPSP synthase, a required enzyme in amino acid biosynthesis. RoundUp Ready crops can be sprayed with the herbicide and survive, but encroaching weeds will die, thus leading to higher crop yields. Some data suggest that excessive use of glyphosate could be harmful to animals and moreover, glyphosate- resistant native plants will emerge

There are ___________ α helices that comprise the secondary structures of thymidylate synthase, and _______________ of these α helices comprise the active site of thymidylate synthase.

eight: some

There are ______________ α helices that comprise the secondary structures of thymidylate synthase, and _____________ of these α helices comprise the active site of thymidylate synthase.

eight; some

A nucleophilic amino group of the amino acid bound to the 3' terminus of A-site tRNA attacks the electrophilic carbonyl carbon in the ester bond between the 3' terminus of P-site tRNA and its bound amino acid during the __________ step of translation.

elongation

A nucleophilic amino group of the amino acid bound to the mc227-1.jpg terminus of A-site tRNA attacks the electrophilic carbonyl carbon in the ester bond between the mc227-2.jpg terminus of P-site tRNA and its bound amino acid during the __________ step of translation. initiation elongation translocation termination

elongation

It is important that proteolytic enzymes in the lysosome are optimized to work at low pH because low pH

enhances protein denaturing

It is important that proteolytic enzymes in the lysosome are optimized to work at low pH because low pH

enhances protein denaturing.

It is important that proteolytic enzymes in the lysosome are optimized to work at low pH because low pH enhances protein denaturing. deactivates cysteine proteases. makes it easier for ATP to be converted to ADP + Pi. enhances the degradation of ubiquitinated proteins.

enhances protein denaturing.

What class of biomolecule is glycogenin?

enzyme

Glucose-6P is critical in metabolic homeostasis, which mechanism controls metabolic flux?

enzyme levels

Which of the following best describes when a pattern of gene expression is altered without change in the DNA sequence?

epigenetic states

Which of the following groups of molecules are found in the shuffling reactions of stage 3 of photosynthesis?

erythrose-4-phosphate, glyceraldehyde-3-phosphate, and xylulose-5-phosphate

Which of the following groups of molecules are found in the shuffling reactions of stage 3 of photosynthesis? C1, C2, C3, C4, C5, C6, and C7 sugars NADPH and ATP erythrose-4-phosphate, glyceraldehyde-3-phosphate, and xylulose-5-phosphate 3-phosphoglycerate, 1,3-bisphosphoglycerate, and glyceraldehyde-3-phosphate

erythrose-4-phosphate, glyceraldehyde-3-phosphate, and xylulose-5-phosphate

The 10 amino acids that animals need to take in through their diet are called the __________ amino acids.

essential

The 10 amino acids that animals need to take in through their diet are called the __________ amino acids. nonessential essential optional degradation

essential

A purinosome, a multi-enzyme complex, is part of purine biosynthesis in which of the following?

eukaryotes only

The most significant influence on why mRNA is processed differently in prokaryotes than eukaryotes is the fact that

eukaryotes separate transcription and translation with a nucleus

Bicoid, Hunchback, Giant, and Kruppel are all transcription factor proteins involved in __________ pattern.

even-skipped expression

Bicoid, Hunchback, Giant, and Kruppel are all transcription factor proteins involved in __________ pattern. even-skipped expression odd-skipped expression even-skipped transcription odd-skipped transcription

even-skipped expression

Methenyltetrahydrofolate (MTHF) absorbs light to form *MTHF. What does the star on MTHF signify?

excited

Methenyltetrahydrofolate (MTHF) absorbs light to form *MTHF. What does the star on MTHF signify? radical excited cation anion

excited

AMPK can be activated by

exercise

AMPK can be activated by high-fat diet. exercise. fasting. PPARmc163-1.jpg.

exercise.

At any time during nuclear import and export, Ran can be found associated with all EXCEPT

exportin

A comparison of kidney and liver would show that both

express phosphoenolpyruvate carboxykinase

A comparison of kidney and liver would show that both

express phosphoenolpyruvate carboxykinase.

A comparison of kidney and liver would show that both accept blood through the portal vein. express glucokinase. express phosphoenolpyruvate carboxykinase. express adipokines.

express phosphoenolpyruvate carboxykinase.

A 7-methylguanylate cap and poly(A) tail is added to mRNA to

facilitate binding and translation by the ribosome

What enzyme catalyzes the following fatty acid synthesis reaction?

fatty acid synthase

Fully active AMPK results in an increase in all pathways listed EXCEPT

fatty acid synthesis

Carbon skeletons from proteins during protein catabolism can be readily conserved in the structure of which of the following?

fatty acids, glucose, and ketone bodies

Carbon skeletons from proteins during protein catabolism can be readily conserved in the structure of which of the following? A. fatty acids only B. fatty acids and nucleotides C. glucose only D. fatty acids, glucose, and ketone bodies

fatty acids, glucose, and ketone bodies

What is the key enzyme involved in priming fatty acids for degradation?

fatty acyl-CoA synthetase

Compared with glycogen, amylopectin contains __________ glycosidic bonds.

fewer alpha-1,6

A replication fork is necessary because it allows

for DNA replication to start on a single strand

Producing an iPS cell is remarkable because the pathway

for conversion is general, not specific.

Producing an iPS cell is remarkable because the pathway for conversion is general, not specific. is a highly irreversible reaction. for conversion is very specific and limited. produces a very unstable cell.

for conversion is general, not specific.

The debranching enzyme does NOT catalyze the

formation of alpha(1,6) linkages

Which of the following molecules is found in the nonoxidative phase of the pentose phosphate pathway?

fructose-6-phosphate

In the Krebs bicycle, which of the following citrate cycle intermediates links to the urea cycle via argininosuccinate?

fumarate

In the Krebs bicycle, which of the following citrate cycle intermediates links to the urea cycle via argininosuccinate? A. fumarate B. oxaloacetate C. malate D. succinate

fumarate

Which intermediates of the Krebs bicycle would be found in the cytosol?

fumerate and argininosuccinate

Which intermediates of the Krebs bicycle would be found in the cytosol? fumerate and argininosuccinate citrulline and malate arginine and oxaloacetate urea and ammonia

fumerate and argininosuccinate

To increase the efficiency of the nitrogenase reaction, some plants have symbionts. The plant increases the efficiency of the reaction by providing __________, whereas bacteria provide(s) __________.

fumerate and malate; additional NH3

To increase the efficiency of the nitrogenase reaction, some plants have symbionts. The plant increases the efficiency of the reaction by providing __________, whereas bacteria provide(s) __________. fumerate and malate; additional NH3 fumerate and malate; ATP ATP; amino acids NADH; ATP

fumerate and malate; additional NH3

An HIV infection begins with what first step?

fusion of the cell membrane and viral envelope

An HIV infection begins with what first step? fusion of the cell membrane and viral envelope release of viral DNA and synthesis of cDNA protein synthesis and processing budding of infectious virus

fusion of the cell membrane and viral envelope

The proteins encoded by GAL1, GAL2, GAL7, and GAL10 are all needed for converting __________ to glucose-6-phosphate.

galactose

The proteins encoded by GAL1, GAL2, GAL7, and GAL10 are all needed for converting __________ to glucose-6-phosphate. glucose fructose sucrose galactose

galactose

Which type of fatty acid plays important roles in cell recognition such as in ABO blood groups?

gangliosides

Which type of fatty acid plays important roles in cell recognition such as in ABO blood groups? sphingomyelins gangliosides cerebrosides glycerophospholipids

gangliosides

A deficiency in the enzyme tyrosinase causes albinism and is best described as

gene effect

Strong prokaryotic promoters

generally result in a higher rate of transcription.

Strong prokaryotic promoters bind tightly to the transcription factors. have mc199-1.jpg factors that are larger, more stable proteins. generally result in a higher rate of transcription. are less common in prokaryotes.

generally result in a higher rate of transcription.

Tyr122 is important in the mechanism of the E. coli ribonucleotide reductase. The function of this amino acid in the mechanism is to

generate the Cys439 radical species.

Tyr122 is important in the mechanism of the E. coli ribonucleotide reductase. The function of this amino acid in the mechanism is to carry out a nucleophilic attack on the substrate. coordinate water in the active site. generate the Cys439 radical species. hydrogen-bond to the phosphates of the substrate.

generate the Cys439 radical species.

If a mutation of the decarboxylase enzyme that functions in the biosynthetic pathway of UMP caused the pathway to halt at that step, which of the following would no longer occur?

generation of CO2

Which of the following activates triacylglycerol hydrolysis and fatty acid export in adipose tissue?

glucagon

In the breakdown of glycogen, the debranching enzyme catalyzes the formation of

glucose

Which of the following is classified as a monosaccharide?

glucose

Which of the following is classified as a monosaccharide? sucrose glucose lactose maltose

glucose

Which enzyme in the pentose phosphate pathway is regulated to control flux through the pathway?

glucose-6-phosphate dehydrogenase

A product of the first stage of purine biosynthesis is __________, whereas __________ is a by-product of the second stage

glutamate; fumarate

A product of the first stage of purine biosynthesis is __________, whereas __________ is a by-product of the second stage.

glutamate; fumarate

Consider a cell that is provided aspartate in which all carbons are radioactively labeled. Compounds in the cell are analyzed after the biosynthesis and subsequent breakdown of pyrimidines. Which of the following compounds would display no radioactivity signal during the analysis?

glutamine

Glutamine synthetase uses ammonia to covert glutamate into

glutamine

If there was a loss of efficiency in the urea cycle, there would be a buildup of

glutamine

If there was a loss of efficiency in the urea cycle, there would be a buildup of fumarate. urea. glutamine. glucose.

glutamine

If there was a loss of efficiency in the urea cycle, there would be a buildup of which amino acid?

glutamine

Which three enzymes mediate ammonia assimilation?

glutamine synthetase, glutamate synthase, glutamate dehydrogenase

Which three enzymes mediate ammonia assimilation? glutamine synthase, glutamate synthetase, glutamate hydrogenase glutamine synthetase, glutamate synthase, glutamate dehydrogenase glutamine oxidase, glutamate reductase, glutamate dehydrogenase glutamine dehydrogenase, glutamate synthase, glutamate oxidase

glutamine synthetase, glutamate synthase, glutamate dehydrogenase

Which enzyme is involved in Stage 1 of IMP biosynthesis from PRPP to 5PRA?

glutamine-PRPP amidotransferase

The numerical value of a food that indicates how quickly glucose is released into the blood is called the

glycemic index

The numerical value of a food that indicates how quickly glucose is released into the blood is called the calorie. GRP (glucose-release point). set point. glycemic index.

glycemic index.

Overall, the net balanced reaction of the Calvin cycle converts 3 CO2 into one

glyceraldehyde-3-phosphate

Overall, the net balanced reaction of the Calvin cycle converts 3 CO2 into one ribulose-5-phosphate. glyceraldehyde-3-phosphate. acetyl-CoA. glucose.

glyceraldehyde-3-phosphate.

Which of the following metabolites has carbon atoms that can end up in glucose via the gluconeogenesis pathway in humans?

glycerol

Which of the following metabolites has carbon atoms that can end up in glucose via the gluconeogenesis pathway in humans? CO2 glycerol ATP NADH

glycerol

In comparison with gangliosides, the structure of phosphatidylcholine contains a

glycerol moiety

In comparison with gangliosides, the structure of phosphatidylcholine contains a fatty acid. sugar moiety. glycerol moiety. steroid ring.

glycerol moiety.

The synthesis of dihydroxyacetone phosphate from amino acid backbones and lactate is carried out by the __________ pathway.

glyceroneogenesis

20. (6 pts) Which of the following amino acids is a starting material for the synthesis of heme?

glycine

Using the figure below, determine which of the following is NOT a final product from group 1 amino acid degradation.

glycine

Using the figure below, determine which of the following is NOT a final product from group 1 amino acid degradation. glycine pyruvate acetoacetyl-CoA acetyl-CoA

glycine

Which of the following contributes both carbon and nitrogen to the purine ring system?

glycine

Which of the following is a starting material for the synthesis of heme?

glycine

Which of the following molecules aid in the absorption of dietary lipids?

glycocholate

Which of the following molecules aid in the absorption of dietary lipids? prostaglandin cortisol glycocholate aldosterone

glycocholate

Which glycan contains a covalently linked dimeric protein that functions as a protein anchor?

glycogen

Which glycan contains a covalently linked dimeric protein that functions as a protein anchor? amylose glycogen amylopectin cellulose

glycogen

Analysis of enzyme activity in both liver and muscle cells on insulin exposure would show a decrease in the activity of

glycogen phosphorylase

Analysis of enzyme activity in both liver and muscle cells on insulin exposure would show a decrease in the activity of glycogen phosphorylase. fructose-1,6-bisphosphatase. pyruvate dehydrogenase complex. phosphofructokinase-1.

glycogen phosphorylase

The hormone insulin stimulates __________ and inhibits __________, leading to a(n) __________ in the glucose levels.

glycogen synthase; glycogen phosphorylase; decrease

The hormone insulin stimulates __________ and inhibits __________, leading to a(n) __________ in the glucose levels. glycogen synthase; glycogen phosphorylase; decrease glycogen phosphorylase; glycogen synthase; decrease glycogen synthase; glycogen phosphorylase; increase glycogen phosphorylase; glycogen synthase; increase

glycogen synthase; glycogen phosphorylase; decrease

AMP-dependent protein kinase (AMPK) is activated by low energy charge in the cell, which is the result of direct AMP binding to the gama subunit and phosphorylation of Thr172. Name 2 enzymes that are inhibited by AMPK activation and thereby lead to increased ATP synthesis and a higher energy charge:

glycogen sythetase acetyl-CoA carboxylase

Gene expression can be controlled by the modification of histones by all of the following EXCEPT

glycosylation

Gene expression can be controlled by the modification of histones by all of the following EXCEPT phosphorylation. methylation. acetylation. glycosylation.

glycosylation.

A person unable to produce new melanocytes is likely to have black hair. red hair. gray hair. hair loss.

gray hair.

A person unable to produce new melanocytes is likely to have

grey hair

Alkaptonuria is a disease coming from a deficiency in which pathway?

group 3

Which is central to the addition of the 7-methylguanosine cap to mRNA?

guanine-N7 methyltransferase

Which of the nucleotides is/are synthesized from a pathway that involves inosine-5'-monophosphate?

guanosine monophosphate adenosine monophosphate

Which of the nucleotides is/are synthesized from a pathway that involves inosine-5'-monophosphate?

guanosine monophosphate and adenosine monophosphate

Compared with a fatty acid, a wax

has long hydrocarbon chains connected with an ester linkage.

Compared with a fatty acid, a wax contains a glycerol backbone. has long hydrocarbon chains with a carboxylate group. has long hydrocarbon chains connected with an ester linkage. contains a polar phosphate group.

has long hydrocarbon chains connected with an ester linkage.

An advantage of viral DNA using LTR to produce a four-nucleotide overhang on the 3′ end is to

help with alignment of the viral DNA to the host DNA.

Hyperlipidemia

high LDL and low HDL levels

Under what conditions might the assimilation of NH4+into glutamate be favorable? Explain your answer by referring to the mass action ratio?

high levels of (NH4)2so4 will lead to a more fa vocable DeltaG because the mass action ratio will be less than one

Zone diet

high-protein, low-carb diet

A protein that has a weak affinity for a DNA site and is at ___________ concentrations of protein will bind to DNA __________.

high; strongly

A protein that has a weak affinity for a DNA site and is at ___________ concentrations of protein will bind to DNA __________. low; strongly high; strongly low; indiscriminately high; indiscriminately

high; strongly

Compared with a normal patient, a person with type 1 diabetes would display __________ during a glucose tolerance test.

higher starting blood glucose level

Raffinose-series oligosaccharides are hard for humans to digest because

humans do not have the enzyme necessary to hydrolyze the glycosidic bonds.

Raffinose-series oligosaccharides are hard for humans to digest because humans do not have the enzyme necessary to hydrolyze the glycosidic bonds. they bind to bifidobacteria in the gut. they function as soluble decoys that inhibit pathogenic bacteria from invading the epithelial cells. humans have a large number of competing glycan binding sites.

humans do not have the enzyme necessary to hydrolyze the glycosidic bonds.

The lipase-mediated release of fatty acids can always be expected to occur when a person is

hungry

The lipase-mediated release of fatty acids can always be expected to occur when a person is hungry. walking. studying. watching TV.

hungry.

Amylose can form stable left-handed helical structures because of

hydrogen bonds

What type of bond is responsible for the rigid nature of the polysaccharide fibril structure?

hydrogen bonds

What type of bond is responsible for the rigid nature of the polysaccharide fibril structure? London forces ionic bonds covalent bonds hydrogen bonds

hydrogen bonds

Amylose can form stable left-handed helical structures because of London forces. ionic bonds. covalent bonds. hydrogen bonds.

hydrogen bonds.

The gluconeogenesis reaction shown below is missing some reactants and products. What class of enzyme catalyzes this reaction?

hydrolase

The gluconeogenesis reaction shown below is missing some reactants and products. What class of enzyme catalyzes this reaction? lyase hydrolase transferase oxidoreductase

hydrolase

Spices such as eugenol are added to oils to enhance the flavor because of

hydrophobic interactions

Which is found in the ketone body pathway?

hydroxybutyrate

Which is found in the ketone body pathway?

hydroxybutyrate.

Which is found in the ketone body pathway? succinyl-CoA. propionyl-CoA. dihydroxyacetone phosphate. hydroxybutyrate.

hydroxybutyrate.

In Lesch-Nyhan syndrome patients, which enzyme is not functional?

hypoxanthine-guanine phosphoribosyltransferase

PPARα

in liver and skeletal muscle stimulates fatty acid oxidation

how could an enzyme deficiency in dihydropyrimidine dehydrogenase could lead to severe side effects in a cancer treatment regimen using 5-fluorouracil

in patients with a DPD (dihydropyrimidine dehydrogenase) deficiency, high doses of 5-FU can be toxic because most of it is not degraded

The hormone insulin activates phosphofructokinase 2 (2PFK2). This leads to a(n) __________ in concentration of fructose 2,6 bisphosphate, which favors the __________ pathway.

increase; glycolysis

Which of the following causes inhibition of fatty acid synthesis?

increased AMP-activated protein kinase (AMPK

Which of the following causes inhibition of fatty acid synthesis?

increased AMP-activated protein kinase (AMPK)

Which of the following causes inhibition of fatty acid synthesis? increased insulin increased AMP-activated protein kinase (AMPK) decreased cytoplasmic acyl-CoA increased cytoplasmic acetyl-CoA

increased AMP-activated protein kinase (AMPK)

After a meal is consumed by an individual with chronically elevated levels of free fatty acids, __________ is expected to occur.

increased activation of protein kinase C

After a meal is consumed by an individual with chronically elevated levels of free fatty acids, __________ is expected to occur. increased glucose uptake increased activation of protein kinase C increased glycogen synthesis activation of the phosphoinoside-3 kinase pathway

increased activation of protein kinase C

Under what condition(s) do the wasteful side reaction of O2 with rubisco become significant?

increased temperature and intense light

Under what condition(s) do the wasteful side reaction of O2 with rubisco become significant? higher levels of CO2 than O2 at night increased temperature and intense light increased rubisco concentrations

increased temperature and intense light

what would you conclude if replacing this food group resolved most Kwashiorkor symptoms except pigmentation, and why was this not noticed before treatment?

individual likely has defects in pigmentation pathway (not albinism).

Increased TNF-a leads to

inhibition of insulin signaling in muscle, liver, and adipose tissue - insulin resistance. Inactivates NF kappa beta. In adipocytes it inhibits adiponectin which is involved in fatty acid uptake/storage. In muscle, liver and adipose it induces serine phosphorylation ofinsulin receptor substrate

When the Trp repressor is bound to the trp operon, it results in the __________ by RNA polymerase.

inhibition of transcriptional initiation

When the Trp repressor is bound to the trp operon, it results in the __________ by RNA polymerase. inhibition of transcriptional initiation disruption of transcriptional elongation initiation enhancement of transcriptional elongation

inhibition of transcriptional initiation

TNF-alpha causes the __________ of insulin signaling in __________.

inhibition, muscle cells

Secondary downstream of the cap structure in the mRNA is unwound during the __________ step of translation.

initiation

Secondary downstream of the cap structure in the mRNA is unwound during the __________ step of translation. initiation elongation translocation termination

initiation

Process that would disrupt the regulation of metabolic homeostasis

insulin release from the pancreas, glucagon release from the pancreas, somatostatin release from the pancreas, PPAR nuclear receptor activation

What might be the outcome of a mutation that causes the adiponectin receptor to be constitutively activated?

insulin resistance

What might be the outcome of a mutation that causes the adiponectin receptor to be constitutively activated? very low levels of GLUT4 in muscle cell membranes high levels of acetyl-CoA carboxylase activity insulin resistance highly activated AMPK

insulin resistance

A steep drop in blood glucose concentration occurs after consumption of high glycemic index foods because of a(n)

insulin spike

Lysogeny by bacteriophages causes what result to the host DNA?

integration of virus DNA into host DNA

Trans activity refers to the __________ for a ribozyme.

intermolecular cleavage of substrate

Chylomicrons are produced in

intestinal epithelial cells

Both the synthesis and b-oxidation of saturated fatty acids require FAD. require NADPH. occur in the cytosol. involve acetyl-CoA.

involve acetyl-CoA.

Both the synthesis and beta-oxidation of saturated fatty acids

involve acetyl-CoA.

The beta-oxidation of mono- and polyunsaturated fatty acids

involves isomerization and in some cases reduction of the double bonds before the continuation of beta-oxidation

Which of the following is NOT a common mechanism for modifying transcription factor activity?

ionic modification

The bioactive 5-fluorouracil ____________ a six-atom heterocyclic ring.

is

PGC-1a

is a transcriptional coactivator of AMPK with multiple domains that interact with specific transcription factors and recruits histone deacetylase and RNA polymerase II activator proteins to target genes. Muscle contraction (1) stimulates the activity of AMPK, as well as p38 MAP kinase (2) and CAMK (3),which phosphorylate the transcription factors CREB and ATF2 that activate PGC-1α gene expression (4); AMPK phosphorylates and activates PGC-1α protein (5). Together, this results ATP synthesis (6) to support ongoing muscle contraction

Weight Watchers diet

is based on calorie counting to achieve short-term negative energy imbalance.

The first stage of the three-stage Calvin cycle

is catalyzed by the enzyme rubisco.

Semiconservative, as it relates to DNA replication, can be defined as when the original duplex DNA template

is separated into single strands before replications.

Dihydropyrimidine dehydrogenase deficiencies can lead to an intolerance of high doses of 5-fluorouracil because 5-fluorouracil __________ dihydropyrimidine dehydrogenase.

is usually metabolized by

Dihydropyrimidine dehydrogenase deficiencies can lead to an intolerance of high doses of 5-fluorouracil because 5-fluorouracil __________ dihydropyrimidine dehydrogenase. is usually metabolized by inhibits allosterically activates denatures

is usually metabolized by

Different tRNAs for the same amino acid that bind alternate codons are known as __________ tRNAs.

isoacceptor

Different tRNAs for the same amino acid that bind alternate codons are known as __________ tRNAs. cognate isoacceptor variable class I and class II

isoacceptor

The function of the beta-clamp is to

keep the Pol III complex associated with the DNA.

The function of the mc170-1.jpg-clamp is to keep the torsional strain reduced. add the RNA primer. keep the Pol III complex associated with the DNA. separate the DNA into a single strand.

keep the Pol III complex associated with the DNA.

What enzyme is activated by HDL to release cholesterol from the cell membranes of peripheral tissues?

lecithin-cholesterol acyltransferase

DB mice have

leptin levels that are 35 times higher than those of normal mice.

The transformation of ribose-5-phosphate to AIR in the first stage of purine biosynthesis in E. coli consumes the equivalent of 5 ATP. The second stage consumes

less than 5 ATP

The transformation of ribose-5-phosphate to AIR in the first stage of purine biosynthesis in E. coli consumes the equivalent of 5 ATP. The second stage consumes

less than 5 ATP.

Which eicosanoid acts as an inflammatory mediator that also regulates smooth muscle contraction?

leukotriene

Which eicosanoid acts as an inflammatory mediator that also regulates smooth muscle contraction? prostaglandin prostacyclin thromboxane leukotriene

leukotriene

What is a difference between group I and group II introns?

linear versus lariat intron products

What is a difference between group I and group II introns? intron cleaving versus exon ligating abilities cis versus trans cleaving capabilities linear versus lariat intron products nucleophilic versus electrophilic hydroxyl attacks

linear versus lariat intron products

The water-soluble enzyme in the small intestine that hydrolyzes the acyl ester bonds in triacylglycerols is

lipase

The outer membrane of Gram-negative bacteria contains glycolipids called

lipopolysaccharides.

The outer membrane of Gram-negative bacteria contains glycolipids called cellulose. lipopolysaccharides. lipoteichoic acid. hyaluronic acid.

lipopolysaccharides.

Analysis of various cell types after exposure to insulin would show an increase in triacylglycerol synthesis in which of the following?

liver

Analysis of various cell types after exposure to insulin would show an increase in triacylglycerol synthesis in which of the following? brain skeletal muscle liver pancreas

liver

Vitamin D3 is converted to 25-hydroxyvitamin D3 in the __________ cells.

liver

VLDLs are produced in

liver cells

VLDLs are produced in liver cells. intestinal epithelial cells. stomach cells. adipocytes.

liver cells.

Cortisol is a glucocorticoid that regulates

liver metabolism

The information gained from the DNA footprinting technique is the

location of a DNA binding protein on DNA

The information gained from the DNA footprinting technique is the DNA sequence. promotor region of a gene. location of a gene in DNA. location of a DNA binding protein on DNA.

location of a DNA binding protein on DNA.

Transcriptional elongation is favored by the trp operon when tryptophan levels are

low

Transcriptional elongation is favored by the trp operon when tryptophan levels are high. low. constant. The trp operon is not affected by tryptophan levels.

low

Predict the state of yeast when GAL 80 is bound to GAL 4 activation domain.

low galactose concentration

Predict the state of yeast when GAL 80 is bound to GAL 4 activation domain. high glucose concentration low glucose concentration high galactose concentration low galactose concentration

low galactose concentration

Atkins diet

low-carbohydrate diet.

Ornish diet

low-fat vegetarian diet with HIGH FIBER

Amino Acids are not efficiently converted to keto acids during starvation.

lower blood glucose level

Compared with a normal patient, a person with type 1 diabetes would display __________ during a glucose tolerance test.

lower starting blood glucose level

what is an alternative treatment for a cancer patient who has a DPD defficiency

lower the dose of 5-FU

When running an assay for luciferase, what reporter gene has been inserted into the plasmid?

luc

The glyoxylate cycle reaction shown below is catalyzed by which enzyme class?

lyase

A condemned protein is covalently modified by ubiquitin at ___ residues

lysine

A condemned protein is covalently modified by ubiquitin at ___________ residues.

lysine

When observing both ubiquitin polypeptides in this molecular structure that are covalently bonded together, the covalent bond that leads to polyubiquitination is a bond between which two residues?

lysine and glycine

When observing both ubiquitin polypeptides in this molecular structure that are covalently bonded together, the covalent bond that leads to polyubiquitination is a bond between which two residues? Choose one: arginine and glycine lysine and serine arginine and serine lysine and lysine arginine and arginine lysine and glycine

lysine and glycine

Other than the proteasome, there is a second organelle mechanism of protein degradation within the eukaryotic cell. Which of the following organelles is also involved in protein degradation?

lysosome

Other than the proteasome, there is a second organelle mechanism of protein degradation within the eukaryotic cell. Which of the following organelles is also involved in protein degradation? A. peroxisome B. nucleosome C. centrosome D. lysosome

lysosome

When Cro is bound to OR3, the

lytic pathway is favored

Of the following protein synthesizing RNA molecules, which codes for a protein?

mRNA

The introduction of 14CO2 into a cell actively synthesizing fatty acids results in 14C labeled

malonyl-CoA

The introduction of 14CO2 into a cell actively synthesizing fatty acids results in 14C labeled

malonyl-CoA.

The introduction of 14CO2 into a cell actively synthesizing fatty acids results in 14C labeled malonyl-CoA. acetyl-CoA. acyl-CoA. palmitate.

malonyl-CoA.

The introduction of 14CO2 into a cell actively synthesizing fatty acids results in 14C labeled

malonyl-coA

Eukaryotic transcription promoters

may bind to both transcription factors and RNA polymerases.

Eukaryotic transcription promoters are typically found after the transcription start site. have a single sequence and bind one RNA polymerase type. may bind to both transcription factors and RNA polymerases. are all controlled by a ubiquitous mc202-1.jpg factor.

may bind to both transcription factors and RNA polymerases.

Many noncoding RNAs are involved in gene silencing. Gene silencing refers to

mechanisms inhibiting gene expression.

Many noncoding RNAs are involved in gene silencing. Gene silencing refers to the splicing of mRNA into alternative transcripts. defense mechanisms against RNA viruses. mechanisms inhibiting gene expression. the modification of genes through RNA-mediated mutation.

mechanisms inhibiting gene expression.

Which of the following is NOT a cellular role for nucleotides?

membrane formation

Which of the following is a coding RNA molecule?

messenger RNA

Which of the following is a coding RNA molecule? short interfering RNA. TERC RNA. small nucleolar RNA. messenger RNA.

messenger RNA.

A compound is added to a cell and dihydrofolate reductase activity is reduced. If this compound is __________, it can be predicted that a similar result would be seen if __________ were added instead.

methotrexate; aminopterin

MGMT is called the suicide enzyme because its active site permanently becomes

methylated

What DNA damage can the MGMT enzyme repair?

methylated guanine

A common RNA base modification is

methylation

Which RNA molecule is expressed in the genome to regulate gene expression?

miRNA

Which class of RNA molecule is typically the shortest in length?

micro RNA

Which class of RNA molecule is typically the shortest in length? transfer RNA messenger RNA small nuclear RNA micro RNA

micro RNA

What is NOT a likely repair system used if a strand of DNA has a pyrimidine dimer?

mismatch repair

What is NOT a likely repair system used if a strand of DNA has a pyrimidine dimer? mismatch repair direct repair pathway base excision nucleotide excision

mismatch repair

Within the urea cycle, citrulline is produced by ornithine transcarbamylase in the ???? and is consumed in the ????

mitochondria cytoplasm

The acetyl-CoA for fatty acid synthesis comes from the

mitochondria via the citrate shuttle

Within the urea cycle, citrulline is produced by ornithine transcarbamylase in the ______ and is consumed in the ________.

mitochondria, cytoplasm

Within the urea cycle, citrulline is produced by ornithine transcarbamylase in the _______________ and is consumed in the _____________________

mitochondria; cytoplasm

In a eukaryotic cell, deamination of amino acids takes place most extensively in the

mitochondrial Matrix

In a eukaryotic cell, deamination of amino acids takes place most extensively in the

mitochondrial matrix

In a eukaryotic cell, deamination of amino acids takes place most extensively in the Choose one: A. cytoplasm. B. nuceloplasm. C. mitochondrial intermembrane space. D. mitochondrial matrix.

mitochondrial matrix

The photosynthetic Z scheme describes the

movement of electrons driven by the absorption of light energy.

The AMP nucleotide in AMP-dependent kinase (AMPK) is shown associating with a subunit that is composed of which of the following?

multiple α helices and four β sheets that are shown in green

Consider an in vitro experiment in which all components needed for protein synthesis are present. If tRNAs charged with radioactively labeled amino acids are added, over time the radioactivity would be located in which of the following components?

nascent protein

The figure below shows __________ autoregulation and will __________.

negative; reach a steady state

When the lysine side chain in a histone is acetylated, the amino group is now

neutral

The function of Dam methylase in DNA synthesis is to methylate the

new DNA strand

The function of Dam methylase in DNA synthesis is to methylate the new DNA strand. old DNA strand. RNA primer. primase.

new DNA strand.

Agricultural fertilizers provide the ground with nitrate and nitrite, but only ammonia can be assimilated by plants. What process converts nitrate and nitrite to ammonia?

nitrification

Agricultural fertilizers provide the ground with nitrate and nitrite, but only ammonia can be assimilated by plants. What process converts nitrate and nitrite to ammonia? nitrification ammonia assimilation nitrogen fixation nitrogen assimilation

nitrification

During IMP biosynthesis, fumarate is released as a byproduct. Fumarate is cleaved from a(n) _________ atom on the developing heterocyclic ring of the purine.

nitrogen

During IMP biosynthesis, fumarate is released as a byproduct. Fumarate is cleaved from a(n) ______________ atom on the developing heterocyclic ring of the purine.

nitrogen

Nitrogen in biological compounds ultimately comes from what source?

nitrogen gas

Nitrogen balance is best described as when the daily intake of __________ equals the amount of __________ lost by excretion.

nitrogen; nitrogen

Nitrogen balance is best described as when the daily intake of __________ equals the amount of __________ lost by excretion. ammonia; nitrogen nitrogen; nitrogen glucose; glucose nitrogen; ammonia

nitrogen; nitrogen

What are the four key enzymes in nitrogen fixation and assimilation in plants and bacteria?

nitrogenase complex, glutamine synthetase, glutamate synthase, glutamate dehydrogenase

Explain why glyphosate works faster in the summer after rain than after two weeks without rain

no correct answer

If plants and bacteria were unable to produce glutamate from ammonia, a possible outcome would be that plants would

no longer be able to produce other amino acids

would high basal levels of uncoupling protein in liver cells be a thrifty gene

no, because uncoupling protein expression would lead to more thermogenesis and high rates of fatty acid oxidation, which would raise basal metabolic rates

A person who has alkaptonuria is easily diagnosed from his or her black urine. What causes the black color?

none of these answers are correct

Order the following steps involving the regeneration of ribonucleotide reductase that occurs in most animals so that it may carry out the formation of deoxyribonucleotides. (Note that not all steps are shown.) 1. Reduction of thioredoxin 2. Reduction of ribonucleotide reductase 3. Oxidation of thioredoxin reductase 4. Reduction of thioredoxin reductase

none of these answers are correct

Which of the following is a starting material for the synthesis of heme?

none of these answers are correct: -Histidine -Alanine -Alanine -Phenylalanine

A person who has alkaptonuria is easily diagnosed from his or her black urine. What causes the black color?

none of these answers are correct: -oxidation of phenylalanine -dehydration -reduction of acetoacetyl-CoA -reduction of homogentisate

Identify the location of action of the spliceosome.

nucleus

Identify the location of action of the spliceosome. nuclear pore nucleus rough endoplasmic reticulum cytoplasm

nucleus

Leptin is an adipocyte hormone that send signals to the brain to "eat less" and "metabolize more." What is the explanation for why leptin injections cause weight loss in the strain of ob/ob mutant mice, but leptin injections has no effect in the majority of people?

ob/ob mutant mice lack leptin, and by providing it through injection, they are able to better maintain energy balance and metabolic homeostasis. In contrast, obese humans have chronically high leptin levels and are in fact leptin-insensitive

Leptin is an adipocyte hormone that send signals to the brain to "eat less" and "metabolize more." What is the explanation for why leptin injections cause weight loss in the strain of ob/ob mutant mice, but leptin injections has no effect in the majority of people?

ob/ob mutant mice lack leptin, and by providing it through injection, they are able to better maintain energy balance and metabolic homeostasis. In contrast, obese humans have chronically high leptin levels and are in fact leptin-insensitive.

The conversion of pyruvate to phosphoenolpyruvate (PEP) in gluconeogenesis

occurs in two steps and is catalyzed by the enzymes pyruvate carboxylase and phosphoenolpyruvate carboxykinase

Together, the eight histone molecules are called the histone

octamer

Together, the eight histone molecules are called the histone octane. octamer. dimer. tetramer.

octamer.

What is the composition of the eukaryotic proteasome?

one 20S core particle and two 19S regulatory particles

Compared with amylopectin, amylose has __________ glycosidic bonds.

only alpha-1,4

Arginine, leucine, and lysine are all essential amino acids. This means that they are

only available from a person's diet.

Metabolic homeostasis relies on maintaining

optimal metabolite concentrations.

When basal metabolic rates are decreased and appetite is increased, which type of neurons are likely activated?

orexigenic

Which of the following neurons is a second-order neuron and therefore does NOT respond directly to leptin?

orexigenic

Which of the following neurons is a second-order neuron and therefore does NOT respond directly to leptin? POMC orexigenic NPY AGRP

orexigenic

Ghrelin stimulates

orexigenic (increased appetite) neuronal signaling bybinding to ghrelin receptors on NPY/POM neurons. Also releases PYY3-36 peptide which binds to Y2 receptors on NPY/AGRP neurons and inhibits orexigenic signaling

The biosynthesis of triacylglycerols in animals uses which citrate cycle intermediate in the liver?

oxaloacetate

The synthesis of tetrahydrofolate is chemically coupled to which of the following

oxidation of NADPH

The synthesis of tetrahydrofolate is chemically coupled to which of the following?

oxidation of NADPH to NADP+

A person who has alkaptonuria is easily diagnosed from his or her black urine. What causes the black color?

oxidation of homogentisate

A person who has alkaptonuria is easily diagnosed from his or her black urine. What causes the black color? oxidation of phenylalanine oxidation of homogentisate reduction of acetoacetyl-CoA dehydration

oxidation of homogentisate

What class of enzyme is the human desaturase enzyme?

oxidoreductase

What class of enzyme is the human desaturase enzyme? isomerase oxidoreductase ATPase transferase

oxidoreductase

What triggers the Cori cycle?

oxygen-limited working muscle cells

A protein is targeted to the plasma membrane. Its final functional location is as a subunit of a transmembrane protein that interacts with the central subdomain of the membrane. Analysis of this protein would most likely show modification with a

palmitoylate

A protein is targeted to the plasma membrane. Its final functional location is as a subunit of a transmembrane protein that interacts with the central subdomain of the membrane. Analysis of this protein would most likely show modification with a myristoylate. palmitoylate. isoprenoid. phosphate.

palmitoylate.

Ubiquitin does not contain which of the following?

parallel β sheets

Ubiquitin does not contain which of the following? Choose one or more: A. α helices B. turns that connect one secondary structure to another C. antiparallel β sheets D. parallel β sheets

parallel β sheets

What is the name for the relationship between the original DNA and replicate DNA?

parent and daughter

What is the name for the relationship between the original DNA and replicate DNA? father and son parent and children parent and daughter mother and daughter

parent and daughter

Which of the following statements is a correct comparison of patients with type 1 versus patients with type 2 diabetes?

patients with Type 1 diabetes are generally of normal weight whereas patients with type 2 diabetes are treated with insulin

The NADPH required by the fatty acid synthesis pathway comes from the

pentose phosphate pathway and the shuttle of citrate from the mitochondria.

The NADPH required by the fatty acid synthesis pathway comes from the pentose phosphate pathway and the shuttle of citrate from the mitochondria. mitochondria. breakdown of glucose. TCA cycle and the shuttle of citrate from the mitochondria.

pentose phosphate pathway and the shuttle of citrate from the mitochondria.

If the ratio of NADP+ to NADPH were high, the

pentose phosphate pathway oxidative phase would be activated.

A person who has phenylketonuria will have a buildup of which molecule in the cell?

phenylalanine

A person who has phenylketonuria will have a buildup of which molecule in the cell? tyrosine phenylalanine pyruvate alanine

phenylalanine

What is the probable diagnosis for Case Study 3.2 "From Russia Needs Love?"

phenylketonuria

The outer layer of the plasma membrane of human erythrocytes contains mostly __________ lipids

phosphatidylcholine

The outer layer of the plasma membrane of human erythrocytes contains mostly __________ lipids.

phosphatidylcholine

The outer layer of the plasma membrane of human erythrocytes contains mostly __________ lipids. phosphatidylinositol phosphatidylserine phosphatidylcholine phosphatidylethanolamine

phosphatidylcholine

The inner layer of the plasma membrane of human erythrocytes contains mostly __________ lipids.

phosphatidylserine

Thiazolidinedione activates PPAR-gamma in adipose tissue. This results in the upregulation of

phosphoenolpyruvate carboxykinase.

Thiazolidinedione activates PPARmc160-1.jpg in adipose tissue. This results in the upregulation of TNF-mc160-2.jpg. plasminogen activator inhibitor 1. interleukin-6. phosphoenolpyruvate carboxykinase.

phosphoenolpyruvate carboxykinase.

Which of the following would show increased activity levels in a liver cell after exposure to insulin?

phosphofructokinase-1

Analysis of phosphorylated proteins in a cell with a fully active AMPK would reveal that __________ is phosphorylated.

phosphofructokinase-2

Analysis of phosphorylated proteins in a cell with a fully active AMPK would reveal that __________ is phosphorylated. cytochrome oxidase cytochrome c phosphofructokinase-2 PPARa

phosphofructokinase-2

Chylomicrons contain a monolayer on the outside of the particle consisting of

phospholipids and cholesterol

Chylomicrons contain a monolayer on the outside of the particle consisting of

phospholipids and cholesterol.

Chylomicrons contain a monolayer on the outside of the particle consisting of cerebrosides and phospholipids. cerebrosides and sphingolipids. phospholipids and cholesterol. cholesterol and sphingolipids.

phospholipids and cholesterol.

The conversion of ribose-1-phosphate to ATP via the salvage pathway would require the activity of several enzymes. Choose the answer that lists them in the order that they would act in this pathway.

phosphopentomutase; PRPP synthetase; phosphoriboxyl transferase; kinase; kinase

The conversion of ribose-1-phosphate to ATP via the salvage pathway would require the activity of several enzymes. Choose the answer that lists them in the order that they would act in this pathway. PRPP synthetase; phosphopentomutase; kinase; kinase; phosphoriboxyl transferase PRPP synthetase; phosphoriboxyl transferase; phosphopentomutase; kinase; kinase phosphopentomutase; PRPP synthetase; phosphoriboxyl transferase; kinase; kinase phosphopentomutase; phosphoriboxyl transferase; kinase; PRPP synthetase; kinase

phosphopentomutase; PRPP synthetase; phosphoriboxyl transferase; kinase; kinase

What specific characteristic must a target protein have to be recognized by an ubiquitinating protein?

phosphorylated residue

What specific characteristic must a target protein have to be recognized by an ubiquitinating protein? phosphorylated residue N terminus C terminus mc112-1.jpg-helix

phosphorylated residue

What cellular condition(s) would cause the upregulation of AMPK activity?

phosphorylation of Thr172 in the α subunit low energy charge in muscle cells

Elevated serum levels of free fatty acids prevents the

phosphorylation of tyrosine in insulin receptor substrate 1 protein

Identify the w-3 fatty acid.

pic in phone

Identify the wax

pic in phone

The glycan fragment shown below on the right has been cleaved by glycosylase enzymes and HPLC analysis has produced the peaks 1 to 6. Which of the following fragments corresponds to peak 1?

pic in phone

Which of the following is an electron transfer molecule in photosynthesis light reactions?

plastoquinone

Which of the following is an electron transfer molecule in photosynthesis light reactions? plastoquinone photon paraquat ATP

plastoquinone

What characteristics of eukaryotic mRNA are recognized through specific interactions during the formation of the translation initiation complex?

poly(A) tail

What characteristics of eukaryotic mRNA are recognized through specific interactions during the formation of the translation initiation complex? 3' Shine-Dalgarno sequence poly(A) tail GTP-binding site

poly(A) tail

Heme and chlorophyll have similar overall structures. What is common between the two molecules?

polycyclic planar structure

We want _ fats

polyunsaturated

Diseases affecting heme biosynthesis as a result of deficiencies in the heme biosynthetic pathway are called

porphyrias

Diseases affecting heme biosynthesis as a result of deficiencies in the heme biosynthetic pathway are called porphyrias. albinism. shikimate. alkaptonuria.

porphyrias.

What kind of control mechanism is ligand-induced binding of an activator protein?

positive

What kind of control mechanism is ligand-induced binding of an activator protein? positive negative neutral covalent

positive

Pluripotent state can be defined as a __________ cell.

predifferentiated

In the glycogenesis pathway, the role of the glucose-6-phosphate isomerization reaction is to

prepare the anomeric carbon for nucleophilic attack.

AZT is such a good HIV drug because it

prevents new DNA from being replicated.

AZT is such a good HIV drug because it prevents new DNA from being replicated. prevents HIV from binding to the cell. allows DNA to repair itself after viral DNA is added. allows DNA to be replicated at a slower rate.

prevents new DNA from being replicated.

High concentration of CI protein in the cell __________ expression of the __________.

prevents; lytic promoter

High concentration of CI protein in the cell __________ expression of the __________. prevents; lytic promoter allows; lytic promoter prevents; trp operon allows; attenuator sequence

prevents; lytic promoter

All of the following are true of both chloroplasts and mitochondria EXCEPT that they

produce NADPH inside the organelle

All of the following are true of both chloroplasts and mitochondria EXCEPT that they have one highly folded membrane. produce ATP inside the organelle. produce NADPH inside the organelle. are present in photosynthetic plant cells

produce NADPH inside the organelle.

Which steroid hormone is synthesized in the corpus luteum?

progesterone

Which of the following amino acids is part of the Group 2 amino acid degradation pathway?

proline

Which of the following amino acids is part of the Group 2 amino acid degradation pathway? A. glycine B. phenylalanine C. cysteine D. proline

proline

Which eicosanoid modulates the secretion of proteoglycans that protects the stomach lining from the effects of low pH?

prostaglandin

What food group is missing from kwashiorkor (distended abdomen and loss of hair pigmentation)

protein deficiency, decrease intact of tyrosine and conversion of dietary phenylalanine to tyrosine (reduced pigmentation)

Which of the following changes in metabolic flux would be expected to occur during long periods without food?

protein degradation in skeletal muscle

The antibiotics streptomycin, tetracycline, and chloramphenicol all interfere with

protein synthesis

What two nucleic acid bases are represented by the structures (1) and (2) shown below?

purines have two rings, pyrimidines have 1

Of the four molecules listed below, three of them reciprocally regulate phosphofructokinase 1 (PFK1) and fructose-1,6-bisphosphatase. Which molecule does NOT fit?

pyruvate

Of the four molecules listed below, three of them reciprocally regulate phosphofructokinase 1 (PFK1) and fructose-1,6-bisphosphatase. Which molecule does NOT fit? pyruvate fructose-2,6-bisphosphate citrate AMP

pyruvate

Which enzyme in the glycolysis pathway catalyzes a reaction that requires two enzymes to reverse in the gluconeogenesis pathway?

pyruvate kinase

The growth rate of nonruminating animals often decreases when they eat feed containing __________ because of the inability to digest it.

raffinose

The growth rate of nonruminating animals often decreases when they eat feed containing __________ because of the inability to digest it. glucose lactose starch raffinose

raffinose

A function of transcriptional activator proteins is to

recruit chromatin modifiers

Which of the following is a benefit of nucleotide salvage pathways compared with de novo synthesis?

reduced energy expenditure

What extra enzyme types are often required in the degradation of polyunsaturated fats by the beta-oxidation pathway?

reductase and isomerase

The third stage of the three-stage Calvin cycle

regenerates a C5 sugar for the Calvin cycle to continue

The third stage of the three-stage Calvin cycle uses ATP and NADPH from the light reactions. generates a glucose molecule for the plant. is similar to the reactions found in the glycolysis pathway. regenerates a C5 sugar for the Calvin cycle to continue.

regenerates a C5 sugar for the Calvin cycle to continue.

The Cori cycle

relies on functioning lactate dehydrogenase in both muscle and liver tissues.

The Cori cycle transports pyruvate from working muscles to the liver, where it is converted back to glucose. is important in organisms respiring aerobically. involves the conversion of muscle pyruvate to alanine followed by transport to the liver. relies on functioning lactate dehydrogenase in both muscle and liver tissues.

relies on functioning lactate dehydrogenase in both muscle and liver tissues.

Which of the following best describes base excision?

removal and replacement of individual bases that have been damaged by various chemical reactions

Which of the following best describes base excision? removal and replacement of individual bases that have been damaged by various chemical reactions repair of large lesions that distort DNA removal of individual bases that have been phosphorylated repair of small lesions that distort DNA

removal and replacement of individual bases that have been damaged by various chemical reactions

The urea cycle's function is to __________ the body of an organism.

remove excess nitrogen from

The complete complex that contains the enzymes and proteins required to replicate DNA is called the

replisome

Group I and group II introns are similar in their

requirement for metal ion cofactors.

Group I and group II introns are similar in their requirement for metal ion cofactors. requirement for exogenous nucleoside binding. DNA composition. intron lariat structure.

requirement for metal ion cofactors.

The synthesis of palmitic acid

requires ATP.

The initiation of transcription in eukaryotes

requires many more transcription factors than prokaryotic transcription.

The initiation of transcription in eukaryotes occurs in the same manner as prokaryotic transcription. requires the complete unfolding of the gene into single-stranded DNA. requires many more transcription factors than prokaryotic transcription. uses RNA polymerase as well as helicase and primase

requires many more transcription factors than prokaryotic transcription.

The following picture is an example of what phenomenon?

resonance energy transfer

What is the function of telomerase in termination of DNA synthesis?

reverse transcription of the telomeres

The biosynthesis of deoxynucleosides is important, yet different organisms require different sets of enzymes to carry it out. If 100 different species were analyzed, which enzyme is most likely to be found in all of them?

ribonucleotide reductase

The RNA type directly involved in protein synthesis is

ribosomal RNA

The RNA type directly involved in protein synthesis is small nuclear RNA. short interfering RNA. ribosomal RNA. long nc RNA.

ribosomal RNA.

What is the name of the enzyme that catalyzes the reaction shown below?

ribulose bisphosphate carboxylase

What is the name of the enzyme that catalyzes the reaction shown below? ribulose bisphosphate carboxylase acetyl-CoA carboxylase glyceraldehyde-3-phosphate dehydrogenase pyruvate carboxylase

ribulose bisphosphate carboxylase

Which is the substrate or product of the reactions that comprise the oxidative branch of the pentose phosphate pathway?

ribulose-5-phosphate

The lipase-mediated release of fatty acids can always be expected to occur when a person is

running from a bear.

Cis-acting enzymes are

self-operating enzymes

Cis-acting enzymes are self-operating enzymes. enzymes that operate on a target molecule. intermolecular-operating enzymes. enzymes that operate on other enzymes.

self-operating enzymes.

Consider a cell that is using protein as a primary carbon/energy source. Which of the following amino acids would be most relevant for this cell in terms of glycogen synthesis?

serine

Consider a cell that is using protein as a primary carbon/energy source. Which of the following amino acids would be most relevant for this cell in terms of glycogen synthesis? A. leucine B. serine C. lysine

serine

Which amino acid does penicillin form a complex with on transpeptidase?

serine

Which of the following amino alcohols act as the functional group attached to a phosphate in a glycerophospholipid?

serine

Which of the following amino alcohols act as the functional group attached to a phosphate in a glycerophospholipid? lysine serine glycine alanine

serine

Which type of RNA facilitates RNA interference by resulting in degraded mRNA?

siRNA

Which type of RNA facilitates RNA interference by resulting in degraded mRNA? snoRNA siRNA miRNA rRNA

siRNA

PPARδ

signaling in adipose tissue and skeletal muscle increases rates of fatty acid oxidation and thermogenesis

Which class of RNA molecules is unique to eukaryotic organisms?

small nuclear RNA

Which class of RNA molecules is unique to eukaryotic organisms? transfer RNA ribosomal RNA small nuclear RNA messenger RNA

small nuclear RNA

Membrane phospholipids are synthesized in which cellular location?

smooth ER

Membrane phospholipids are synthesized in which cellular location?

smooth endoplasmic reticulum

Membrane phospholipids are synthesized in which cellular location? cytoplasm mitochondrial intermembrane space mitochondrial matrix smooth endoplasmic reticulum

smooth endoplasmic reticulum

Which RNA molecule does NOT bind to the ribosome?

snRNA

Which RNA molecule does NOT bind to the ribosome? tRNA rRNA mRNA snRNA

snRNA

What are the two main types of stem cells?

somatic and embryonic

What are the two main types of stem cells? somatic and embryonic somatic and epithelial stromal and somatic stromal and embryonic

somatic and embryonic

Inhibition of which of the following processes would disrupt the regulation of metabolic homeostasis?

somatostatin release from the pancreas, PPAR nuclear receptor activation, glucagon release from the pancreas, insulin release from the pancreas

Compared with other areas of the membrane, lipid rafts contain more

sphingolipids

The addition of phosphocholine to ceramides generates

sphingomyelin

A negative nitrogen balance in a person would be an indicator of

starvation

PPARγ

stimulates lipid synthesis and improves insulin sensitivity in liver and adipose tissue

Glucagon-like peptide-1 (GLP-1) is a hormone peptide secreted by intestinal L cells. It is secreted in response to the nutrient detection in the small intestine. The resulting physiological response is the lowering of blood glucose levels. Which of the following is a potential mechanism for how GLP-1 causes this physiological response? stimulation of glucagon release stimulation of insulin release by pancreatic b cells increase in the apoptosis rate of pancreatic b cells downregulation of GLUT4

stimulation of insulin release by pancreatic b cells

Glucagon-like peptide-1 (GLP-1) is a hormone peptide secreted by intestinal L cells. It is secreted in response to the nutrient detection in the small intestine. The resulting physiological response is the lowering of blood glucose levels. Which of the following is a potential mechanism for how GLP-1 causes this physiological response?

stimulation of insulin release by pancreatic beta cells

Glucagon-like peptide-1 (GLP-1) is a hormone peptide secreted by intestinal L cells. It is secreted in response to the nutrient detection in the small intestine. The resulting physiological response is the lowering of blood glucose levels. Which of the following is a potential mechanism for how GLP-1 causes this physiological response?

stimulation of insulin release by pancreatic cells

What prokaryotic promoter would most likely control a housekeeping gene?

strong promoter

What prokaryotic promoter would most likely control a housekeeping gene? weak promoter -10 and -35 box TATA box strong promoter

strong promoter

What molecule is the odd chain fatty acid product propionyl-CoA converted into?

succinyl-CoA

What molecule is the odd chain fatty acid product propionyl-CoA converted into? butyryl-CoA succinyl-CoA citrate palmitate

succinyl-CoA

The main function of the primosome is to

synthesize primers for lagging strand synthesis.

What enzymatic activity is NOT required during the biosynthesis of UMP from carbamoyl phosphate and aspartate?

synthetase

What enzymatic activity is NOT required during the biosynthesis of UMP from carbamoyl phosphate and aspartate? dehydrogenase decarboxylase phosphoribosyl transferase synthetase

synthetase

The adaptor molecule in translation is

tRNA

Which molecule contains both an amino acid acceptor stem and an anticodon?

tRNA

Which molecule contains both an amino acid acceptor stem and an anticodon? tRNA mRNA rRNA RNAi

tRNA

Which of the following is considered the adaptor molecule in protein synthesis?

tRNA

Which of the following is considered the adaptor molecule in protein synthesis? mRNA DNA tRNA rRNA

tRNA

Which protein-synthesizing RNA molecule carries an amino acid to the ribosome active site?

tRNA

Which protein-synthesizing RNA molecule carries an amino acid to the ribosome active site? rRNA tRNA mRNA siRNA

tRNA

The portion of the histone that can be acetylated is the

tail

The portion of the histone that can be acetylated is the head. body. tail. arm.

tail.

The two mechanisms to regulate protein ubiquitination are biochemical changes to

target proteins and E3 ligases

The two mechanisms to regulate protein ubiquitination are biochemical changes to

target proteins and E3 ligases.

Release factor hydrolyzes GTP during the __________ step of translation

termination

Cholesterol biosynthesis and cholesterol uptake is stimulated by

the binding of SREBPs to SREs and activating the transcription of cholesterol synthesizing genes.

Cholesterol biosynthesis and cholesterol uptake is stimulated by the binding of SREs to cholesterol uptake genes. the binding of SREBPs to SREs and activating the transcription of cholesterol synthesizing genes. increased levels of cholesterol binding to the SREBPs. the uptake of LDLs through the LDL receptor.

the binding of SREBPs to SREs and activating the transcription of cholesterol synthesizing genes.

Within the proteasome, the protease catalytic sites are located within A. the regulatory particle only B. both core and regulatory particles C. the core particle only

the core particle only

Within the proteasome, the protease catalytic sites are located within

the core particle only.

how is 5-fluorouracil used to treat cancer patients

the enzyme DPD modifies 5-FU which leads to its degradation and inactivation as a cancer drug inhibiting DNA synthesis and high doses must be given to reach an effective dose (80% is degraded)

Which metabolic process is the cause of albinism?

the enzyme tyrosinase working inefficiently

The stage of purine biosynthesis that ends with 5-aminoimidazole ribonucleotide synthesis is responsible for building which of the following?

the imidazole ring of the purine base

The glycan fragment shown below on the right has been cleaved by glycosylase enzymes and HPLC analysis has produced the peaks 1 to 6. Which of the following fragments corresponds to peak 1?

the one that looks like the entire thing

Fasting studies performed in an animal model showed that rates of muscle protein hydrolysis change dramatically in the first seven days. In the first seven days. In the first 24 hours, the rates of protein hydrolysis were low, but then increased substantially for the next 3-4 days. By day 5, the rates of protein hydrolysis decreased to much lower levels for the duration of the fast. Given that there are no known regulatory mechanisms controlling muscle protein hydrolysis, how can you explain this pattern?

the pattern results from the maintenance of blood glucose levels during the first days of the fast. Glucose comes initially from glycogen degradation, but once glycogen is depleted, gluconeogenesis in the liver and kidneys occurs using amino acids from protein hydrolysis as the source of carbon. After the brain adapts to using ketones as a component of its energy needs, flux through gluconeogenesis decreases, and protein is spared until survival is the highest priority

Leptin receptor activation and insulin receptor activation have

the same effect on neuronal signaling in POMC (decreased appetite, increased energy expenditure, increased alpha-MSH) and NPY/AGRP neurons (increased appetite, decreased energy expenditure, decreased NPY/AGRP)

The term greenhouse effect refers to the idea that

the temperature of the earth is increasing because of heat trapped by gases such as CO2

The term greenhouse effect refers to the idea that the global production of plant matter is increasing. the capacity of the earth's plant matter to store CO2 is increasing. the temperature of the earth is increasing because of heat trapped by gases such as CO2. tropical rainforests are gradually being eliminated.

the temperature of the earth is increasing because of heat trapped by gases such as CO2.

Given that glutamate dehydrogenase has a Km of 1 mM and deltaG of +30 kJ/mol, glutamate dehydrogenase only operates best when

there is a high concentration of nitrogen available.

Given that glutamate dehydrogenase has a Km of 1 mM and mc108-1.jpgG of +30 kJ/mol, glutamate dehydrogenase only operates best when there is a high concentration of nitrogen available. there is a low concentration of nitrogen available. operates regardless of nitrogen concentration. Not enough information is given.

there is a high concentration of nitrogen available.

LDLs are referred to as "bad cholesterol" because

they are the blood serum particle with the highest concentration of cholesterol.

LDLs are referred to as "bad cholesterol" because they are the blood serum particle with the highest concentration of cholesterol. they contain cholesterol that is not as easily degraded. their cholesterol has no specific cellular purpose other than to form plaques. LDL levels surge after meals high in fat

they are the blood serum particle with the highest concentration of cholesterol.

Transketolase requires the coenzyme

thiamine pyrophosphate

Crystal violet stains Gram-positive bacteria because Gram-positive bacteria have a(n)

thicker peptidoglycan layer that collapses, trapping the crystal violet molecules.

Crystal violet stains Gram-positive bacteria because Gram-positive bacteria have a(n) outer membrane layer that collapses, trapping the crystal violet molecules. thinner peptidoglycan layer that allows the crystal violet molecules to pass through the pores. thinner peptidoglycan layer that collapses, trapping the crystal violet molecules. thicker peptidoglycan layer that collapses, trapping the crystal violet molecules.

thicker peptidoglycan layer that collapses, trapping the crystal violet molecules.

Gram-negative bacteria resist staining with crystal violet because Gram-negative bacteria have a(n)

thinner peptidoglycan layer that does not retain the crystal violet molecules.

Base excision repair removes which kind of bases?

those damaged by ROS

Base excision repair removes which kind of bases? mismatched methylated those damaged by ROS uracil

those damaged by ROS

The two key elements of the oriC include

three 13-bp repeats and four 9-bp repeats with enriched A-T pairs.

Which eicosanoid regulates blood vessel constriction?

thromboxane

Which eicosanoid regulates blood vessel constriction? prostaglandin prostacyclin thromboxane leukotriene

thromboxane

When amino acids from dietary proteins enter a cell, how are they able to enter the urea cycle?

through conversion to aspartate

How does biological fixation convert nitrogen to ammonia?

through the ATP-dependent process catalyzed by nitrogenase complex

In the chloroplasts, photosystem I (PSI) and photosystem II (PSII) are known to localize in different regions. PSI generally localizes in the __________, whereas PSII localizes in the __________.

thylakoid lamellae; thylakoid grana

In the chloroplasts, photosystem I (PSI) and photosystem II (PSII) are known to localize in different regions. PSI generally localizes in the __________, whereas PSII localizes in the __________. thylakoid lumen; thylakoid grana thylakoid lamellae; thylakoid grana stroma; thylakoid lumen thylakoid grana; thylakoid lamellae

thylakoid lamellae; thylakoid grana

Five nucleotide bases are found in nucleic acids. Name a pyrimidine/purine in that order.

thymidine/adenine

Which enzyme activity would be reduced if fluorodeoxyuridine-5'-monophosphate is present?

thymidylate synthase

Five nucleotide bases are found in nucleic acids. Name a pyrimidine/purine in that order.

thymine/adenine Pyrimidine: thymine, guanine, uracil Purine: adenine, cytosine

Using common nomenclature, name the following fatty acid. trans 18:1 (mc063-2.jpg10) cis 18:1 (mc063-3.jpg10) trans 18:1 (D8) cis 18:1 (mc063-5.jpg8)

trans 18:1 (D8)

Using common nomenclature, name the following fatty acid.

trans 18:1 (delta 8)

One disadvantage of using partial hydrogenation in the food industry is that

trans fats are formed during the process.

The phrase trans-acting factors is short for __________ factor protein.

transcription

The phrase trans-acting factors is short for __________ factor protein. translation transcription transunion transform

transcription

What are sigmafactors in RNA synthesis?

transcription factors

The __________ is driven directly by photons from the sun in photosynthesis light reactions.

transfer of electrons

What step in the process of translation of a nascent protein into the lumen of the ER requires an input of energy provided by the hydrolysis of GTP?

transfer of the signal peptide sequence into the translocon

What step in the process of translation of a nascent protein into the lumen of the ER requires an input of energy provided by the hydrolysis of GTP? binding of SRP to the signal peptide sequence release of SRP from the ribosome transfer of the signal peptide sequence into the translocon binding of SRP to the SRP receptor

transfer of the signal peptide sequence into the translocon

What enzyme class in the nonoxidative phase of the pentose phosphate pathway allows for the reaction of ribose-5-phosphate and xylulose-5-phosphate to produce glyceraldehyde-3-phosphate and seduheptulose-7-phosphate?

transferase

What enzyme class in the nonoxidative phase of the pentose phosphate pathway allows for the reaction of ribose-5-phosphate and xylulose-5-phosphate to produce glyceraldehyde-3-phosphate and seduheptulose-7-phosphate? isomerase hydrolase transferase oxidoreductase

transferase

Enzyme activities after PPAR signaling were monitored in both liver and skeletal muscle. When compared with activities before PPAR signaling, which of the following would have been found to increase in liver but NOT in skeletal muscle?

transketolase

Which enzyme does penicillin target in bacteria?

transpeptidase

Which enzyme does penicillin target in bacteria? mc033-1.jpg-lactamase peptidase transpeptidase mc033-2.jpg-galactosidase

transpeptidase

The expected outcome in adipose tissue after exposure to glucagon would be an increased

triacylglycerol hydrolysis

The expected outcome in adipose tissue after exposure to glucagon would be an increased

triacylglycerol hydrolysis.

The expected outcome in adipose tissue after exposure to glucagon would be an increased glucose uptake through GLUT4. glycerol synthesis. fatty acid uptake from lipoprotein particles. triacylglycerol hydrolysis.

triacylglycerol hydrolysis.

Only __________ can be synthesized in plants, fungi, and bacteria by the shikimate pathway.

tryptophan, tyrosine, and phenylalanine

Only __________ can be synthesized in plants, fungi, and bacteria by the shikimate pathway

tryptophan, tyrosine, phenylalanine

Ubiquitin contains which of these following?

turns that connect one secondary structure to another α helices antiparallel β sheet

A knockout mutation in the X-inactive specific transcript (XIST) lncRNA of mice would result in

two active X chromosomes in female mice.

A knockout mutation in the X-inactive specific transcript (XIST) lncRNA of mice would result in one inactivated X chromosome in female mice. the inhibited transcription and translation of X-linked proteins. two inactive X chromosomes in female mice. two active X chromosomes in female mice.

two active X chromosomes in female mice.

A lack of energy balance may lead to

type 2 diabetes

Whipple surgery, sometimes carried out on patients battling pancreatic cancer, is one of the most demanding surgeries to perform. It involves removing parts of the pancreas and rerouting aspects of the digestive track. Which of the following side effects may result from Whipple surgery?

type 2 diabetes

A lack of energy balance may lead to type 1 diabetes. a BMI value of less than 30, indicating obesity. type 2 diabetes. increased skeletal muscle breakdown.

type 2 diabetes.

In the figure below of the E. coli ribonucleotide reductase enzyme, which amino acid is important in the R2 subunit?

tyrosine

In the proposed mechanism for the E. coli ribonucleotide reductase reaction, a ______________ radical in the R2 subunit is reduced by Cys439 in the R1 subunit.

tyrosine

In the proposed mechanism for the E. coli ribonucleotide reductase reaction, a ______ radical in the R2 subunit is reduced by Cys439 in the R1 subunit.

tyrosine

In the proposed mechanism for the E. coli ribonucleotide reductase reaction, a ________ radical in the R2 subunit is reduced by Cys439 in the R1 subunit.

tyrosine

Complete the following pathway of the degradation of UMP.

uracil, N-carbamoyl-beta-alanine

Which of the following is NOT a fate of the carbon atoms from amino acid catabolism? A. Pyruvate B. Citrate cycle C. Urea cycle D. Acetyl-CoA

urea cycle

What purine metabolite is the likely cause of gout?

uric acid

A difference between FA synthesis and mc098-1.jpg-oxidation is that FA synthesis __________, whereas beta-oxidation __________.

uses acyl-ACP; uses acyl-CoA

Both methicillin and penicillin are inactive when exposed to

variant transpeptidase

Both methicillin and penicillin are inactive when exposed to transpeptidase. transpeptidase and mc039-1.jpg-lactamase. b-lactamase. variant transpeptidase.

variant transpeptidase.

Which of the following oligosaccharides are found in high abundance in some types of vegetables and have been known to cause intestinal distress?

verbascose

The following fatty acid __________ be metabolized by __________. will; being run through the b-oxidation pathway until the double bond is isomerized for the pathway to continue will not; not being recognized by b-oxidation pathway enzymes will not; entering the pathway as normal but eventually inhibiting the enzymes will; being run through the b-oxidation pathway as normal

will; being run through the b-oxidation pathway until the double bond is isomerized for the pathway to continue

The following fatty acid __________ be metabolized by __________.

will; being run through the beta-oxidation pathway until the double bond is isomerized for the pathway to continue

Bidirectional replication is defined as replication

with 2 replication forks

Under what conditions does cyclic photophosphorylation take place?

with high levels of NADPH

Under what conditions does cyclic photophosphorylation take place? in an acidic stroma with high levels of NADPH with high levels of ATP with an inactivated photosystem I

with high levels of NADPH

Bidirectional replication is defined as replication

with two replication forks.

Bidirectional replication is defined as replication with one replication fork. with two replication forks. that unwinds the DNA completely to replicate. that replicates the DNA in a given direction.

with two replication forks.

Describe the experimental evidence for a eukaryotic "purinosome" and what the results in Figure 18.10 panel F reveal about the purine biosynthetic pathway.

wo human enzymes TrifGART and FGAMS are required for steps 2-5 of the E. coli purine biosynthetic pathway. Using purine-depleted media to induce high levels of purine biosynthesis, it was observed that both of these enzymes co-localize to the same subcellular localization in human cells grown in culture. These punctate foci were observed using fluorescence microscopy with different fluors for each enzyme (GFP and OFP) in a merged overly (panel F in Figure 18.10), providing strong support for purinosomes

If an alteration occurred in the activity site of the R1 subunit of E. coli ribonucleotide reductase that allowed dATP to bind but precluded ATP binding to the site, a possible result would be that ribonucleotide reductase

would be decreased when dATP bound both the activity and specificity sites.

Which of the following is a common breakdown product between AMP and GMP degradation to uric acid?

xanthine

In humans, allopurinol can be used as treatment for gout because this drug is a competitive inhibitor of

xanthine oxidase

The complete deactivation of which of the following would destroy the ability of a cell to generate uric acid?

xanthine oxidase

In humans, allopurinol can be used as treatment for gout because this drug is a competitive inhibitor of

xanthine oxidase.

In analyzing the intermediates formed during the conversion of IMP to GMP and AMP, which molecule would one expect to find in the conversion to GMP but NOT in the conversion to AMP?

xanthosine-5'-monophosphate

In analyzing the intermediates formed during the conversion of IMP to GMP and AMP, which molecule would one expect to find in the conversion to GMP but NOT in the conversion to AMP? xanthosine-mc128-1.jpg-monophosphate adenylosuccinate 5-phosphoriboxylamine 5-aminoimidazole ribonucleotide

xanthosine-mc128-1.jpg-monophosphate

would an insulin receptor that has an increased affinity for insulin be a thrifty gene

yes, because even at low insulin levels, fatty acid synthesis and lipogenesis would increase fat stores

would a hyperactive lipoprotein lipase on the plasma membrane in adipose cells be a thrifty gene

yes, because it would scavenge as much lipid as possible from circulating lipoprotein particles to increase fat storage in adipose tissue

Olestra

zero-calorie food substitute containing fatty-acid side chains covalently linked to sucrose. The taste of olestra is similar to dietary fats.

Which of the following is the product of uracil degradation?

β-alanine

Which of the following is a photosynthetic light harvesting pigment?

β-carotene

Which of the following is a photosynthetic light harvesting pigment? FeS cluster NADPH plastoquinone β-carotene

β-carotene


Kaugnay na mga set ng pag-aaral

Chapter 3 - Business in the Global Economy

View Set

USAFA History 300 GR2 Terms (Fall 2018)

View Set

Photography basics/exposure quiz

View Set

España: Las Comunidades Autónomas

View Set